Download as pdf or txt
Download as pdf or txt
You are on page 1of 100

All of us are/were television, internet or computer games addicts at some

point of times in our lives. It is up to us to convert this kind of addiction


into distraction or a utility e.g., I was a TV addict during early days of my
life, but today it is more or less a utility for me. I use TV, Movies etc., for
recreation purposes when I am mentally tired. Infact these two elements
(TV shows and Movies) work as energy boosters for my body.

Scientifically frames in a movie or a TV show change TELEVISION


faster that the functioning rate of our brain. That’s why Let’s begin with TV, ‘DOORDARSHAN’ which have the capability of
our real thinking power decreases up to a large extent converting a normal human being into ‘Buddhu’ or an ‘idiot’. That’s why
while watching these. Actually our brain works only some of us also call it ‘Buddhu Baksa’ or ‘idiot box’. For one kind of people
over those directions reaching to it via our eyes while this can be the source of entertainment through reality shows, serials etc.,
watching a movie or a TV show. Thus, keeping other while for others it provides entertainment through games like IPL, IHL,
portions of our brain at minimum functional level, this FIFA, Grand slams, ICC world cups etc.
in turn provides relief up to a large extent to our tired Thus, something or the other always visible on the screen and pulling you
functional brain. On the other hand, watching TV and away from your books just like spider webs. For a person like my son, it was
Movies may affect our body in negative manner also. even a bigger problem because he (and many more like him) equally fits in
Their over exposure is always hazardous to our eyes both the categories. Likewise his younger cousin says that he used to be
and other body parts. glued to the television every time Virat stepped out to bat, just like a normal
cricket fan. First watching a match and then the guilt of wasting time is a
The journey to IIT is too long. It seems like walking on the top of a
great distraction for him.
moving object which is rough, slippery and narrow too. Walking on
such an object’s top requires ultimate skills, courage, consistency and To overcome this kind of problem, you really need to have burning
determination as key players. One ‘slip’ can easily deviate you from the desire to hit your target. Once you are involved with such attractions, it
track. The word ‘slip’ here signifies ‘Distraction’. There will be all will be hard to come back. Suppose you decided to sit for an hour in front of
kinds of things like TV, movies, internet, phone, friends, ‘Bad’ habits the TV and ended up watching a 3 hour T20 match. This in turn followed by
(smoking, drinking etc.,), crushes/infatuations, the pressure of looking wasting another hour in discussing the outcomes of match or feeling guilty.
good, the desire of being noticed in social circle etc., which can be Everybody understands the fact that it is almost impossible to be passive
categorised to distractions or stumbling block for your progress. emotionally during this tenure of 2 years. There will be ups and downs
Sometimes, a mode of recreation also becomes a distraction. signifying good and bad times during this tenure. These up and downs are
in fact the ‘Testing Times’ for us. I consider these times as the eliminators as
THE REAL STUMBLING BLOCKS these are the indicators of our success and failures. What would give you more
From the example given above, it must be clear in your mind that happiness – a good rank in JEE or India lifting the World Cup 2015 in
controlled exposure to gadgets like TV, internet etc., may not distract cricket? Think about this question and your heart would give good rank in
you, but uncontrolled exposure certainly distract you just like a JEE as the answer.
hydrogen bomb which demolished your studies completely. Hence, I You can, or rather should, surely watch programs of your interest but it is
strongly suggest every JEE aspirant to analyse and remove from his better to fix up a time for TV and try to follow it honestly. Be stiff with
path such stumbling blocks which can be one of the following or yourself. Build the road map of your priorities in such a way to get
something else maximum help from it in ensuring your success.
INTERNET CRUSHES AND INFATUATIONS
Internet entered in our life as a time saving connecting tool which provides Crushes and infatuations are the part of every adolescent’s life.
manifold help in increasing our exposure towards the real world. Although Attraction to opposite sex at this age is very common, but at the
internet has something for everybody, but amazingly everyone among us is using same time its proper incorporation in life is essential. At such
it for the same purpose i.e. Social Networking means everybody is running times, when each minute has its own value and requires proper
towards social networking through many apps like Facebook, Twitter, Instagram planning, don’t even think of having a girlfriend or a boyfriend
etc., including you, your parents, your siblings and even your pets may have an for time pass. Their presence is acceptable only if, they do not
account. Your worth is judged by the number of friends or followers you have. deviate you from what you are supposed to do? Crushes may keep
You are called COOL, if you have a good collection of songs or movies else you on changing every month i.e., you can fall in love 24 times or
stand nowhere. For most people, social networking is not a requirement, but just more in these 2 years, but you won’t allowed to appear 24 times to
an addiction. crack JEE. So you have two options i.e., either learn to handle both
I realised it when I lost my account for a month due to some reason. In real sense, the things simultaneously or become a saint until you crack the
when you don’t have anything to post, you don’t even expect a comment also. Just JEE.
try to deactivate your account once, then only you will realise the worth of life In my opinion the second option is better as at your age
without such social networking. In real sense it is much – much better than the nobody is mature enough to handle both the things
one you are living with Facebook or Instagram. In a very recent research, this was simultaneously.
disclosed that addiction of this kind of social media is more prone in those
persons, which have some sort of insecurities in their relations. The solution of ATTENTION SEEKERS
these kinds of distractions lies in your determination and commitment for Seeking attention may also be considered as a distraction. See, if
success. So, I prefer the usage of internet could be the best, if students use it for looking good makes you feel ‘confident’, then it is fine, but if it
• The purpose of clearing their study related doubts
makes you feel ‘acceptable’, then it is wrong. Try to please
• For online tutorials/classes everyone with your result and not with your so called personality.
• For reading e-books My perception is, in your age personality keeps on changing in
• For exploring new horizons in different fields etc. days or weeks. Accept yourself as a future star, who can do much
Likewise, if their parents use it for more than the rest without following their ways. Love yourself
• Office automation,
and respect your uniqueness. Let people judge you, but what you
• Accounting purposes,
think about yourself matters the most. Just decide what appeals
• Financial operations like bank accounts,
you the most - a ‘show man’ or a ‘show stopper’.
• Online shopping i.e., e–commerce, and All the above written distractions are those common factors which
• So many other useful things. everyone among us faces or faced in his/her life. You are not alone
the only victim of these.
MOBILE PHONE
I consider it a dragon that always stays in your pocket and literally grabs the peace REMEMBER!
of your life. I always prefer its limited usage as it left me unnecessarily exhausted. For clearing JEE, you need not at all isolate yourself
It is suggestive to switch off your mobiles when you are studying. Try to avoid its from your friends, family or any modes of recreation.
use in your ‘relaxation time’ too i.e., your own time in which Rather, it is very necessary to give them the time they
• You can have a chat with your family members or deserve. Play for half an hour or talk to your
• You can play some outdoor games, or
parents or play with your siblings or watch TV,
• You can take a walk in the park to rejuvenate your exhausted body
chat, dance, sing, sketch etc., in short, do whatever,
• You can listen to some music to re-energize yourself.
which makes you forget that you are up for a job, but
Simply, this means the time of your personal usage, which must be used for
with a restriction. In the early days of my career, I had
constructive purposes. Some of you may be of the opinion that the usage of
a student who was very fond of cricket. He got his
mobile phone is necessary to increase and maintain their social circle. But as per
selection to IIT with a marvelous rank of 325. Just
my understanding an increase in ‘social circle’ does not necessarily means an
imagine, he was playing a cricket match 4 days before
increase in number of ‘friends’. Likewise, its size has inverse relation with your
his IIT-JEE, in which he got this rank. This was a
success.
lesson for me regarding distractions. If we analyse
REMEMBER! this incident properly, we can easily understand the
The day you compromise with your success for them, they will difference between stumbling blocks and utilisation
compromise on their ‘friendship’ for you. In simple words, they are of above written distractions with focused approach.
not your true friends rather they want to gain something from your That’s why I mentioned above, do whatever you like,
success. Hence, when you make compromises with your success but be focused in your approach. See how beautifully
they may leave you. So limit your social circle only to the people, Abraham Lincoln connected these distractions with
who know the worth of your time. destiny

“‘Always bear in mind that your resolution to succeed is


more important than any other one thing.”
Surface Chemistry
ADSORPTION
G
The accumulation/adhesion of atoms/ions/molecules of gas, G
All solids adsorb gases to some extent. Adsorption is not
liquid or solid (adsorbate) at the surface rather than the whole very pronounced unless an adsorbent possesses a large
mass of a substance (adsorbent) is called adsorption. surface area for a given mass.
G
It proceeds with a negative value of ΔH (exothermic),
ΔS (decrease in entropy) and ΔG (spontaneous). Types of Adsorption
Distinction between Adsorption and Absorption
Physisorption
Adsorption Absorption
The adsorbate and adsorbent are held together by weak
It implies concentration at the It implies penetration into the body of van der Waals’ forces of attraction.
surface only. the solid. G
It is reversible and not specific in nature.
It is a rapid process and In this, the equilibrium is reached G
Rate of physisorption is directly proportional to ease of
equilibrium is maintained in a slowly. liquefaction of adsorbate. Also, it increases with
short time. increase in surface area of adsorbent.
Note Both adsorption and absorption often take place side by side and it is G
Rate of physisorption increases with increase in
difficult to distinguish between these two processes. pressure upto a certain extent and decreases with
G
Mc Bain introduced the general term sorption, which includes increase in temperature.
both the processes.
Chemisorption
Characteristics of Adsorption G
The adsorbate and adsorbent are held together by
chemical bond. It is irreversible and highly specific. It
G
It is a specific and selective phenomenon, refers to the existence increases with increase in surface area of adsorbent.
of a higher concentration of any particular component at the G
Rate of chemisorption increases with increase in
surface of a liquid or a solid phase.
temperature till the activation energy is achieved.
G
For adsorption, ΔG < 0, ΔS < 0 and ΔH < 0. Hence, the adsorption
is associated with decrease in enthalpy of the system. Thus,
adsorption is always an exothermic process.
Rate

G
The atoms or molecules of a solid surface have unbalanced or
residual attractive forces on the surface, which can hold
adsorbate particles together. Thus, the adsorbed atoms or
molecules can be held on the surface of a metal by physical van
der Waals’ forces or chemical forces due to residual valence bond.
T

4
Isotherms Related to Adsorption
(I) Freundlich Adsorption Isotherm
x x 1
It gives the variation of rate of adsorption with pressure, = kp 1/ n ; ( n > 1) or log = log k + log p
m m n
T3
(T1>T2>T3)

x T2 x
m log m 1
m= n
T1
C=log k

p log p

1 x 1 x
When = 0, = constant (i.e. independent of pressure). When = 1, = kp (rate of adsorption is directly proportional to pressure)
n m n m
• It fails at high pressure.
• It is also applicable to adsorption by a solid in solution, in which pressure is replaced by concentration (C ).
x x 1
= kC 1/ n or log = log k + logC
m m n
(II) Langmuir Adsorption Isotherm
Langmuir derived an adsorption isotherm on theoretical considerations based on kinetic theory of gases. x
m
The Langmuir adsorption isotherm is represented by the relation.
x ap
= (a and b are two Langmuir parameters)
m 1 + bp
x a x Pressure ( p)
At high pressure, = . At low pressure, = ap
m p m
At low pressure, x/m linearly increases with p. At high pressure, x/m becomes constant, i.e. the surface is fully covered and change in pressure has no
effect and no further adsorption takes place.

Applications of Adsorption
G
Adsorbents like silica and aluminium gels are used to remove moisture to control humidity.
G
Noble gases are separated by adsorbing them on coconut charcoal at different temperatures due to different degree of
adsorption of noble gases.
G
Coloured impurities are removed from a solution by adsorbing them with the help of animal charcoal.
G
Froth floatation process for the concentration of sulphide ores is based on adsorption.
G
In the creation of completely vacuum space etc.

1. The area of interface depends upon the (c)


1 a
= +
p
(d)
1 b
= +
1
⎛ x⎞ b a ⎛ x⎞ a ap
I. size of the particles of bulk phase. ⎜ ⎟ ⎜ ⎟
⎝ m⎠ ⎝ m⎠
II. size of the particles of other than bulk phase.
III. shape of the particles of bulk phase. 3. The characteristics of an adsorption process are given
Which of the above statements is/are correct? below.
(a) Only I (b) Only II I. It is an exothermic process.
(c) Only III (d) I and III II. Adsorption increases with increasing
2. If x is the mass of adsorbate adsorbed per unit mass of temperature.
m III. The bond involved in the chemisorption process
adsorbent, p is the pressure of the adsorbate gas and a may be covalent or ionic.
and b are constants, which of the following represents IV. The material on the surface of which adsorption
‘Langmuir adsorption isotherm’? takes place is called adsorbate.
Which of the above statements are correct?
(a) log ⎛⎜ ⎞⎟ = log ⎛⎜ ⎞⎟ + log p (b)
x a 1 x 1 b
= +
⎝ m⎠ ⎝ b⎠ a m ap a (a) I and III (b) II and IV (c) I and IV (d) I and II

5
4. Freundlich adsorption isotherm is given by the Codes
x A B C D
expression = k ⋅ p1/ n . Some conclusions that can be
m (a) p q r s
drawn from this expression are given below. (b) p s q r
(c) q p r s
1
I. When = 0, the adsorption is independent of (d) p r q s
n
pressure. 8. Which one of the following graphs represents
1 Freundlich adsorption isotherm?
II. When = 0, the adsorption is directly
n
proportional to pressure.
1 x (a) log x (b) log x
III. When = 0, versus p graph is a line parallel to m m
n m
x-axis.
1 x log p log p
IV. When = 0, plot of versus p is a logarithmic
n m
type curve.
(c) log x (d) log x
Which of the above statements are correct? m m
(a) I and IV
(b) I and II log p log p
(c) I and III
(d) I, II and III
9. At the equilibrium position for the process of
5. Plot of log x against log p is a straight line inclined at adsorption, the correct relation is
m
an angle of 45°. When the pressure is 0.5 atm and (a) ΔH > 0
Freundlich parameter, k is 10, the amount of solute (b) ΔH = TΔS
adsorbed per gram of adsorbent will be (log 5 = 0.6990) (c) ΔH > TΔS
(a) 1 g (b) 2 g (d) ΔH < TΔS
(c) 5 g (d) 2.5 g Hint At equilibrium, ΔG = 0
6. Which among the following statements are correct with ΔH − TΔS = 0, Δ H = TΔS.
respect to adsorption of gases on a solid?
10. 1 g of charcoal is placed in 100 mL of 0.5 M CH3 COOH to
I. The extent of adsorption is equal to Kp n
form an adsorbed monolayer of acetic acid molecule and
according to Freundlich isotherm.
(1 + bp) thereby, the molarity of CH3 COOH reduces to 0.49.
II. The extent of adsorption is equal to Surface area of charcoal = 3.01 × 102 m2 / g. The
ap
according to Langmuir isotherm. surface area of charcoal adsorbed by each molecule of
ap acetic acid is
III. The extent of adsorption is equal to
(1 + bp) (a) 5 × 10 −9 m 2
according to Langmuir isotherm. (b) 4 × 10 −9 m 2
IV. The extent of adsorption is equal to Kp1/ n (c) 5 × 10 −19 m 2
according to Freundlich isotherm. (d) 6 × 10 −19 m 2
V. Freundlich adsorption isotherm fails at low Hint Number of acetic acid molecules adsorbed
pressure. = 1 × 10 −3 × 6.023 × 10 23
where, K, a and b are constants and p is pressure.
= 6.023 × 10 20
Choose the correct option.
(a) I and III (b) III and IV ∴ Area of 1 molecule of acetic acid
(c) II and III (d) II and IV 3.01 × 10 2
=
7. Match the Column I with Column II and select the 6.023 × 10 20
correct option from the codes given below. = 5 × 10 −19 m 2

Column I Column II 11. Which is not correct regarding the adsorption of a gas
A. Adsorption p. ΔH always negative on surface of a solid?
(a) Enthalpy and entropy change is negative
B. Freundlich isotherm q. High specificity
(b) Adsorption is more for some specific substance
C. Chemisorption r. x = K ⋅ p1/ n( n > 1) (c) On increasing temperature, adsorption increases
m progressively
D. Adsorption at equilibrium s. ΔH = TΔS (d) It is a reversible reaction
Hint Adsorption decreases with increasing temperature.

6
CATALYSIS
A substance that alters the rate of a reaction without being Step II Reactant molecules are adsorbed at the surface of the
consumed in the reaction is called catalyst and the catalyst.
phenomenon is known as catalysis. Step III Chemical reaction occurred at the surface of catalyst
through the formation of intermediate.
Types of Catalysis Step IV Products formed are desorbed making the surface
(a) Homogeneous catalysis Both reactants and the catalyst available for new reaction.
are in same phase.
e.g. 2SO2 (g) + O2 (g) ⎯→ 2SO3 (g) [Catalyst– NO(g)] Shape Selective Catalysis (Zeolites)
(b) Heterogeneous catalysis The reactants and the catalyst
are in different phase. The catalytic reaction depends on size of reactants, products
and pore sizes of catalysts.
e.g. N2 (g) + 3H2 (g) ⎯→ 2NH3 (g) [Catalyst– Fe(s)]
e.g. Zeolites (microporous aluminosilicates in which some Si-atoms
2SO2 (g) + O2 (g) ⎯→ 2SO3 (g) [Catalyst– Pt(s)] are replaced by Al-atoms). ZSM-5, a zeolite directly converts alcohol
(c) Positive catalysis When a catalyst increases the rate of to gasoline.
reaction. Enzyme catalysis Enzymes are biochemical catalysts and
e.g. 2H2 + O2 ⎯ ⎯⎯Pt (s)
→ 2H2O [Catalyst - Pt(s)] catalyse a specific chemical reaction in plants and animals.
(d) Negative catalysis When a catalyst decreases the rate of
An enzyme is a protein molecule and forms colloidal solution in
reaction, e.g.
water.
Alcohol( l )
2Na 2SO3 (s) + O2 (g) ⎯ ⎯⎯⎯ ⎯→ 2Na 2SO4 (s) G
Enzymes are highly efficient, highly specific, highly active
[Catalyst-alcohol]
under optimum temperature range (298-310 K), highly active
(e) Auto-catalysis Sometimes during a reaction either of the
under optimum pH (5-7).
products acts as catalyst and this phenomenon in which a
chemical reaction is catalysed by product of the reaction
G
Enzyme activity is increased in the presence of activators
is known as auto-catalysis. and coenzymes and it decreases in the presence of inhibitors
e.g. CH3COOC2H5 + H2O ⎯⎯→ CH3COOH + C2H5OH or poisons.
[Auto-catalyst-CH3COOH]
G
Enzyme catalysis proceeds through the formation of
enzyme-substrate complex according to lock and key
mechanism, in which the substrate acts as the closed lock
Adsorption Theory of Heterogeneous
`
and enzyme acts as the key.
Catalysis E +
Enzyme
S
Substrate
E- S complex
( Concept of its
It is based on theory of intermediate complex formation and existence was proposed

`E- P complex ` E + P
involves the following continuous steps: by Charles A. Wurtz in 1880)

Step I Reactant molecules are diffused to the surface of the


catalyst.
Applications of Catalysis in Industry
S.No. Industrial process Catalyst used
1. Acetic acid from acetaldehyde by oxidation with air V2O 5
2. Bosch process for the manufacture of H 2 Fe 2O 3 + Cr2O 3
3. Contact process for the manufacture of H 2 SO 4 V2O 5, Pt
4. Deacon’s process for the manufacture of Cl 2 CuCl 2
5. Dehydrogenation of alcohol to aldehyde Cu
6. Haber’s process for the synthesis of NH 3 Fe 2O 3 or Fe + Mo
7. Lead chamber process for the manufacture of H 2 SO 4 NO
8. Manufacture of vegetable ghee Ni
9. Methanol from water gas and H 2 ZnO, Cr2O 3
10. Ostwald’s process for the manufacture of HNO 3 Pt gauze
11. Polymerisation of alkenes H 3 PO 4 on kieselguhr
12. Preparation of O 2 from KClO 3 MnO 2
13. Synthetic petrol from coal (Bergius process) Iron oxalate

7
12. The steps of adsorption mechanism are given below. 17. Consider the following statements. Identify the true (T)
I. Adsorption of reactant molecules on the surface or false (F) statements and choose the correct option
of catalyst. given below.
II. Diffusion of reactant at the surface of catalyst. I. A catalyst remains unchanged chemically and
III. Desorption of reaction products from the catalyst physically at the end of the reaction.
surface. II. The activity of enzymes is increased in the
IV. Occurrence of chemical reaction on the catalyst’s presence of certain substances known as
surface through the formation of an intermediate. coenzymes or activators.
Arrange the steps in the correct order and choose the III. Acetic acid formed during hydrolysis of ester acts
correct option. as an induced catalyst.
(a) I, III, IV, II (b) II, I, IV, III (c) I, II, IV, III (d) I, II, III, IV IV. Combination of N2 and H2 in the presence of Fe as
13. Match the reactions given in Column I with Column II a catalyst is an example of heterogeneous catalysts.
Codes
representing corresponding processes and select the
I II III IV I II III IV
correct option from the codes given below. (a) T F T F (b) F F T T
Column I Column II (c) F T F T (d) T T F F

A. 4 NH 3( g ) + 5 O 2( g ) ⎯ ⎯
⎯→ Pt( s )
p. Homogeneous 18. Which of the following reactions is an example of
4NO( g ) + 6 H 2O( g ) catalysis heterogeneous catalysis?
Fe( s ) (a) O 3( g ) + O( g ) ⎯⎯
Cl
→ 2O 2( g ) (gas phase)
B. N 2( g ) + 3H 2( g ) ⎯ ⎯⎯→ 2NH 3( g ) q. Haber’s process
NO( s ) (b) 2CO( g ) + O 2( g ) ⎯NO
⎯⎯→ 2CO 2( g )
C. 2SO 2( s ) + O 2( g ) ⎯ ⎯⎯→ 2SO 3( g ) r. Ostwald’s process
O
Codes ⏐⏐ H 2SO 4
A B C A B C (c) CH 3 ⎯ C ⎯ OC 2 H 5( l) + H 2O( l) ⎯ ⎯ ⎯⎯→
(a) p q r (b) r q p O
(c) r p q (d) p r q ⏐⏐
CH 3 ⎯ C ⎯ OH( l) + C 2H 5OH ( l)
14. Which of the following statements are true about Cu, ZnO -Cr O
(d) CO( g ) + 2H 2( g ) ⎯ ⎯ ⎯ ⎯ ⎯⎯→ CH 3OH( l)
2 3

zeolite?
I. They are microporous structure of
19. On the basis of data given below, predict which of the
aluminosilicates. following gases show least adsorption on a definite
amount of charcoal?
II. Catalytic activity depends upon the pores and
cavities of the zeolites. Gas CO 2 SO 2 CH 4 H2
III. Zeolites are not found in nature. Critical temp./K 304 630 190 33

IV. ZSM-5 is a zeolite. (a) CO 2 (b) SO 2 (c) CH 4 (d) H 2


Hint Lesser the value of critical temperature of gases, lesser will
Identify the correct option.
be the extent of adsorption.
(a) I and III (b) I, II and III (c) I, II and IV (d) I and IV
20. In which of the following reactions, heterogeneous
15. Which reaction characteristics are changing by the catalysis is involved?
addition of a catalyst to a reaction at constant NO( g )
I. 2SO2 (g) + O2 (g) ⎯ ⎯⎯ ⎯→ 2SO3 (g)
temperature?
I. Activation energy II. Equilibrium constant II. 2SO2 (g) ⎯ ⎯⎯
Pt( s )
→ 2SO3 (g)
III. Reaction entropy IV. Reaction enthalpy III. N2 (g) + 3H2 (g) ⎯ ⎯⎯
Fe( s )
→ 2NH3 (g)
(a) Only I (b) Only II IV. CH3 COOCH3 (l) + H2 O(l) ⎯ ⎯⎯ ⎯→
HCl( l )
(c) I and II (d) All of these
CH3COOH(aq) + CH3OH (aq)
16. Identify the correct statement regarding enzymes. (a) II and III (b) II, III and IV (c) I, II and III (d) Only IV
(a) Enzymes are specific biological catalysts that can normally
function at very high temperatures (T ~ 1000 K) 21. Which one of the following is an example of
(b) Enzymes are normally heterogeneous catalysts that are homogeneous catalysis?
very specific in their action (a) Manufacture of sulphuric acid by contact process
(c) Enzymes are specific biological catalysts that cannot be (b) Manufacture of ammonia by Haber’s process
poisoned (c) Hydrolysis of sucrose in the presence of dilute hydrochloric
(d) Enzymes are specific biological catalysts that possess well acid
defined active sites (d) Hydrogenation of oil

8
CLASSIFICATION OF COLLOIDS
AND COLLOIDS AROUND US
Colloidal state is a state in which a substance is dispersed in another medium in
the form of very small particles having size in the range 1-100 nm.

Classification of Colloids
Based on the nature of interaction between the dispersed phase Gold Number
and the dispersion medium, colloidal solutions are classified Gold number of a protective colloid is the minimum weight of it in
as: milligrams which must be added to 10 mL of a substance (red gold sol)
(a) Lyophilic colloids These are liquid loving colloids and so that no coagulation of the gold sol takes place when 1 mL of 10%
are formed directly by mixing substances like gelatin, sodium chloride solution is rapidly added to it.
starch, etc. These are more stable and cannot be easily or
coagulated. Weight of the dried protective agent in milligrams, which, when added
(b) Lyophobic colloids These are liquid hating colloids and to 10 mL of a standard gold sol (0.0053 to 0.0058%), is just sufficient to
are usually formed by metals, metal sulphides etc. prevent a colour change from red to blue on the addition of 1 mL of 10%
sodium chloride solution, is equal to the gold number of protective
G
These are prepared by special methods like chemical 1
methods, Bredig’s arc method etc. colloid. Protective power ∝
Gold number
G
These are also called irreversible sols because once they are Note Gold number is a practical value, it cannot be obtained
precipitated, it is difficult to convert them back to colloidal theoretically.
solution.

Classification of Colloids based on Properties of Colloidal Solution


Types of Particles of Dispersed Phase (a) Tyndall effect Colloidal solutions show Tyndall effect
(a) Multimolecular colloids A large number of due to scattering of light by colloidal particles in all
atoms/molecules/ions aggregate together to form directions in space.
colloids, e.g. gold sol, sulphur sol. G
Tyndall effect is observed only under the following
(b) Macromolecular colloids Macromolecules like protein, conditions:
starch, enzymes etc., form colloidal solution. (i) The diameter of the colloids should not be much
(c) Associated colloids (micelles) These colloids behave as smaller than the wavelength of light used.
strong electrolyte at low concentration but behave as (ii) The refractive indices of the dispersed phase and the
colloids at high concentration due to association of dispersion medium should differ greatly in magnitude.
particles called micelles.
(b) Colour The colour of colloidal solution depends on
G
Micelles are formed only at a particular temperature called
wavelength of light scattered by dispersed particles, size
Kraft temperature (TK ) and concentration called Critical
and nature of colloidal particles and direction of
Micelle Concentration (CMC), e.g. cleansing action of soap.
observation.
(c) Brownian movement Colloidal particles show a
Methods of Preparation of Colloids continuous zig-zag motion which is independent of
(a) Chemical methods In these methods, chemical reactions nature of particles but small sized particles show faster
are carried out and products formed associated leading to movement.
the formation of colloidal solution, e.g. (d) Charge on colloidal particles Each colloidal particle of a
solution is either positively or negatively charged, the
As2O3 + 3H2S ⎯→ As2S3 (sol) + 3H2O
reason for this charge is best explained by preferential
(b) Bredig’s arc method Colloidal solution of metals like Au, adsorption of ions from solution.
Ag, Pt etc., are prepared by this method. Metal electrodes G
Positively charged sols Sols of haemoglobin, hydrated
are dipped in dispersion medium and high voltage is metallic oxides like Al2O3 ⋅ xH2O, CrO3 ⋅ xH2O, Fe2O3 ⋅ xH2O
provided. Metal atoms are vaporised due to intense heat, etc., basic dye like methylene blue, oxides like TiO2 etc.
which are then condensed to form colloidal solution. G
Negatively charged sols Sols of starch, gum, gelatin, clay,
(c) Peptisation Precipitates are converted back to colloidal
charcoal, acid dyes like eosin, congo red sols etc. Metals
solution by shaking it in dispersion medium with the help
like Cu, Ag, Au etc., metallic sulphides like As2S3 , Sb2S3 ,
of a small amount of electrolyte.
CdS etc.

9
G
For the coagulation of a negatively charged sol, the
Cleansing Action of Soap coagulation power of different positively charged ions is
The hydrophobic part (hydrocarbon) of soap is attached to dirt Al3 + > Ba 2 + > Na + .
particles, while the hydrophilic part is projected towards water Coagulation of lyophilic sols These sols having high solvation
forming micelles. Now, the micelle along with dirt particle is energy and charge are more stable than lyophobic sols.
pulled away towards water due to emulsification and washed G
These are coagulated by adding an electrolyte and a suitable
away. solvent like alcohol, acetone etc.
Theory of preferential adsorption of ions When two or more
ions are present in the dispersion medium, the colloidal Emulsions (Liquid-Liquid Colloidal System)
particles prefer to adsorb the ion common among them, e.g.
When two immiscible or partially miscible liquids are mixed
FeCl3 + H2O (hot) ⎯→ Fe2O3 ⋅ xH2O / Fe3 + and shaken together, one liquid is dispersed over another
(Positively charged) forming a colloidal system called emulsions. Emulsions are of
FeCl3 + NaOH ⎯→ Fe2O3 ⋅ xH2O / OH− two types such as:
(Negatively charged) (a) Oil dispersed in water (O/W) type Here, water acts as a
Helmholtz electrical double layer The charged colloidal dispersion medium, e.g. milk.
particles layer after preferential adsorption, further attracts (b) Water dispersed in oil (W/O) type Here, oil acts as a
oppositely charged ions from dispersion medium forming a dispersion medium. e.g. butter, cream etc.
second layer called Helmholtz electrical double layer. e.g.
[AgI] I − K+ Emulsifying Agents
(Layer-1, fixed layer) (Layer-2, diffused layer) Due to lyophobic nature, oil in water type of emulsion is
G
The potential difference between the fixed layer and diffused unstable and requires an emulsifying agent for their
layer is called electrokinetic potential or zeta potential. stabilisation. e.g. proteins, gums, long chain alcohols, heavy
metal salts, fatty acids, soaps etc.
Coagulation (Precipitation) of Colloidal
Particles Applications of Colloids
Coagulation of lyophobic sols These are coagulated by Colloids have many important applications such as:
electrophoresis, mixing two oppositively charged sols, boiling G
Electrical precipitation of pollutants present in smoke.
or persistent dialysis etc. G
Purification of drinking water by coagulating suspended
G
A positively charged ion causes the precipitation of a particles.
negatively charged sol and vice-versa. G
Colloidal medicines are more effective due to the large
Hardy-Schulze rule According to this rule, higher the charge surface area of colloids in dispersion medium.
on the flocculating ion added for coagulation, higher is the G
Cleansing action of soaps and detergents are due to micelle
coagulating power of the ion. formation.
G
For the coagulation of a positively charged sol, the Photographic film is an emulsion of light sensitive AgBr in
coagulation power of different negatively charged ions is gelatin.
[Fe(CN)6 ]4 − > PO43 − > SO42 − > NO3−

22. Some statements are given below. K4 [Fe(CN)6 ] (III) and NaCl (IV). Their coagulating
I. Soap molecules form micelle around the oil power should be
droplet. (a) I > II > III > IV (b) III > II > I > IV
II. Polar groups of soap molecules interact with water. (c) III > I > II > IV (d) IV > I > II > III
Hint According to Hardy-Schulze rule, greater the valency of
III. The negatively charged sheath around the
coagulating ion added, the greater is its power to cause
micelles prevent further aggregation.
coagulation.
IV. Hydrophobic group of the soap molecule interact [Fe(CN)6 ]4 − > PO 43 − > SO 42 − > Cl −
with water. III II I IV
Which of the above statements are true?
24. A freshly formed precipitate of SnO2 is peptised by a
(a) II, III and IV (b) I, II and IV
(c) I, II and III (d) III and IV small amount of NaOH. Their colloidal particles may
be represented as
23. In an electric field, the particles of a colloidal system (a) [SnO 2 ] SnO 32− ; 2Na+ (b) [SnO 2 ] Sn4 + ; O 2−
move towards cathode. The coagulation of the same (c) [SnO 2 ] Na + ; OH − (d) [SnO 2 ] Sn4 + ; OH −
sol is studied using K2 SO4 (I), Na3 PO4 (II),

10
25. The volume of colloidal particle, VC as compared to the 30. The gold numbers of a few protective colloids are given
volume of a solute particle in a true solution VS , could be below
VC VC VC VC
(a) ≈ 10 3 (b) ≈ 10 −3 (c) ≈ 10 23 (d) ≈1 x 0.005, y 3.5, z 40
VS VS VS VS The protective nature of these colloidal solutions
follows the order
26. Consider the following statements.
(a) z > x > y (b) x < y > z (c) z > y > x (d) x > y > z
I. The collodion is 4% solution of nitrocellulose in a
mixture of alcohol and ether. 31. Among the following, the surfactant that will form
II. Colloidal solution shows colligative properties. micelles in aqueous solution at the lowest molar
III. The coagulation power of Na+ is more than Ba2 + . concentration at ambient conditions is
IV. Detergents with high CMC are more economical (a) CH 3(CH 2 )15N + (CH 3 )3Br − (b) CH 3(CH 2 )11OSO 3− Na +
to use. (c) CH 3(CH 2 )6COO− Na + (d) CH 3(CH 2 )11N + (CH 3 )3Br −
Which of the above statements are correct?
32. Pick out the statements, which are relevant in the
(a) I and III (b) I and II
discussion of colloids.
(c) I and IV (d) II and IV
I. Sodium aluminium silicate is used in the
27. Match the Column I with the Column II and select the softening of hard water.
correct option from the codes given below. II. Potash alum is used in shaving wounds and as a
styptic in medicines.
Column I Column II
III. Artificial rain is caused by throwing electrified
A. Micelles p. 10 −4 to 10 −3 mol L−1
B. CMC of soaps q. Kraft temperature sand on the clouds from an aeroplane.
C. Lyophilic r. Sulphur sol IV. Deltas are formed at a place where the river pours
D. Multimolecular colloid s. Liquid loving its water into the sea.
Codes Choose the correct option.
A B C D A B C D (a) I and II (b) I and IV (c) II, III and IV (d) I, II and III
(a) p q r s (b) q p r s Hint In the softening of hard water, simple chemical substitution
(c) q p s r (d) p q s r of calcium salt with zeolite occurs so that calcium zeolite
precipitates out and hardness of water removes.
28. The amount of electrolytes required to coagulate the
given amount of AgI colloidal solution (negative 33. The dispersed phase in colloidal iron (III) hydroxide
charge) will be in the order and colloidal gold is positively and negatively charged
(a) NaNO 3 > Al 2(NO 3 )3 > Ba(NO 3 )2 respectively. Which of the following statement(s) is/are
(b) Al 2(NO 3 )3 > Ba(NO 3 )2 > NaNO 3 not correct?
(c) Al 2(NO 3 )3 > NaNO 3 > Ba(NO 3 )2 I. Coagulation in both sols can be brought about by
electrophoresis.
(d) NaNO 3 > Ba(NO 3 )2 > Al 2(NO 3 )3
II. Mixing of sols cause no effect.
29. Consider the following statements. III. Sodium sulphate solution causes coagulation in
I. Micelle formation by soap in aqueous solution is both sols.
possible at all temperatures. IV. Magnesium chloride solution coagulates the
gold sol more readily than the iron (III) hydroxide
II. Micelle formation by soap in aqueous solution
sol.
occurs above a particular concentration.
(a) I and II (b) Only III (c) II and III (d) I and IV
III. On dilution of soap solution, micelles may revert
to individual ions. 34. Bleeding due to a cut can be stopped by applying
IV. Soap solution behaves as a normal strong ferric chloride solution in the laboratory. This is due to
electrolyte at all concentrations. (a) coagulation of negatively charged blood particles by
Fe 3 + ions
Which of the above statements are correct? (b) coagulation of positively charged blood particles by
(a) I and II (b) II and III Cl − ions
(c) III and IV (d) II and IV (c) reaction taking place between ferric ions and the
Hint When soap is mixed with water above the critical micelle haemoglobin forming a complex
concentration (CMC), micelles are formed, i.e. colloids are (d) common element, iron, in both FeCl 3 and haemoglobin
associated.
Answers
1. (a) 2. (d) 3. (a) 4. (c) 5. (c) 6. (b) 7. (d) 8. (c) 9. (b) 10. (c)
11. (c) 12. (b) 13. (b) 14. (c) 15. (a) 16. (d) 17. (c) 18. (d) 19. (d) 20. (a)
21. (a) 22. (c) 23. (b) 24. (a) 25. (a) 26. (b) 27. (c) 28. (d) 29. (b) 30. (d)
31. (a) 32. (c) 33. (b) 34. (a)

11
1. Match the Column I with Column II. Select an The intercept on the log ⎜⎛ ⎞⎟ axis was found to be
x
appropriate answer from the codes given below. ⎝ m⎠
0.3010. The amount of the gas adsorbed per gram of
Column I Column II charcoal under a pressure of 0.5 atm is
A. Placing silica gel in water vapour. p. Enzymatic catalysis (a) 1.0 (b) 1.5 (c) 2 (d) 2.5
B. Placing anhyd. CaCl 2 in water q. Occlusion 6. Consider the following statements for the
vapour. electro-osmosis process of Fe(OH)3 sol.
C. Placing finely divided nickel in a r. Adsorption I. Sol particles move towards anode.
closed vessel containing H 2 gas.
II. Sol particles move towards cathode.
D. Shaking dil.KCl solution with s. Absorption
III. The dispersion medium move towards anode.
blood charcoal.
IV. The sol particles do not move in either direction.
E. Conversion of proteins into t. Negative adsorption
amino acids. Which of the above statements are true?
(a) III and IV (b) I and II (c) I and III (d) I and IV
Codes
Hint In electro-osmosis, the sol particles are prevented from
A B C D E A B C D E migration whereas the dispersion medium migrates in the
(a) t r s p q (b) t r p s q direction opposite to those of particles. Here, the medium is
(c) t r s q p (d) r t s q p negatively charged.
2. Which of the following is/are application(s) of 7. Match the following reactions given in Column I with
adsorption? the conditions given in the Column II and select the
I. Deionisation of water. correct option from the codes given below.
II. Gas masks.
Column I Column II
III. Hygroscopic nature of CaCl2 .
A. Haber’s process for the p. Platinised asbestos,
IV. Heterogeneous catalysis.
manufacture of ammonia at 573 K
(a) I and III (b) II and IV N 2( g ) + 3 H 2( g ) ⎯⎯→ 2NH 3( g )
(c) I, II and IV (d) I and IV
B. Ostwald’s process for the q. Platinised asbestos
3. Which of the following is/are not correctly matched? manufacture of nitric acid, or vanadium
4 NH 3( g ) + 5O 2( g ) ⎯⎯→ pentoxide ( V2O 5 ) at
I. Emulsion - Curd
673-723 K
II. Foam - Mist 4 NO( s ) + 6 H 2O( g )
III. Aerosol - Smoke 2NO( g ) + O 2( g ) ⎯⎯→ 2NO 2( g )
IV. Solid sol - Cake 4 NO 2( g ) + 2 H2O + O 2( g ) ⎯⎯→
(a) I and III (b) I and IV (c) I, II and IV (d) Only III 4 HNO 3( aq )
Hint Smoke is an aerosol (solid carbon particles dispersed in air). C. Contact process for the r. Finely divided iron,
4. Match Column I with Column II and select the correct manufacture of sulphuric acid molybdenum as
option from the codes given below. 2SO 2( g ) + O 2( g ) ⎯⎯→ 2SO 3( g ) promoter; at 200 bar
pressure and
SO 3( g ) + H 2SO 4 ( l) ⎯⎯→ 723 -773 K
Column I Column II
H 2 S 2 O 7( aq ) temperature
A. Argyrol p. Kala-azar
B. Antimony q. Intramuscular injection H 2S 2O 7( aq ) + H 2O( l) ⎯⎯→
C. Colloidal gold r. Stomach disorder 2 H 2 SO 4 ( aq )
D. Milk of magnesia s. Eye lotion
Codes
Codes A B C A B C
A B C D A B C D (a) p q r (b) r p q
(a) s p r q (b) s p q r (c) r q p (d) p r q
(c) q r p s (d) r q p s 8. Methylene blue, from its aqueous solution, is adsorbed
5. In an adsorption experiment, a graph between log ⎛⎜ x ⎞⎟ on activated charcoal at 25°C. For this process, the
⎝ m⎠ correct statement is
versus log p was found to be linear with a slope of 45°. (a) The adsorption requires activation at 25°C
(b) The adsorption is accompanied by a decrease in enthalpy

12
(c) The adsorption increases with increase of temperature (a) T1 < T2 < T3 (b) T3 < T2 < T1
(d) The adsorption is irreversible (c) T2 < T3 < T1 (d) T3 > T1 < T2
Hint Physical adsorption takes place with decrease in enthalpy,
thus exothermic process occurs. Being physical adsorption,
13. Which of the following statements are correct?
ΔH<0. I. Mixing two oppositely charged sols neutralises
their charges and stabilises the colloid.
9. When a beam of light is passed through a colloidal II. Presence of equal and similar charges on
solution, it is colloidal particles provides stability to the
(a) reflected (b) scattered (c) transmitted (d) absorbed colloids.
Hint When a beam of light is passed through a colloidal solution, III. Any amount of dispersed liquid can be added to
it is scattered. emulsion without destabilising it.
10. 20% surface sites have adsorbed N2 . On heating, N2 IV. Brownian movement stabilises sols.
gas is evolved from sites and were collected at Choose the correct option.
0.001 atm and 298 K in a container of volume 2.46 cm3 . (a) I and II (b) II and III (c) II and IV (d) II, III and IV
Density of surface sites is 6.023 × 1014 molecules/cm3
and surface area is 1000 cm2 . The number of surface 14. Which of the following factors are responsible for the
sites occupied per molecule of N 2 are increase in the rate of surface catalysed reaction?
(a) 12.046 × 1016 (b) 13.046 × 1016 I. A catalyst provides proper orientation for the
(c) 15.046 × 1016 (d) 12.046 × 10 6 reactant molecules to react.
II. Heat of adsorption of reactants on a catalyst helps
pV 0.001 × 2 .46 × 10 −3
Hint Number of moles of N2 = = reactant molecules to overcome activation
RT 0.0821 × 298
energy.
= 1.0 × 10 −7
III. The catalyst increases the activation energy of
Molecules of adsorbed N2 = 6.023 × 10 23 × 1.0 × 10 −7 the reaction.
= 6.023 × 1016 IV. Adsorption increases the local concentration of
Total surface sites available = Number of sites/cm 2 × area
reactant molecules on the surface of the catalyst.
= 6.023 × 1014 × 1000 = 6.023 × 1017 Select the correct option.
(a) I and II (b) I and III (c) II and IV (d) I, II and III
Surface sites at which N2 was adsorbed

=
20
× 6.023 × 1017 = 12 .046 × 1016
15. 10 g of gelatin is required to be added to 100 cm3 of a
−4

100 standard gold solution to just prevent its precipitation


by the addition of 1 cm3 of 10% NaCl solution to it.
11. The gold number of some colloidal solutions are given
Hence, the gold number of gelatin in mg is
below
(a) 10 (b) 1.0
Colloidal solution Gold number (c) 0.1 (d) 0.01
A 0.01 Hint NaCl solution prevents coagulation when gelatin is added
B 2.5
10 −4
C 20 = × 10 = 10 −5 g
100
The protective nature of these colloidal solutions So, gold number of gelatin in milligram
follow the order
= 10 −5 × 1000 = 10 −2 = 0.01
(a) C > B > A (b) A > B > C (c) A = B = C (d) B > A > C
16. Match the Column I with Column II.
12. Freundlich adsorption isotherm is represented at
temperatures T1 , T2 and T3 , Column I Column II
(Catalyst) (Industrial product)
T1
A. V2O 5 p. High density polyethylene
T2 B. Ziegler-Natta q. Polyacrylonitrile
x
m T3 C. Peroxide r. NH 3
D. Finely divided Fe s. H 2 SO 4

p Codes
A B C D A B C D
Arrange the temperature in the increasing order as (a) s q p r (b) p q s r
shown in the graph. (c) s p q r (d) s p r q

Answers
1. (c) 2. (c) 3. (c) 4. (b) 5. (a) 6. (a) 7. (b) 8. (b) 9. (b) 10. (a)
11. (b) 12. (b) 13. (c) 14. (c) 15. (d) 16. (c)

13
Paper 1
Integer Type Questions 3. A hydrated metallic salt A is light green in colour.
1. Consider the following reaction sequences, On careful heating, it gives a white anhydrous
residue B. B is soluble in water and its aqueous
CONH2 NH2 solution reacts with NO to give a dark brown
compound C. B on heating, produces a mixture of
Br2
I. products. How many products are formed?
H+
4. A C9 H20 hydrocarbon (linear molecule), on
NO2 NH2 monochlorination, gives a mixture of products. The
optically active products formed in the reaction is
conc. HNO3 (i) Fe, HCl, Heat
II. conc. H2SO4 (ii) OH
5. The mass of an electron is 9.1 × 10− 31 kg. If its
kinetic energy is 3.0 × 10−25 J and wavelength in
metre is 8.9 × 10−x , find the value of x.
Br MgBr COOH NH2
6.
-
The following reaction is non-spontaneous at 25°C,
1
III.
Mg (i) CO2 SOCl2
Cu2 O(s) 2Cu (s) + O2 (g); Δ rG° = 141 kJ
Et2O (ii) H3O+ 2
If Δ r S° = 75.18 JK − 1 and above (2170 + x) K, the
COOH NH2 reaction becomes spontaneous. Calculate the
value of x.
IV.
(i) aq. NaOH
7. The number of resonance structure(s) for A is
(ii) Br2 / KOH
OH
Among the above, how many reaction sequence(s)
is/are correct for the synthesis of amines? + Na A

2. Phenol associates in benzene to a certain extent to form


dimers. A solution containing 2.0 × 10− 2 kg of phenol 8. How many different Grignard reagents can be
directly used to prepare the following alcohol
in 1.0 g of benzene has its freezing point decreased by
alongwith other suitable compounds?
0.69 K. Calculate the degree of association of phenol
OH
and approximate it to the nearest integer value.
(Kf for benzene = 5.12 K molal −1 ) CH3
C2H5

14
One or More than One Options Correct Type COOH

9. Consider the following molecules, (a) COOH (b)


4 b
3 N O
a COOH
2 (c)
1N N N
COOH (d) O
H H
Pyridine Pyrrole Imidazole 14. Consider the following reaction steps,
(I) (II) (III) Roasting Roast without
Which one of the following statement(s) is/are correct? CuS ⎯ ⎯⎯⎯
⎯→ A ⎯ ⎯⎯⎯⎯⎯
⎯→ B
in air air
(a) (I) and (III) are modest Bronsted bases whereas (II) is not Which of the following statements is correct regarding
(b) In (III), Na is more basic than Nb the above reaction steps?
(c) When (III) is protonated in the presence of a strong acid, (a) It is a self reduction process
protonation occurs at C-2 (b) It involves disproportionation, Cu2S ⎯→ Cu + CuS
(d) All the nitrogen present in (I), (II) and (III) are (c) A is a mixture of Cu2O and SO 2 and B is a mixture of Cu
sp2 hybridised and SO 2
10. Consider the following reactions, (d) A is a mixture of Cu and SO 2 and B is CuSO 4
I. Me Me
H2
B
D2 /Pd/C
C 15. Ti3 + , V3 + , Fe3 + and Co 2 + afford a large number of
Pd+BaSO4 tetrahedral complexes while Cr3 + never. The correct
A
reason for this is
(i) Na+liq. NH3 D2 /Pd/C
II. Me Me D E (a) Cr 3 + imparts high crystal field splitting with different
(ii) EtOH
A ligands
Which of the following statements is/are correct? (b) In Cr 3 + system, crystal field stabilisation energy plays a
(a) B is cis-but-2-ene and D is trans-but-2-ene deciding role for the formation of tetrahedral or
(b) B is trans-but-2-ene and D is cis-but-2-ene octahedral complexes
(c) C is meso form and E is racemic forms (c) The ionic radius of Cr 3 + is the largest among the other
(d) C is racemic forms and E is meso form M 3 + ions mentioned in the problem
11. With reference to the scheme given below, which of (d) Electronegativity of Cr 3 + is the largest among these
trivalent 3d-metal ions, hence, chromium prefers to be
the given statements about T, U, V and W is/are
associated with as many ligands as its ionic radius
correct? permits
O
16. Consider the following structure,
O

H3C T
LiAlH4
+
CrO3/H (CH3CO)2O (excess)
V U W
(a) T is soluble in hot aq. NaOH
(b) U is optically active
(c) Molecular formula of W is C10H18O 4
(d) V gives effervescence on treatment with aq.NaHCO 3
12. Which of the following statements is/are correct
about the given reaction?

MeCHO +[Ag(NH3 )2 ]+ + OH ⎯→ MeCOO − + Ag
Anion =
(a) The equivalent weight of MeCHO is 22
− Cation =
(b) Three moles of OH are required in the above reaction
(c) MeCHO acts as an oxidising agent Which of the following statements is/are correct
(d) [Ag(NH 3 )2 ] + gets reduced about tetrahedral voids in the given unit cell?
13. The major product of the following reaction is (a) Number of tetrahedral voids per unit cell is 2
(b) Number of tetrahedral voids per unit cell is 8
(i) OH (c) Number of tetrahedral voids is twice the number of
O
(ii) H3O atoms in the given unit cell
(d) Number of tetrahedral voids is equal to the number of
Cl atoms in the fcc unit cell
A

15
17. When HCN reacts with chlorine, it gives A. On Paper 2
further reaction with HCl, it readily forms B. B when
reacts with KNH2 , gives (RNHCN)(ClCN)2 and
R NH3+ Cl − .
Integer Type Questions
Using the above information, select the correct 1. The total number of isomers produced by prismane
statements about molecule B. on bromination.
(a) It is a cyclic molecule 2. Among the following, the number of reaction(s) that
(b) Geometry at carbon is triangular planar with sp2 produce(s) alcohol is/are
hybridisation of carbon
(i) THF, Δ
(c) Geometry at nitrogen is triangular planar with sp2 I. H—C N + C 2H5MgBr
(ii) H3O+
hybridisation of N
(d) Ratio of electrons α : π : lp is 3 : 1 : 4 (i) THF, Δ
II. CH2== O+PhMgBr
(ii) H3O+
18. Identify the final product from the following
transformation.
(i) THF, Δ
III. Ph Ph+MeMgBr
MeO (ii) H3O+
CN 2NaH H2SO4
A Product
MeI H2O Me (i) THF, Δ
IV. MeMgBr +
(ii) H3O+
MeO MeO O
(a) CN (b) COOH
3. When a trisubstituted unsymmetric benzene gets
converted into tetrasubstituted compound, then
how many derivatives can be obtained?
MeO
CHO
HO
OH 4. A piece of aluminium weighing 2.7g is titrated with
(c) (d)
75.0 mL of H2 SO4 (specific gravity 1.18 g mL− 1 and
24.7% H2 SO4 by weight). After the metal is
Matching Type Questions completely dissolved, the solution is diluted to
19. Match the complex with its geometry/magnetic 400 mL. Calculate the molarity of free H2 SO4
property. solution. If actual molarity is twenty times of
calculated molarity, then what is actual molarity?
Column I Column II
5. NaCl structure has a lattice parameter a = 400 pm.
A. [Ni(CO)4 ] p. Square planar The molar volume (in mL) of the lattice including all
B. [Ni(CN)4 ]2 − q. Tetrahedral the empty space is
C. [Cu(NH 3 )4 ]2 + r. Paramagnetic
6. Phosphorus belongs to nitrogen family. It has eight
D. [FeCl 4 ] 2 − s. Diamagnetic
electrons in the penultimate shell. It exists in X form.
Codes When this X combines with oxygen, it readily forms Y
A B C D A B C D which is a waxy solid having white colour. The
(a) q,r p,r p,s q,s (b) q,s p,s p,r q,r number of P ⎯ O ⎯P bonds in the Y is/are
(c) q,r p,r p,s q,r (d) q,s p,r p,s q,s 7. x and y are the number of times of HEM takes
place to open the chain of the compounds X and Y
20. Match the Column I with Column II. respectively.
Column I Column II (HEM = Hofmann Exhaustive Methylation)
A. Conversion of proteins p. Shape selective catalysis
into amino acids
B. Conversion of alcohols q. Enzymatic catalysis
N =X
into gasoline
C. Polymerisation of ethylene r. Zeiglar-Natta catalyst
D. Manufacture of margarine s. Heterogeneous catalysis
=Y
Codes N
A B C D A B C D
CH3
(a) q q,s r,s s (b) p q,s r,s q
Sum of x + y =
(c) q p,s r,s s (d) p r,s q,s s

16
8. What is the theoretical value of bond length in 13. Consider the following two important oxides of
H ⎯F, if rH and rF are 0.37Å and 0.72Å respectively? phosphorus.
Electronegativities of F and H are 4.0 and 2.1 O
respectively. If experimentally bond length of HF,
0.08 Å, is also increased, what is the total value in P P
O O O O
term of Angstrom?
O O
P P P P
One or More than One Options Correct Type O O O O
9. Which among the following is aromatic in nature? O
P
O O
P
O
(a) Azulene
O
(b) Furan Which of the following statements is/are correct?
(c) Cyclohepta-1, 3, 5-trienyl anion
(a) P4O 6 is formed by burning phosphorus in a limited
(d) Borazine
supply of air
10. What is the end product (D) of the following (b) P4O10 absorbs water from the air and becomes sticky
reaction? (c) P4O 6 is basic and hydrolyses in water
OH
(d) All of the above
s
+ H+ (i) BH3, THF/H2O/OH (i) NBS 14. For a closed (not rigid) container containing10 moles
A B C
(ii) H+ (ii) Base of an ideal gas fitted with movable, frictionless and
weightless piston operating such that pressure of
CHBr3 gas remains constant at 0.821 atm. Which graph
D represents correct variation of log V versus log T
t-BuOs
where, V is in litre and T in kelvin?
Br
(a) (b)
Br
Br Br (a) (b)
log V

log V
(c) (d) 45° 35°
log T log T
s
O
11.
RBr RBr
B Protic solvent C
DMF solvent

A
log T
log V

The compounds given are (c) (d)


45°
R
OH OR 45°
log T log V
15. The wave function of 2s electron is given by
I II 3 /2
1 ⎛ 1⎞ ⎛ r ⎞ − r / a0
Which of the following statements is/are correct? ψ2 s = ⎜⎜ ⎟⎟ ⎜⎜2 − ⎟ e
(a) The compound B is I 4 2π ⎝ a0 ⎠ ⎝ a0 ⎟⎠
(b) The compound B is II It has a node at r = r0 . What is the relation between r0
(c) The compound C is I
and a?
(d) The compound C is II
(a) r0 = 2 a0 (b) r0 = a / 2
12. The correct statement(s) regarding (c) r0 = 6a0 (d) r0 = 2 a
PH3 , AsH3 , NH3 , SbH3 and BiH3 is (are) 16. The dissociation constants of aniline, acetic acid
(a) Phosphine (PH3 ) has the highest boiling point and water at 25°C are, respectively, 3.83 × 10− 10 ,
(b) The compounds given are as we move from PH3 to BiH3 , . × 10− 5 and
175 1008
. × 10−14 . Calculate the
the molecular mass increases. As a result, the van der percentage hydrolysis of aniline acetate in a
Waals’ forces of attraction decreases decimolar solution.
(c) The boiling point of NH3 is higher than those of PH3 and (a) 55.62% (b) 50.00%
AsH3 (c) 45.62% (d) 54.95%
(d) NH3 and PH3 also act as a Lewis base

17
Paragraph I 18. Anti-addition takes place in
Consider the following reactions, (a) I, II and IV (b) III, IV and VI
(c) I, II and IV (d) III, IV and V
I
B
Me Me KMnO4 Paragraph II
II A mixture of CO and CO2 when treated with I2 O5 gives I2
H H C
OsO4/NaHSO3 vapours according to the following equation,
cis-but-2-ene
III 5CO + I2 O5 ⎯→ 5CO2 + I2
D
HCO3H/H2O I2 vapour was separated and treated with HClO4 and the
IV resultant HIO4 required 0.001 mole of glycerol for
E
MMPP + EtOH/H2O complete oxidation.
V After treatment with I2 O5 and removal of I2 , the mixture
F
H2O2 + OsO4 was treated with excess of 0.01 N NaOH solution and
finally this solution required 20 mL of 1N HCl to reach
VI
G
end-point using phenolphthalein as an indicator, followed
H2O2 + SeO2 by methyl orange as an indicator after the first end-point,
(MMPP= Magnesium monoperoxyphthalate) 10 mL of further HCl was consumed.
17. Which of the following are regioselective reactions? 19. The number of moles of CO present initially was
(a) I, II and IV (a) 0.001 (b) 0.002 (c) 0.005 (d) 0.01
(b) III, IV and VI
20. The total volume of NaOH used in the problem was
(c) I, II and IV (a) 30 mL (b) 300 mL
(d) III, IV and V (c) 60 mL (d) 600 mL

18
Answers with Explanation
Paper 1 Substituting these values, we get
1. (2) II and IV are the correct reaction sequences for the 1
3 × 10 − 25 = × 91 . × 10 − 31 × v 2
synthesis of amines but reaction (i) and (ii) not correct. 2
1/ 2
CONH2 NH2 ⎛ 2 × 3 × 10 −25 ⎞
or v =⎜ ⎟ . × 10 2 ms −1
= 812
⎝ 91. × 10 − 31 ⎠
Br2
(I) – Using de-Broglie relationship, we get
OH
h 6.6 × 10 −34
λ= =
Br MgBr COOH COCl me v . × 10 −31 × 812
91 . × 10 2

Mg (i) CO2 SOCl2 = 8.932 × 10 −7 m


(II) Et2O (ii) H3O+ 6. (6) At T = 25° C = 298 K, we have
ΔG = ΔH − TΔS
2. (1) Mass of phenol, W 2 = 2.0 × 10− 2 kg; K f = 512
. K molal −1;
141 = ΔH − 298 × 0.0758
Mass of benzene, W1 = 1.0 kg; ΔTf = 0.69 K
ΔH = 141 + 298 × 0.0758 = 163.6 kJ
Therefore, according to the equation,
1000 K f W 2 The temperature above which the reaction becomes
M 2 (observed) = spontaneous under standard conditions corresponds to
W1 ΔTf
ΔG = 0 and is given by
1000 kg −1 × 512
. K kg mol −1 × 2.0 × 10 −2 kg ΔH
= T =
. kg × 0.69 K
10 ΔS
10.240 × 10 −2 Substituting the values of ΔH and ΔS, it gives
= = 14840.58 × 10 −2 = 148.4 g mol −1
0.69 163.6 × 10 3 J
T = = 2176 K
Also, M 2 (calculated) for . JK −1
7518
C 6 H 5 OH = 12 × 6 + 1 × 5 + 16 + 1 = 94 g mol −1 Above 2176 K, the reaction will become spontaneous.
M (calculated) 94gmol −1 As T = 2176 K
Therefore, i = 2 = = 0.633
M 2 (observed) 148.4gmol −1
-
So, x = 6 or T = (2170 + 6) K
2C 6 H 5OH (C 6 H 5OH)2 Hence, x=6
Initial moles 1 mol 0
Moles after dissociation 1− α α /2 7. (5) OH O Na
– +

α α
Total moles after dissociation = 1 − α + = 1−
2 2 + 1/2 H2
1 − (α / 2 ) + Na
Therefore, i =
1 Sodium phenoxide
α = 2(1 − i ) = 2(1 − 0.633) = 0734
. −1
~
Phenoxide ion undergoes resonance and get stabilised.
Δ
3. (3) FeSO 4 ⋅ 7H 2O ⎯→ FeSO 4 s
O O Os
A B
(Light green) (Soluble in water)
s s
FeSO 4 ( aq ) + NO ⎯→ FeSO 4 ⋅ NO ( aq )
C s
Δ
2FeSO 4 ⎯⎯→ Fe 2O 3 + SO 2 + SO 3
B Three products are obtained
OH
Monochlorination
8. (3) Three Grignard reagents are used to prepare CH3
4. (3) Cl
+ as there are three different alkyl groups
C2H5
+
, — CH3 and —C2H5, respectively.
Cl Cl
There are three optically active products obtained from the 9. (a,c,d)
chlorination of n-nonane. r H
N N—
H+ H+
5. (7) The kinetic energy ( E K ) is expressed as (I) (III)
N rN N N
1
mv 2 EK =
2 H H H
Given that, E K = 3 × 10 −25 J and me = 91
. × 10 −31 kg But (II) is not protonated, hence the statement (a) is true.

19
Molecular mass ( M ) of CH 3 CHO = 44 gmol −1
b
Nb N
In (III) : M 44
a s + Equivalent weight ( EW ) = = = 22 gmol −1
N Na n -factor 2
H H 13. (c) It is an example of favorskii rearrangement reaction, since
So, N b is more basic due to the presence of lone pair of
the compound (A) is an α-haloketone. Base abstracts acidic
electron. Hence, the statement (b) is wrong.
α -H atom, followed by SN 2 displacement of Cl – to give a
3 N r N
H bicycloketone. Ring opening followed by acidification, gives
Statement (c) : 2 1+
s + cyclopentane carboxylic acid.
N H N
H H H
H H s s s
Statement (c) is true. α OH – Cl
O O O
Statement (d) : Due to resonance, all the N in I, II and III are
sp2 hybridised and hence, the statement (d) is true. Cl Cl H
10. (a,c) s
Me Me OH
syn-addition D2 +Pd/C
(I) Me Me H
of H2 syn-addition
A H
(cis)
H s
(i) H2O
B Me
H D (ii) H3Or
H D COOH COOH
H
Me
RoastingRoast
(Meso) 14. (a,c) CuS ⎯ ⎯ ⎯ ⎯
⎯→ Cu2O + SO 2 ⎯ ⎯ ⎯
⎯→ 6Cu + SO 2
C in air (A) without air ( B)
Birch H Me
reduction D2 +Pd/C It is a self reduction process. Oxides of unreactive metals
(II) Me Me
Anti-addition syn-addition (like those of Hg, Pb, Cu etc) are reduced by air/anion of ore.
A of H2 Me (trans) H Here, no external reducing agent is added.
D
Me Me 15. (b)
H D D H
16. (b,c)
D H H D
17. (a,b,d) Cl
Me Me
(+) (–)
C
N N
Racemic mixture
(E) C C
11. (a,c,d) CH3
Cl N Cl

(aq) NaOH 18. (b)


(Hot) C H
HO – MeO H H MeO MeO H Me
ONa+
H 3C s
(Soluble in aq. NaOH) C N NaH C N MeI C N
T
CH3

NaH
T + LiAlH4 HO OH
U
Optically inactive
Me Me Me
MeO MeO
(Plane of symmetry is present)
H2SO4 C N s
OCOCH3 H2 O Me—I C N
U+ (CH3 CO)2O
(Excess) OCOCH3
H3 C W Me Me
MeO
COOH

+ OH NaHCO3
H
U+ CrO3
COOH
CO2 19. (b) [Ni(CO)4 ]
H3 C V Ni = [Ar]3d 84s2
− 3d 4s 4p
12. (a,b,d) Oxidation CH 3CHO + 3O H ⎯→
Ni in free state =
CH 3COO − + 2e − + 2H 2O
Reduction [Ag(NH 3 )2 ] + + e − ⎯→ Ag + 2NH 3 ] × 2 3d sp3
− − Ni in[Ni(CO)4] =
CH 3CHO + 2[Ag(NH 3 )2 ] + + 3OH ⎯→CH 3 COO + 2H 2O
CO CO COCO
+ 2Ag + 4NH 3 (Tetrahedral)

20
It is diamagnetic in nature as all the electrons are paired. 2. (3) II, III and IV reactions produce alcohols.
[Ni(CN)4 ]2 −
s r (i) THF, Δ
Ni = [Ar] 3 d 8 4s 2; Ni 2 + = [Ar] 3d 8 CH2==O + Ph MgBr Ph CH2OH
(ii) H3O+
3d 4s 4p
Ni2+ in free state= OH
2 (i) THF, Δ
3d dsp Ph Ph + MeMgBr Me Ph
(ii) H3O+ Ph
Ni2+ in [Ni(CN) 4]2–=
– – – –
CN CN CN CN sr Me (i) THF, Δ Me
(Square planar) MeMgBr + Me
(ii) H3 O+
It is diamagnetic in nature as all the electrons are paired. O
[Cu (NH 3 )4 ]2 + OH
In reaction I, aldehyde is produced.
Cu = [Ar ] 3d 10, 4s1
Cu2 + = [Ar ] 3d 9 – + (i) THF, Δ Br
3d 4s 4p H —C N + C2H5MgBr C2H5—CH==O +Mg +NH 3
(ii) H3O+
OH
Cu2+ in free state = [Ar],
3. (3) CH3 CH3 CH3
3d sp3 CH3 CH3 CH3
Chlorination
Cu2+ in[Cu(NH3)4]2+= +
NH3 NH3NH3 NH3 Cl Cl
2− CH3 CH3 CH3
[FeCl 4 ]
Trisubstituted CH3
Fe = [Ar ] 3 d 6 4s 2; Fe 2 + = [Ar ] 3 d 6 unsymmetric benzene
Cl CH3
3d 4s 4p
+
Fe2+ in free state =
sp3 CH3
2+
Fe in[FeCl4]2– =
Three isomers
– – – –
Cl Cl Cl Cl 4. (4) Milliequivalent of Al = milliequivalent of H 2 SO 4 reacted
Tetrahedral
2.7
paramagnetic Milliequivalent of Al = × 1000 = 300
9
20. (c) A - q, B - p,s, C - r,s, D - s
Molecular weight 27
Conversion of proteins into amino acids is an enzymatic [Equivalent weight (EW) of Al = = = 9g]
catalysis. Conversion of alcohols into gasoline is a shape 3 3
selective catalysis. It is also an example of heterogeneous (Al ⎯→ Al 3 + + 3 e − )
catalysis. Zeigler-Natta catalyst is used in the polymerisation % by weight × 10 × d 247. × 10 × 118
.
Normality of H 2 SO 4 = =
of ethylene which is an example of heterogeneous catalysis. (N) EW 2 98 /2
Manufacture of margarine is an example of heterogeneous
= 5.95
catalysis.
Milliequivalent of H 2 SO 4 = N × volume in mL
Paper 2 = 5.95 × 75 = 446.25
1. (3) Milliequivalent of Al added = 300
Br Br Br Milliequivalent of H 2 SO 4 left after reaction = 446.25 − 300
Br2
and = 146.25
Solution is diluted to 400 mL
Br
Prismane Milliequivalent left after reaction
∴ NH 2SO 4 left =
Br Br Volume of solution after dilution (mL)
146 . 25
and = = 0.367 N
400
Br Br N 0.367
MH 2SO 4 left = = = 0183
. M
(+) (–) n 2
Racemate ∴ Actual molarity = 20 × 0183 . = 3.660 ~− 4M

There are three isomers produced by prismane on 5. (9) As it is NaCl structure. So, it exists in fcc lattice.
bromination. However, one of them is chiral and exists as a Number of atoms = NA
pair of enantiomers. NA
Number of unit cells = [Q 4 atoms in each unit cell]
4

21
Volume of 1mole lattice =
NA
× volume of unit cell Borazine is cyclic, planar and both N and B are sp2
4 hybridised. Each N has two non-bonding electrons in a
6.023 × 10 23 p-orbital and each B has an emply p-orbital giving a total
= × ( 400 × 10 −10 )3 cm3 = 9.6 mL ~
− 9 mL number of six delocalised π-electrons, hence aromatic.
4
6. (3) There are three P ⎯O ⎯P bonds in the Y molecule. H H H
Δ H H H s
P4 + 5 O 2 ⎯⎯→ P4O10 B H r sB H B H
N N N rN Nr Nr
It is cyclic metaphosphoric acid, s s
B B H B Bs H B r B H
HO H N H s N H N
r
P
O O H H H
O H
OH
P P H
HO O O B H
7. (5) N N
B B H
H N
CH3I Δ
+
N (excess) N –H2O N H

CH3 CH3
whereas cyclohepta-1, 3, 5-trienyl anion is an anti-aromatic
compound. It is cyclic, planar and 8 π-electron system
CH3I, Ag 2O
follows 4n π-electron rule.
s
s
HOC + H
N CH3 I r
H3 C Ag2O N N H
CH3
CH3 CH3 CH3 CH3 10. (a)
OH Hr
Δ +
r + H2O
–H2O (CH3)2N –
BH3/THF/H 2O/OH
+ +
So, x=3 Hydroboration
Similarly, 2 times HEM occur in case of oxidation
A

Allylic H+
NBS
N Br B H
⏐ OH
Base –HBr
CH3
So, y=2
CHBr3 + t-BuO–
Total x + y = 3 + 2 = 5 Br
C
8. (1) Given, rH = 0.37Å , rF = 0.72 Å ; χ F = 4.0 ; χ H = 2.1 C
Br
Ring
d A ⎯ B = rH + rF − 0.09 ( χF − χH ) expansion

= 0.37 + 072
. − 0.09 ( 4.0 − 2.1) = 0.92 Å
d A ⎯ B = 0.92 Å + 0.08 Å = 1 Å
Br
D
9. (a,b,d) Azulene is aromatic as resonance structure bears a
positive charge in the seven-membered ring, while the five 11. (a,d) The resonance structure of 2-naphthoxide ion (A) is
membered ring bears a negative charge making it similar to s
the aromatic cyclopentadienyl anion. It is cyclic, planar with s
O
10 π electron system.
r
s s A A′
s
r A is an ambident nucleophile and a better nucleophile than
r
A ′ because negative charge is on more electronegative
Furan is cyclic, planar and has 6 π-electron system. oxygen atom. In the presence of polar non-protic solvent
(DMF, dimethyl formamide), alkylation on oxygen atom is
s favoured to give product II (C).
s In the presence of protic solvent, hydrogen bonding with the
O O O O-atom occurs, thus reducing the nucleophilicity of oxygen.
r r
Therefore, C-alkylation is favoured to give product (B).

22
12. (c,d) Phosphine (PH 3 ) has lowest boiling point. As we move 18. (b) Syn-hydroxylation occurs in reactions I, II and V, whereas
from PH 3 to BiH 3, the molecular mass increases. As a result, anti-hydroxylation occurs in reactions III, IV and VI.
the van der Waals’ forces of attraction increases and the Mechanism of these reactions are:
boiling point increases regularly from PH 3 to BiH 3. With KMnO 4 (syn-addition) Os OH
However, the boiling point of NH 3 is higher than those of PH 3
and AsH 3 due to intermolecular hydrogen bonding in ammonia r s s s s
C C O O C C
but boiling point of NH 3 is lower than those of SbH 3 and BiH 3 KMnO4 OH OH
Mn
because higher van der Waals’ forces of attraction in SbH 3 and C C O O C C
BiH 3 compensate the increase in boiling point of NH 3. s
Thus, the boiling points of hydrides of group 15 elements OMnO3H OH
follow the order. With OsO 4 (syn-addition)
OH
PH 3 < AsH 3 < NH 3 < SbH 3 < BiH 3
O
13. (a,b) P4O 6 is formed by burning phosphorus in a limited C OsO4 C O OH
s C
O
supply of air.
Limited supply of C C O O C
P4 + 3O 2 ⎯ ⎯ ⎯ ⎯ ⎯ ⎯⎯→ P4O 6
air
OH
P4O 6 is acidic and hydrolyses in water, forming phosphorous with peracid (trans-addition)
acid. P4O10 absorbs water from the air or from other r
OH2 OH
compounds and becomes sticky.
P4O10 + 6H 2O ⎯→ 4H 3 PO 4 C C r C r C C
Per acid H3O H2O
14. (c) O OH
C C C C C
15. (a) The wave function of 2 s electron is
3/ 2
1 ⎛ 1⎞ ⎛ r ⎞ − r / a0 OH OH
ψ2 s = ⎜ ⎟ ⎜2 − ⎟e
4 2 π ⎝ a0 ⎠ ⎝ a0 ⎠ 19. (c) 5CO + I 2O 5 ⎯→ 5CO 2 + I 2
Thus, the probability of finding of 2 s electron at a point is I 2 + 2HClO 4 ⎯→ 2HIO 4 + Cl 2
2
⎡ 1 ⎛ 1 ⎞ 3/ 2 ⎛ r ⎞ − r / a0 ⎤ 2HIO4 + CH2—CH—CH2 2HCHO + HCOOH
ψ2 s =⎢ ⎜ ⎟ ⎜2 − ⎟e ⎥
⎢⎣ 4 2 π ⎝ a0 ⎠ ⎝ a0 ⎠ ⎥⎦ OH OH OH
3 2 OH OH OH
1 ⎛ 1⎞ ⎛ r ⎞ − 2r / a0
= ⎜ ⎟ ⎜2 − ⎟ e
32 π ⎝ a0 ⎠ ⎝ a0 ⎠ Number of moles of HIO 4 required = 2 × 0.001 = 0.002
Node is the point at which the probability of finding an 2 moles of HIO 4 = 1 mole of I 2 = 5 moles of CO
electron is zero. It means the value of ψ 2 s is zero when r = r0. (0.002) (0.001) ( 0.001 × 5 = 0.005)
3 2 Number of moles of CO present initially = 0.005
1 ⎛ 1⎞ ⎛ r ⎞ − 2r / a r
So, ⎜ ⎟ ⎜2 − 0 ⎟ e 0 0 = 0 ⇒ 2 − 0 = 0 20. (b) In the beginning, mixture of CO and CO 2 was given and
32 π ⎝ a0 ⎠ ⎝ a0 ⎠ a0
5 moles of CO 2 was formed by the reaction of 5 moles of CO
∴ r0 = 2 a0 with I 2O 5.
16. (d) The degree of hydrolysis ( x) of a salt of weak acid and a Let the number of mmol of CO 2 initially = n
weak base is given by ∴ Number of mmol of CO 2 = ( n + 5)
Kw 1008
. × 10 −14 CO 2 + 2NaOH ⎯⎯→ Na 2CO 3 + H 2O
x= = = 1.219
Ka × Kb . × 10 −5 × 3.83 × 10 −10
175 ( n + 5) ( n + 5)
Let the mmoles of NaOH = b
But, x cannot be greater than 1. This indicates that the
mmoles of NaOH left = b − 2( n + 5)
degree of hydrolysis in this case is very large and therefore,
the above formula, which is based on the assumption that x When phenolphthalein is used
is small, is not applicable. b − 2( n + 5) + ( n + 5) = 20
In the present case, we have to calculate x as follows: b − (n + 5) = 10 … (i)
Kw x2 When methyl orange is used, ( n + 5) = 10 … (ii)
Kh = ; Kh =
Ka × Kb (1 − x )2 From Eqs. (i) and (ii), we get
x Kw n=5
= = 1.219 or x = 0.5495 ⇒ mmol of CO 2 = 5
1− x Ka × Kb
∴ b = 30
i.e. per cent hydrolysis = 54.95%
⇒ mmol of NaOH = 30
17. (a) All the reactions are hydroxylation reactions. All of them
Thus, mmol of NaOH taken = 30
are stereospecific, but if one product is formed they are
regioselective. If more than one product is formed in major Volume of NaOH, 30 = 01 . ×V
amount, then they are non-regioselective. V = 300 mL

23
Paper 1
One or More than One Options Correct Type (a) A is more stable than B
(b) Both A and B show keto-enol tautomerism
1. Beryllium chloride exists in vapour as well as in solid
(c) A has acidic hydrogen whereas B has not
phase, choose the correct option for solid phase
(d) The enol-form of A and B are more stable than keto-form
beryllium chloride.
(a) Beryllium chloride has zero dipole moment 6. Consider the statements, ‘hydrolysis of NCl3 gives
(b) Lone pairs present on Cl - atoms are used for bonding with ammonia but hydrolysis of PCl3 gives phosphoric acid’
vacant orbitals of the Be-atoms and choose the correct option from the following.
(c) It exists in dimeric form of BeCl 2 (a) Because N is more electronegative than phosphorus
(d) It functions as a Lewis acid (b) Chlorine is more electronegative than phosphorus
2. Choose the correct feature(s) about ortho-phosphoric (c) Phosphorus has vacant 3d-orbitals
(d) N-atom in NCl 3 has spare orbital to accept lone pair of
acid from the following.
electrons of O-atom of water molecule
(a) It is tribasic acid
(b) It is syrupy liquid 7. The molar conductances at infinite dilution of
(c) Trisodium salts of this acid show basic nature in water HCl, NaCl and Na Z (sodium crotonate) are 425 ´ 10 -4 ,
(d) Phosphate ion is square pyramidal in shape 125 ´ 10 -4 and 80 ´ 10 -4 Sm2 mol -1 , respectively. The
specific-conductance of 0.001 M aqueous solution of
3. One mole of an ideal gas expands against a constant crotonic acid is 3.8 ´ 10 -3 Sm-1 . What would be the
external pressure of 1 atm from a volume of 10 dm 3 to a
dissociation constant of the acid?
volume of 30 dm 3 . What would be the work done by the
(a) 1.11 mol dm-3
gas in joules?
(b) 1.11 ´ 10 -5 mol dm-3
(a) - 2026 J (b) -2.026 J (c) 1648 J (d) -1947 J
(c) 121 ´ 10 -3 mol dm-3
4. Choose the correct statement(s) from the following (d) 0.11 ´ 10 -5 mol dm-3
options.
(a) Fluorine is the strongest oxidising agent 8. Consider the reactions given below,
Zn, 900 °C
(b) If negative valencies of a species increase after reaction, I. BCl3 ¾ ¾ ¾ ¾® P
it is known as reduction process
O
(c) Conversion of glucose into CO 2 is an exothermic process II. B ¾ ¾
2
®Q
(d) Hypo prefix in hypophosphorous acid indicates it will act as Electrolysis
an oxidising agent III. KBF4 ¾ ¾ ¾ ¾
¾® R
800 ° C
5. Consider the two structures A and B,
Identify P, Q and R in the option given below.
P Q R
(a) B B 2O 3 B
(b) B 2O 3 B B
(c) BH 3 B B 2O 3
A B
and choose the correct options from the following. (d) BH 3 H 3 BO 3 B

24
9. Phenol undergoes Reimer-Tiemann reaction to give 14. Consider the following sequence of the reaction,
salicylaldehyde. What will happen, when 4-nitrophenol Ph
is taken instead of phenol in this reaction? SO2Cl2
A
(a) Rate of reaction becomes slower
Me
(b) Product formed in this reaction becomes highly
electrophilic NBS
(c) Product formed in this reaction becomes highly nucleophilic
(d) Rate of reaction becomes faster B
10. Which of the following reaction /s may give tertiary Sum of the active hydrogen atoms present in A and B is
alcohol as a product P? 15. The complex compound K4 [Fe(CN)6 ] is 45%
COCl dissociated in 0.1 M aqueous solution at 27°C. The
(i) 2CH3MgBr osmotic pressure of the solution approximately is
(a) P
(ii) H3O+
16. A dipeptide on hydrolysis gives two amino acids, which
COCl are NH2 CH(R)COOH A and H2 NCH(R¢)COOH(B).

OH This dipeptide is also hydrolysed by leucine amino
(b) + HN P
H2O peptidase enzyme to give only (B).How many
(c) CH 3 COCl + urea ¾® P asymmetrical carbon atom(s) is/are present in the
structure of dipeptide?
(d) CH 3 CH(CH 3 )CH 2 Br ¾ ¾¾® P
H 2O 17. Consider the following transition metal complex
compounds, [Fe(CN)6 ]3 - , [Fe(CN)6 ]4 - , [Co (NH3 )6 ]3 + ,
Integer Type Questions NiCl3 (PMe3 )2 , [Co(CN)6 ]3 -
11. The energy corresponding to one of the lines in the How many of them show diamagnetic behaviour?
Paschen series for H- atom is 1816. ´ 10 -20 J. What
18. 0.45 g of pure crystal of Na2 C2 O4 × xH2 O was dissolved
should be the quantum number for the transition which
in water and diluted to 100 mL. A 25 mL portion of this
produces this line?
solution required 10 mL of a 0.05 M H2 SO4 solution to
12. The compound HOOC ¾ (CHOH)2 ¾ COOH is a chiral reach the end point. The value of x in the salt is?
compound. Calculate the number of optical isomers
exhibited by this compound. 19. What would be the pressure in atm that exerted by
one mole of CO2 gas at 40°C, confined to a volume of
13. Consider the following reactions, 0.107 dm3 ? Assuming that the
I. 2S(l) + Cl2 (g) ¾® P (l) Vc = 0.0957 dm3 , Tc = 304 K and pc = 7 atm
II. SO2 + PCl5 ¾® P + Q
20. How many inorganic compounds among the following
III. Fe(OH)3 +3SOCl2 ¾® P + Q + R exhibit octahedral geometry?
IV. CH3COOH + SOCl2 ¾® P + Q + R IF7 , SeF6 , BrF5 ,BCl3 ,XeF4 ,SF4
How many of these reactions produce SO 2 as byproduct?

Paper 2
1. The rate of the homogeneous gaseous reaction, 3. Consider the reaction,
2NO(g) + Cl2 (g) ¾¾® 2NOCl(g) is doubled, when the CHCl3 ¾ ¾¾
3 SbF
® CHF2 Cl ¾1070K
¾¾ ¾® A + 2HCl,
chlorine concentration is doubled but increases by a HF
factor of eight, when the concentration of both Product A is
the reactants are doubled. Overall order of the (a) F2 C ==CF2
reaction is (b) ClFC ==CFCl
(a) one (b) two (c) F2 C ==CFCl
(c) three (d) zero (d) F2 C ==CCl 2

2. When xenon hexafluoride is treated with water a 4. Choose the correct choice for the reaction,
product P is obtained with biproduct HF. Select the Na2 CrO4 + H2 SO4 ¾¾® P + H2 O
correct option regarding the product P. (a) It is a redox reaction in which green solution of [Cr(H 2O)6 ]3+
(a) It oxidises Pu2+ to Pu4 + in presence of acid is produced
(b) It forms xenate ion on reaction with aqueous alkali (b) Product P obtained in reaction has trigonal planar structure
(c) Both (a) and (b) (c) Product P is dimeric bridged tetrahedral ion
(d) None of the above (d) Dark blue colour is obtained in reaction

25
5. For a reaction, A -
B, k values at 40°C and 60°C are Paragraph Type
0.86 and 0.35, respectively. Choose the correct
prediction about this reaction. Paragraph I
(a) The reaction is non-spontaneous at 27°C In a unit cell, atoms A are present at all corners of lattice, B
(b) The reaction is spontaneous at 27°C atoms are present at alternate faces and all edge centres.
(c) The reaction does not favour at lower temperature than Atoms C are present at face centres left from B and at each
27°C
body diagonal at a distance of 1/4th of the body diagonal
(d) None of the above
from corner.
6. Oxalic acid ¾¾
D
® X (g) + Y(g) + Z(g), Y and Z, both are
11. A tetrad axis is passed from the given unit cell and all
polar and neutral, X is non-polar and acidic. Z gas is
the atoms touching the axis are removed. The possible
condensed and formed liquid having pH =7. The
formula of the compound left is
hybridisation states of X , Y and Z respectively are
(a) AB3 C 6 and AB4C 5 (b) A 3B6 C 7 and A 3B6C 5
(a) sp, sp2, sp3 (b) sp2, sp3, sp3d
(c) A 4 B5 C 8 and A 4 B5 C 7 (d) AB2 C and ABC 2
(c) sp, sp, sp3 (d) sp2, sp2, sp3
12. Total fraction of voids occupied is
7. Choose the correct option regarding the characteristics (a) 0.58 (b) 0.25
of sodium peroxide. (c) 0.48 (d) 0.86
(a) It liberates O 2 with cold water
(b) It turns into white colour on exposure to moist air Paragraph II
(c) It reduces chromic compounds to chromate CH2CH2CN
(d) All of the above
Cl
8. When 2-butene reacts with peroxy acids, the possible 2 eq.KNH2
product(s) may be respectively. B
Liq.NH3
(a) Meso, racemate –33ºC
(b) Racemate, meso A
(c) d, l-pair, Z-isomer
(d) Both (a) and (b) CH2(CH2)8CH2NHCH3
9. Consider the reaction,
O 2 eq.KNH2
Reduction D
Me Excess Liq.NH3 , –33ºC
A B
Me NH3
MeI/AgOH Cl
O C
Δ
C+D
13. The product B of the reaction would be
Choose the correct option.
CH2CH3 CH2CH3
(a) A is an aromatic compound
(b) B is an antiaromatic compound CN
(c) D is a Lewis acid (a) (b)
(d) None of the above CN
10. In the following reaction, the final product can be
prepared by two paths I and II. (c) (d)
Path I CN
Mg /ether (i) CO2
Δ CN
(ii) H3O+
CH3 CH3 CH3
Br MgBr COOH 14. The formation of D from C proceeds via, the formation
Path II NaCN of which of the following intermediate species?
CH2(CH2)8CH2NHCH3 CH2(CH2)8CH2NHCH3
+
H3O

CH3 CH3 (a) (b)


CN COOH

According to the above reactions, which option is


CH2(CH2)8CH2NHCH3 CH2(CH2)8CH2NHCH3
correct?
(a) Path I is facile
(b) Path II is facile (c) (d)
(c) Both paths are facile
(d) Neither I nor II is facile

26
Passage III Codes
Boron forms many compounds like boric acid, borax, and A B C D
inorganic graphite etc. Orthoboric acid contains (a) q,r p s,p p,s
triangular BO33 - units. In the solid, the B(OH)3 units are (b) q,r,s q,r,s q,r,s p,s
bonded through hydrogen bonding into 2D sheets with (c) s p r q
almost hexagonal symmetry. Borax is used in borax bead (d) q r p s
test. 19. Match the Column I with Column II.
15. The number of the H-atoms replaced from boric acid, Column I Column II
when it is dissolved in water is
(a) two (b) one Me
O3

C—CH—CH
(c) three (d) zero A. 3
Ag2O p. Reduction
Me
16. Boric acid can behave as a strong acid in the presence
of Hg2+, H2SO4

B. CH3—CH —CH

(a) anhyd. H 2 SO 4 (b) glycerol Ag2O q.
CH3—C—CH 3
(c) mannitol (d) All of these

Matching Type Questions


Red P + Hl r. Oxidation reaction
17. The complex given in Column I shows missing digit in C. CH3—C—CH3
its formula, which is given in Column II. Match the
Column I with Column II.
2 4 NH
D. CH3—CH—CH 2—C—CH3 OH/Δ
– s.
Column I Column II
Cl
A. K n [Fe(CN)6 ] diamagnetic p. n=2
-
B. [Ag(CN)n ] q. n=3 Codes
2-
C. Facial -[PtCl 3 Brn ] r. n=4 A B C D
D. [Pt(NH 3 )n ]Cl 4 (octahedral) s. n=6 (a) q,r q,r p p,s
(b) p q r s
Codes
(c) p,q q,s p,r p,s
A B C D A B C D
(d) s r p,s q
(a) s p q r (b) r p q s
(c) q p r s (d) p r q s 20. Match Column I with Column II and select correct
code for your answer.
18. Match the Column I with Column II.
(M = Central metal atom of molecule )
Column I Column II
Column I Column II
(Molecule) (Property)
A. p. Hyperconjugation
A. H2 O p. Largest M ¾H length
B. H2 S q. Largest H— M—H bond angle
q. All C-atoms are
B. C. H 2 Se r. Lowest boiling point
sp2-hybridised
D. H 2Te s. Two lone pairs

C. r. Aromatic Codes
A B C D
CH3 (a) r q s p

D. H3 C C s. Diamagnetic (b) q r s p
(c) p q r s
CH3
(d) q s r p

According to their multi-scale analyses, a single ion has an influence on millions of


water molecules, i.e. 10,000 times more than previously thought.

27
Answers with Explanation
Paper 1 5. (a,d)
1. (b,c) Beryllium chloride exists in vapour as well as in solid
phase.
In vapour phase, it exists in monomeric form
A B
Cl ¾ Be ¾ Cl
Due to the presence of a-hydrogen, they show tautomerism,
While in solid phase, it exists in dimeric form. i.e. show keto-enol tautomerism.
Cl Cl Cl
Enol form of both A and B have an aromatic ring. So, they are
Be 98º
82º Be 263 pm Be Be more stable than their respective keto-forms.
Cl Cl Cl H H
In this chain structure, the lone pairs present on Cl-atoms OH
participate in bonding with empty p-orbitals of Be-atom.
Vapour phase BeCl 2 acts as a Lewis acid because Be has
vacant p-orbitals to accept the lone pair of electrons.
Keto-form Enol-form
2. (a,b,c) The structure of ortho-phosphoric acid is OH
OH

P
HO OH
Enol-form
Due to the presence of three replaceable H-atoms, it is H H
tribasic in nature. Also, trisodium salts of it show three Keto-form
polarised negative ions in water hence it is basic too. In 6. (b,c) In NCl 3, nitrogen is more electronegative than chlorine.
aqueous solution, H-bonding predominates which gives it Furthermore, nitrogen, in NCl 3 does not have any spare orbital
syrupyness. to accept lone pair of electrons of O-atom of water molecule.
Structure of phosphate ion shows it is tetrahedral in shape. So, in the presence of H 2O, reaction looks like
Cl +

NCl3 + H2O H—N—Cl + OH

P –
O O
Cl
O– –
OH
3. (a) We know that,
V2 H Cl
w= - ò pd V = - p ( V2 -V1 ) 2H2O
V1 HOCl + H—N H—N + HOCl
= - (1 atm) (30 dm3 - 10dm3) = - 20 dm3 atm H Cl
But we have to get the result in Joules Ammonia Hypochlorous acid

As R = 8.314 JK -1mol -1 = 0.08206 dm3atm K -1mol -1 On the other hand, in PCl 3, phosphorus has available vacant
3d-orbitals to accept lone pair of electrons of O-atom of
8.314 JK -1 mol -1 water. Furthermore, Cl is far more electronegative than
1 atm =
0.08206 dm3 K -1 mol -1 phosphorous. Hence, hydrolysis takes place as
8.314 JK -1mol -1 Cl Cl Cl
\ w = - 20 dm3 atm = - 20 dm3 ´ H H
0.08206dm3 K -1mol -1 Cl—P + O O P—Cl HO—P + HCl
H H
– 2026.36 J Cl Cl Cl
4. (a,b,c) 2H2O
G Fluorine is the strongest oxidising agent because it has the OH
tendency to accept an electron to gain noble gas HO—P + HCl.
configuration.
G [Fe(CN)6 ]3- ¾® [Fe(CN)6 ]4 - , negative valency increases, OH
7. (b) Crotonic acid is formed as follows
which means, oxidation state decreases and hence, it is a
reduction process. HCl + NaZ ¾® NaCl + HZ
Crotonic acid
G During conversion of glucose into CO 2, a huge amount of
energy get released, so it is an exothermic process. where, Z = Crotonate ion
C 6H12O 6 ( aq ) + O 2( g ) ¾® 6CO 2( g )+ 6H 2O ( l)+ Energy Since, crotonic acid is a weak organic acid and HCl, NaZ
G Hypo prefix indicates that central atom of compound has and NaCl are strong electrolytes. Hence, we use
the minimum oxidation state, so it will act as a reducing Kohlrausch’s law of independent migration of ions.
agent.

28
L°m (HZ ) = L°m(HCl ) + L°m (NaZ ) - L°m (NaCl ) Due to the presence of two powerful electron withdrawing
groups, i.e. — CHO and —NO 2 in the product, it is highly
= ( 425 + 80 - 125) ´ 10 -4 Sm2 mol -1
electrophilic in nature.
= 380 ´ 10 -4 Sm2 mol -1
10. (a,d)
Also, at the given concentration of crotonic acid
MgBr
k 3.8 ´ 10 -3Sm-1
Lm = =
C 0.001 ´ 10 3mol m-3 C—Cl C—CH3 C—CH3
-4 2 -1 CH3MgBr MgBrCl
= 38.0 ´ 10 Sm mol (a) Cl
Lm 38.0 ´ 10 -4 Sm2 mol -1
a= = = 0.1 Acetophenone
L°m 380 ´ 10 -4 Sm2 mol -1
CH3MgBr
Using the Ostwald’s dilution law, OH
MgBr
Ca2 (1.0 ´ 10 -3mol dm-3 ) (0.10)2
Ka = = C—CH3
1- a (1 – 0.10) C—CH3
-5 -3 Mg(OH)Br + CH3 H3O+
= 111
. ´ 10 mol dm . CH3
3°-alcohol
8. (a) I. BCl 3 when reacts with zinc at very high temperature, P
i.e. at 900°C is reduced into Boron (B) forms.
BCl 3 ¾ Zn
¾®B
900° C P O H
II. Boron after reaction with O 2 gives B 2 O 3
C—Cl C—N
3O
4B ¾ ¾¾
2
®2B 2 O 3
Q (b) + H—N Cl
III. Electrolytic reduction of fused tetrafluoro borate in
molten KCl /KF at 800°C gives 95% pure boron O—H
KBF4 ¾Electrolysis
¾¾¾¾® B . C—N
800°C (95% pure)
R C— N
9. (a,b) OH OH
Cl

CHO
CHCl3 + KOH P
340 K
Phenol Salicylaldehyde O H
But, when we use 4-nitrophenol as a substrate in this
reaction, the rate of Reimer-Tiemann reaction becomes (c) CH3 —C—Cl + NH2—C—NH2 CH3—C—N—C—NH2
Urea
slower due to the presence of strong electron withdrawing Cl H
—NO 2 group which reduces the electron density of benzene
ring and slow down the rate of electrophilic reaction.
O—H

-
Reimer-Tiemann reaction
- -
CHCl 3 + OH CCl 3 + H 2O CH3—C—N—C—NH2 CH3—C—N—C—NH2
-
C Cl 3 ¾® ·· CCl 2 + Cl -
H Cl H
Dichlorocarbene P
– –
OH OK+ O
CHCl2 H H
KOH CCl2
Br–
–H2O (d) CH3—C—CH2 Br CH3—C—CH2 CH3—C—CH3
1,2-hydride shift
CH3 CH3 CH3
NO2 NO2 NO2 1º-carbocation 3°-carbocation
4-nitrophenol H2 O
H2O
OH O O H H
CHO CHO CH(OH)2 OH O
H2 O Δ –H+
CH3 —C—CH3 CH3—C—CH3
–H2O
CH3 CH3
NO2 NO2 NO2 3º-alcohol
Final product

29
11. (6) In the Paschen series, lower quantum number ( n1 ) is fixed R = gas constant
which is ‘3’. T = temperature of the solution
Given, . ´ 10 -20 J.
D E = 1816 \ From Eq. (i)
Let, the higher quantum number ( n2 ) be n. p = ( 01. mol dm-3 ) ( 0.08206 dm3 atm K -1mol -1 ) ( 300K )
Then, = 2.4618 atm
æ1 1ö or
DE = 2.18 ´ 10 -18 ç 2 - 2 ÷ J
è n1 n2 ø The complex is dissociated in aqueous solution. If a be the
degree of dissociation, then the dissociation reaction can be
æ1 1ö
Þ . ´ 10 -20 J = 2.18 ´ 10 -18 ç - 2 ÷ J
1816 represented as,
è9 n ø
K 4 [Fe(CN)6 ] -
4K+ + [Fe (CN)6 ]4 -
4a
On solving this quadratic equation for n, we get n = 6. 1- a a

12. (3) The structure of the given compound can be drawn as, Total number of moles after dissociation
H H = 1 - a + 4a + a = 1 + 4a
and number of moles without dissociation = 1
* *
HOOC—C——C—COOH Since, osmotic pressure is directly proportional to the
OH OH number of moles,
p 1 + 4a
There are two asymmetric atoms present in this compound. Hence, observed =
p normal 1
\ n =2
(1 + 4 ´ 0.45)
If the molecule can be divided into two equal halves which \ p observed = 2.4618 ´ (a = 0.45)
are mirror images of each other and if n is even, then number 1
of optically active isomers (a) = 6.89304 » 7 atm.
a = 2 ( n - 1) = 2 2-1 = 2 16. (2) The enzyme leucine amino peptidase hydrolyses
Number of meso-isomers (m) = 2 ( n/ 2 - 1) N-terminal amino acid.
æ2 ö
So, (B) is N-terminal and (A) is C-terminal amino acids.
ç -1÷ Therefore, the structure of dipeptide is
è2 ø
=2 =1
R′ R
\ Total number of optical isomers
= a + m = 2 + 1 = 3. H2N—CH—C—NH—CH—COOH
* *
Dipeptide
13. (2) I. 2S ( l ) + Cl 2( g ) ¾® S 2 Cl 2 ( l)
P
H3O+
II. SO 2 + PCl 5 ¾® SOCl 2 + POCl 3
P Q * *
III. Fe(OH)3 + 3SOCl 2 ¾® 3SO 2 + 3HCl + FeCl 3 H2N—CH—COOH + H2N—CH—COOH
P Q R
IV. CH 3COOH + SOCl 2 ¾® CH 3COCl + SO 2 + HCl R R′
P Q R (A) (B)
14. (2) Halogenation occurs at the a-position of the carbonyl Hence, there are two asymmetric C-atoms present in
group (C == O) via free radical mechanism. dipeptide.
Ph Ph 17. (3) In [Fe(CN)6 ]3 - , Fe is in + 3 oxidation state.
SO2Cl2
α H Valence shell electronic configuration (VSEC) of Fe 3+ is
Allylic chlorination
Me H Me Cl 3d 4s 4p
H (A) [Ar]
Allylic NBS VSEC of Fe 3+ in [Fe(CN) 6 ]3- =
bromination
Ph
3d d 2sp3
[Ar] ×× ×× ×× ×× ×× ××

α H 6CN–
144424443
Me Br
(B) Number of unpaired electron (n) = 1; Complex is
Hence, sum of active hydrogen in A and B is 1 + 1 = 2 . paramagnetic
15. (7) If the complex K 4 [Fe(CN)6 ] had not been dissociated, its In [Fe(CN)6 ]4 - , Fe is in +2 oxidation state
3d 4s 4p
normal osmotic pressure would have been given by the 2+
van’t Hoff equation. VSEC of Fe =[Ar]
p = CRT …(i) 3d d 2sp3
2+
where, p = osmotic pressure Fe in [Fe(CN)6 ]4– =[Ar] ×× ×× ×× ×× ×× ××

6CN–
144424443
C = concentration of the solution

30
n = 0 ; complex is diamagnetic 20. (3)
In [Co(NH 3 )6 ]3+ Co is in +3 oxidation state F
4s 4p F
3d F
3+ IF7 I F : Octahedral
VSEC of Co =[Ar]
F
F
3d d2sp3 F
3+
Co in [Co(NH3 )]3+= ×× ×× ×× ×× ×× ×× F
6NH3 F F
144424443 SeF6 Se Octahedral
F F
n = 0 ; complex is diamagnetic F
In [NiCl 3(PMe 3 )2 ] Ni is in +3 oxidation state
4s 4p F F
3d
3+ BrF5 Br : Octahedral
VSEC of Ni =[Ar] F F
F
3d sp3d 2
3+ Cl Cl
Ni in [NiCl3(PMe3)2] =[Ar] ×× ×× ×× ×× ×× ××
– BCl3 B : Trigonal planar
3Cl + 3PMe3
144424443
n = 3; complex is paramagnetic Cl

In [Co(CN)6 ]3- ; Co is in + 3 oxidation state


F F
3d 4s 4p XeF4 : Octahedral
3+ Xe
VSEC of Co ; [Ar] F F

3d dsp3
3+ 3–
Co in [Co(CN)6] = [Ar] ×× ×× ×× ×× ×× ×× F
– SF4 S F : Trigonal planar
6CN
144424443 F F

n = 0 ; complex is diamagnetic Paper 2


18. (5) mmol of H 2 SO 4 = 10 ´ 0.05 = 0.50 1. (c) We have the rate equation,
Total mmol of Na 2C 2O 4 . x H 2O = 0.50 ´ 4 Rate = [NO]a [Cl 2 ]b …(i)
= 2 (Both have same n-factor) where, a and b are the orders of reaction with respect to NO
Now, molar mass of pure crystal and Cl 2.
0.45 If a = 0, b = 1, then r = k1[Cl 2 ] …(ii)
= ´ 1000
2 If a = 1, b = 1, then r = k 2[NO][Cl 2 ] …(iii)
= 225 Doubling the concentration of both the reactants will increase
225 - 135 the rate by a factor of 4, which is not given.
Þ Moles of H 2O = =5
18 If a = 0, b = 2, then r = k1[Cl 2 ]2
\ Formula of pure crystal = Na 2C 2O 4 × 5H 2O .
Doubling the concentration of Cl 2 will increase the rate by a
19. (8) According to the law of corresponding states, factor of 4 which is not given.
pr =
8Tr
-
3 If a = 1, b = 1, then r = k 2[NO][Cl 2 ]2
3Vr -1 Vr 2
Doubling the concentrations of both the reactants will increase
8 (T / Tc ) 3 the rate by a factor of 8 but doubling the concentration of Cl 2
= - …(i)
3 ( Vm / Vm,c ) - 1 ( Vm / Vmc
, )
2 alone will increase the rate by a factor of 4, which is again not
given.
where, pr and Tr are reduced pressure and temperature
The other alternative for 3rd order reaction is
respectively.
and p/pC = pr , Vm / Vm,c = Vr , T/Tc = Tr a = 2, b = 1, so that r = k 3[NO]2[Cl 2 ]
pc, Vc , Tc = Critical constants for pressure volume and We see that this rate equation satisfies both the given
temperature, respectively. conditions. Hence, overall order of reaction = 2 + 1 = 3.
So, from Eq. (i) 2. (c) Xenon hexafluoride (XeF6 ) when treated with water, xenon
8 ( 313 / 304) 3
pr = - trioxide is obtained with a byproduct HF.
3( 0107
. / 0.0957 ) - 1 ( 0107
. / 0.0957 )2 XeF6 + 3H 2O ¾¾® XeO 3 + 6HF
8 ´ 103. 3 P
= - = 3.51 - 2.40 = 111
.
( 3 ´118
. ) - 1 (11 . )2 XeO 3 is a powerful oxidising agent so, it oxidises Pu2+ to
\ p = pr pc = 111
. ´ 7 atm = 7.77 atm Pu4 + in presence of acid.
» 8 atm. 3Pu2+ + XeO 3 + 6H+ ¾¾® 3Pu4+ + Xe + 3H 2O

31
Also, it reacts with aqueous alkali to give xenate and 6. (c)
perxenate ions.
- C—O—H Δ
XeO 3 + OH ¾¾® HXeO -4 CO + CO2 + H2O
- - C—O—H
2HXe O 4 +2 OH ¾® XeO 64 - + Xe + 2H 2 O + O 2 X Y Z
O
3. (a) Cl F Non-polar, X ; CO ; ¾ C ºº O; sp-hybridised C-atom.
SbF3 1070 K
2H—C—Cl 2H—C—Cl F2C — CF2+2HCl Polar, neutral, Y ; CO 2 ; O== C == O ; sp-hybridised C-atom.
HF
O
Cl F Polar neutral Z ; H2O ; ; sp3-hybridised O-atom
Chloroform H H

4. (c) The given reaction is 7. (b) (a) Sodium peroxide liberates O 2 with hot water.
+6 +6 2Na 2O 2 + 2H 2O(warm) ¾¾® 4NaOH + O 2
2Na2 CrO 4 + H 2 SO 4 ¾¾® Na 2Cr2O 7 + Na 2 SO 4 + H 2O
P (b) Sodium peroxide is a yellow coloured substance but
It is not a redox reaction because there is no change in turns white on exposure to moist air due to the formation
oxidation state of Cr-metal. of NaOH and Na 2CO 3.
P i.e. Na2 Cr2O 7 is a dimeric bridged tetrahedral ion having Na 2O 2 + 2H 2O ¾¾® 2NaOH + H 2O 2
orange colour.
2NaOH + CO 2 ¾¾® Na 2CO 3 + H 2O
O
O (c) Sodium peroxide is a powerful oxidising agent and it
O 1.77A°
O Cr Cr 110 oxidises chromic compounds (Cr 3+ ) to chromate (Cr 6+ ).
°O +3 +6
123°
°
2A

O O 5Na 2O 2 + Cr2(SO 4 )3 ¾¾® 3Na 2SO 4 +2Na 2 CrO 4 + O 2


1 .6

8. (d) 2-butene can be written as,


5. (b) Given, T1 = 40°C, T2 = 60°C
CH3 CH3
H CH3
k1 = 0.86, k 2 = 0.35
C HCO3H H C OH HO C H
We have the equation to check the spontaneity of the +
reaction is C H2O+[O] HO C H H C OH
DG° = DH° - TDS ° [Gibb’s Helmholtz reaction] …(i) H CH3
CH3 CH3
where, DG° = Standard Gibb's free energy cis-2-butene
d l
DH° = Standard enthalpy of the reaction
Racemate
DS° = Standard entropy of the reaction CH3
H CH3
T = Temperature at which reaction occurs
C HCO3H H C OH
So, from Eq. (i), Plane of
Symmetry
DG° = DH° - TDS ° = - RT ln k C H C OH
DS ° DH° H3 C H
Þ ln k = - …(ii) CH3
R RT trans
2-butene meso-compound
Now solving Eq. (ii) for T = 40° C and 60° C
9. (a) H H
DS ° DH°
ln 0.86 = - …(iii) H H Tautomerism
R 313 R
DS ° DH° Me Me Me OH HO Me
and ln 0.35 = - …(iv)
R 333 R NH3

On solving both equations (iii) and (iv), we get Reduction


[H]
-1 N
DS° = - 125.7 JK H3 C N CH3 H3C CH3
H H
DH° = - 38.95 kJ B A
2,4-dimethyl pyrrole
Þ at 27°C, DG° = DH° - TDS ° AgOH+Δ 2 Me–I
(Aromatic compound)
= - 38.95 kJ - 300K ´ ( -125.7 JK -1 )
H α αH + NMe3
= - 1.24 kJ H2 C
N CH2 C D
Me Me
Since, DG° has negative value hence, this reaction is
spontaneous at 27°C. It is not necessary that at the
temperature lower than 27°C, reaction is non-spontaneous.
10. (a) Since, the reactant is a 3°-alkylhalide, so in the presence of
NaCN, it will show E 2 rather than SN 2 .
Spontaneity of a reaction only depends upon negative value of
DG. So, path II is not facile.

32
The possible product by path II is, 14. (c) CH2(CH2)8CH2NHCH3 CH2(CH2)8CH2NH CH3

H
H 1eq
3NaCN
+
CH2 NH2
CH3 CH3 Cl Cl
Br A B
H H
H
+
CH2(CH2)8CH2 N—CH3 CH2(CH2)8CH2 N CH3
CH2
C
NH2
11. (a) Number of A atoms per unit cell
1
=8´ =1
8
Number of B atoms per unit cell
CH2 CH2
1 1
= 2 ´ + 12 ´ = 4 (CH2)8
2 4 (CH2)8
Number of C atoms per unit cell +
CH2
CH2
1 N
= 4 ´ + 1´ 4 = 6
2 N
CH3
Hence, the formula of compound is AB4C 6.
CH3
But in the given question two cases arise.
D
Case I If the tetrad axis passes through the face-centres
where, B lies, then, 15. (d) Boric acid when dissolved in water only one H + ion is
removed to form [B(OH)4 ]- .
-
æ1 ö
Number of B-atoms = 4 - ç ´ 2 ÷ = 3
è2 ø H 3 BO 3 + H 2O [B(OH)4- ] + H+
The formula of the compound left is AB3 C 6. There is no replacement of H-atom, occurs in the above
Case II If the tetrad axis passes through the face-centres reaction.
where ‘C’ lies, then 16. (d) In the presence of anhyd. H2SO 4 , glycerol or mannitol,
æ1 ö boric acid furnishes H + ions, so it acts as a strong acid.
number of C-atoms = 6 - ç ´ 2 ÷ = 5
è2 ø
\ Formula of the compound is AB4C 5. C OH
H3BO3 + 2 H3O+ +2H2O
12. (a) There are 8-effective tetrahedral voids out of which four C OH
are occupied (one at each body diagonal) and 4-effective
octahedral voids such that one is not filled (body-centred). Polyhydroxy C O – O C
alcohol + B
So, out of 12 ( 8 tetrahedral voids + 4 octahedral voids), only 7
C O O C
are filled (4 tetrahedral voids + 3 octahedral voids)
7
\ Fraction of void occupied = = 0.58 H 3BO 3 + 6H 2SO 4 ¾¾® 3H 3O+ + 2HSO 4- + [B(HSO 4 )4 ]-
12
(Anhydrous)
13. (c) —
CH2CH2C—N
— —
CH2CH2C—N
— 17. (b) A - r, B - p, C - q, D - s
Cl Cl
K+NH2
A. K n[Fe(CN)6 ]. Diamagnetic; n = 4
Q K 4 [Fe(CN)6 ] contains Fe in +2 oxidation state having
H 3 d 6 configuration showing low-spin in the presence of
A
H strong ligand, e.g. CN - . Hence, it is diamagnetic in
nature.

CH2CHC—N
— B. [Ag(CN)n ]- : n = 2
Q[Ag(CN)2 ]- contains Ag in +1 oxidation state.
\ +1 - n = -1 or n = 2
C. Facial -[PtCl 3 Brn ]2- ; n = 3
NH2 Br
Cl Br

CH2—CH—C—N

CN Pt
Cl Br
+H+ Cl
Liq·NH3 Facial isomer
CN
B

33
D. [Pt(NH 3 )n ]Cl 4 octahedral n = 6 19. (a) P - q,r ; Q - q,r ; R - p ; S - p,s
\ [Pt(NH 3 )6 ]Cl 4 is an octahedral compound having
Pt 4 + ion. Me
O3 /Ag2O
A. —
C—CH—CH 3 H3C—C—CH3 + CH3CHO
18. (b) A - q, r, s; B - q, r, s ; C - q, r, s; D - p, s Me
Oxidation
Acetone Acetaldehyde
reaction
All C-atoms are sp2 - hybridised Aromatic
Hg2+, H2SO4
A. in nature because it follows Huckel rule. B. —
CH3—C—CH
— CH3—C—CH3
oxidation
Diamagnetic compound as both
p-electrons have opposite spins. Red P + Hl
C. CH—C—CH3 CH3—CH2—CH3
2
All C-atoms are sp - hybridised.
B. Aromatic compound Reduction
D. CH3—CH—CH2—C—CH3 –
Diamagnetic. N2H4 ,OH/Δ
Cl
All C-atoms are sp2-hybridised
C.
Aromatic compound 20. (b) A - q, B - r, C - s, D -p
Diamagnetic A. H 2O has largest H ¾ O ¾ H bond angle.
CH3 It exhibits hyperconjugation as it has nine B. H 2S has lowest boiling point.
D. hyperconjugative structures. C. H 2Se has two lone pairs.
H3C C
Diamagnetic.
D. H 2Te has largest Te ¾ H bond.
CH3

dñVw{ZîR
‘yb^yV ûm¥§Ibm
Aݶ B§Or{Z¶[a¨J àdoe narjm

{hÝXr ‘mܶ‘ H s EH ‘mÌ nwñVH {OgH s Aܶ¶Z gm‘J«r d àíZm| H m


H {RZVm ñVa JEE MAIN & ADVANCED Ho AZwê n h¡

34
1. The de-Broglie wavelength associated with a ball of decomposition of acetone dicarboxylic acid. The
mass 1kg having kinetic energy 0.5 J is energies of activation (cal mol −1 ) in these cases were
(a) 6.626 × 10 −34 m . × 10 −34 m
(b) 1320 (where, R = 3 cal /mol K)
(c) 10.38 × 10 −21
m (d) 6.626 × 10 −34 Å (a) 61990 and 51764 (b) 41764 and 51940

-
(c) 51990 and 41765 (d) 71990 and 41764
2. The value of log10 K at 298 K for a reaction A B is 6. Which two elements in the periodic table would you
−1 expect to combine in the most violent fashion?
[Given, Δ r H° = − 54.07 kJ mol
(a) H and O (b) Cl and F (c) Cs and F (d) Mg and N
Δ r S° = 10 J K −1
B2 H6 , THF CH3 COOH
and R = 8.314 JK −1 mol −1 2.303 × 8.314 × 298 = 5705.84] 7. Ph ⎯ C ≡≡ C ⎯Me ⎯⎯⎯⎯→ A ⎯⎯⎯⎯⎯→ B
(a) 5 (b) 10 A and B are
(c) 95 (d) 100 A B

3. Which one of the following statements is correct? Ph Me Ph Me


(a) Zeolites are hydrated aluminosilicates which can be used (a)
H BH HO H
as shape-selective catalysts 2

(b) Enzymes show maximum activity when pH is either very low Ph Me Ph Me


or very high (b)
H BH3 H OH
(c) Chemically, all enzymes are globular proteins
(d) All of the above Ph Me Ph Me
(c)
4. The correct decreasing order of reactivity for the H 3
B
following coupling compounds with PhN2 Cl is
I. Toluene II. Ethyl benzene Ph Me Ph
(d)
III. Cumene IV. t-butyl benzene H B Me
3
V. Anisole
(a) IV > I > III > II > VI (b) V > I > II > III > IV 8. Which one of the following statements is correct?
(c) II > I > IV > V > III (d) III > I > II > VI > IV (a) Persons working in cement plants and limestone quarries
5. In the decomposition of acetaldehyde in gaseous are more prone to disease like silicosis
1 (b) Lead in water can cause eye disease
phase, various values of log10 K were plotted against
T (c) Ferrocene is responsible for depletion of the ozone layer in
and a straight line with a slope of − 7525 K was obtained. the upper stratosphere of the atmosphere
Similarly, a slope of − 6045 K was obtained in case of (d) Fishes can grow both in warm and in cold water

38
9. Which of the following statements is incorrect? CH 3 N(CH 3 )2
(a) The freezing point of water is depressed by addition of ⏐ ⏐
(a) CH 3 ⎯ C ⎯ CH 2 ⎯ C HN(C H 3 )2
glucose ⏐
(b) The degree of dissociation of a weak electrolyte decreases CH 3
as its concentration decreases CH 3
(c) Energy is released when a substance dissolves in water ⏐
provided that the hydration energy of the substance is more (b) CH 3 ⎯ C ⎯ CH 2C ≡≡ N
than its lattice energy ⏐
(d) All of the above CH 3
CH 3 O
10. In the brown ring test for NO3− ion, complex ⏐ ⏐⏐
[Fe(H2 O)5 NO]2 + is formed. In this complex (c) CH 3 ⎯ C ⎯ C N(CH 3 )2

(a) NO transfers its electron to Fe 2 + such that iron is Fe + and CH 3
NO is NO +
CH 3
(b) Three unpaired electrons are present so that its magnetic
moment is 3.87 BM ⏐
(d) CH 3 ⎯ CCH 2 CH 2 N(CH 3 )2
(c) The colour is due to charge transfer ⏐
(d) All of the above statements are correct CH 3
11. Which of the following species could be expected to 15. A white crystalline salt A reacts with dil. HCl to liberate
exhibit aromatic character? a suffocating gas B and it also forms a yellow
precipitate. The gas B turns potassium dichromate
acidified with dil. H2 SO4 , to a green coloured solution
C. A, B and C respectively, are
I II III IV (a) Na 2 SO4 , SO 2, Cr2(SO 4 )3 (b) Na 2 S, SO 2, Cr2(SO 4 )3
(c) Na 2 S 2O 3, SO 2, Cr2(SO 4 )3 (d) Na 2 SO 3, SO 2, Cr2(SO 4 )3
Select the correct answer from the following.
(a) I and II 16. The colour is not due to d-d transition in which of
(b) II and III the following?
(c) III and IV (a) [CoF6 ]3− (b) CrO 2−
4
3+
(d) I and IV (c) [Ti(H 2O)6 ] (d) [Cu(NH 3 )4 ]2 +
12. 1 dm solution containing 10
3 −5
moles each of Cl and −
17. Give the decreasing order of reactivity of Diels-Alder
CrO24 − ions, is treated with 10 −4 moles of silver nitrate. reaction for the following
Which one of the following observations is found to be
correct? Δ
I. +
[K sp (Ag2 CrO4 ) =4 × 10 −12 , K sp (AgCl) = 1 × 10 −10 ]
(a) Silver chromate gets precipitated first Me COOR
(b) Precipitation does not occur II. Δ
+
(c) Silver chloride gets precipitated first
(d) Both silver chromate and silver chloride start precipitating Me COOR
simultaneously Δ
III. +
13. The complex K4 [Fe(CN)5 O2 ] has t26 geg° electronic Me
configuration then which of the following statements is Me COOR
correct? Δ
IV. +
(a) It is d 2sp3 hybridised and paramagnetic
(b) It is d 2sp3 hybridised and diamagnetic Me COOR
3 2 (a) II > I > III > IV (b) IV > III > I > II
(c) It is sp d hybridised and paramagnetic
3 2
(d) It is sp d hybridised and diamagnetic (c) IV > III > II > I (d) I > II > III > IV
18. Which of the following statements is incorrect?
14. Identify product D in the following reaction sequence (a) CaCl 2 has electrovalent linkage
CH3 (b) NO 3− and CO 2−
3 are not isoelectronic and isostructural
⏐ K2 Cr2 O7 , H2SO4 A (c) H 2O is liquid while H 2 S is a gas due to H-bonding
CH3 ⎯ C ⎯ CH2 CH2 OH ⎯ ⎯⎯⎯⎯⎯⎯ ⎯→ SOCl2 (d) Peroxide ion is diamagnetic and has bond order 1
⏐ H2 O, Heat
B 19. The K sp for AgCl is 10 ° + / Ag = 0.799 V then
. × 10 −10 . If EAg
CH3
Ecell for Ag+ /Ag electrode immersed in 1M KCl at 25°C is
(i) LiAlH4 , dithylether (CH3 ) 2 NH
D ⎯ ⎯⎯⎯⎯⎯⎯⎯⎯⎯→ C ←⎯⎯⎯⎯
⎯⎯ (a) 0.148 V (b) 0.312 V (c) 0.208 V (d) 0.481 V
(ii) H2 O 2 mol

39
20. Identify the product P and Q, respectively are 23. The average speed is greatest in which of the following
HCl + ZnCl2 gas samples?
P ←⎯⎯⎯⎯⎯⎯⎯ (H3 C)3 C ⎯ CH2 OH
(a) 2.0 mole of He at 140 K
SOCl /PCl
⎯ ⎯⎯⎯⎯⎯
2

5
→Q (b) 0.20 mole of CO 2 at 440 K
(a) (H 3C)3C ⎯ CH 2Cl and (H 3 C)3C ⎯ CH 2Cl (c) 0.50 mole of Ne at 500 K
CH 3 (d) 1.0 mole N 2 at 560 K
⏐ 24. One mole of SO3 was placed in a 1 L flask at a given
(b) H 3C ⎯C ⎯ CH 2CH 3 and (H 3C)3C ⎯ CH 2Cl

Cl the reaction, 2SO3 -
temperature, when the equilibrium was established in
2SO2 + O2 the vessel was
found to contain 0.6 mole of SO2 . The value of the
CH 3
⏐ equilibrium constant (molL−1 ) is
(c) (H 3C)3C ⎯CH 2Cl and H 3 C ⎯C ⎯ CH 2 ⎯CH 3 (a) 0.36 (b) 0.675
⏐ (c) 0.45 (d) 0.54
Cl
CH 3 CH 3 25. Which of the following is the adsorption isobar for
⏐ ⏐ chemisorption?
(d) H 3C ⎯ C ⎯ CH 2CH 3 and H 3C ⎯ C ⎯ CH 2 ⎯ CH 3
⏐ ⏐
Cl Cl
21. The number of lone pair on iodine and number of (a) mx (b) mx
d-orbitals used in hybridisation by iodine are same in
(a) ICl 2− (b) ICl 2+
(c) IF7 (d) ICl −4 T T

22. Identify the product formed in the given reaction


O
C ≡≡CH BH3·THF
(c) mx (d) mx
I H2O2/OH–

Li/NH3
II C ≡≡C—C ≡≡C T T

26. Hydrogen peroxide when added to a solution of


potassium permanganate acidified with sulphuric acid
H
(a) reduces sulphuric acid
(a) (b) acts as a reducing agent
(c) acts as an oxidising agent
(d) forms water only

CH3 27. The correct decreasing order of reactivity of the


(b) following compound with HBr is

OH H3C OH
I II

CH3
(c) H3CO OH OH
and III H3CO IV
(a) III > II > IV > I (b) III > II > I > IV
(c) II > III > IV > I (d) III > IV > II > I
28. Which of the following statements is correct?
(a) H 3 PO 3 is dibasic and reducing in nature
(b) The number of S—S bonds in sulphur trioxide trimer (S 3 O 9 )
CH3
(d) is three
(c) O 2 diffuses faster than N 2 through an orifice
(d) NO 2 gas is evolved when PbO 2 is treated with conc. HNO 3

40
29. If same quantity of electricity is passed through three 36. Cuprous ion is colourless while cupric ion is
electrolytic cells containing FeSO4 , Fe2 (SO4 )3 and coloured because
Fe(NO3 )3 , then choose the incorrect statement. (a) cupric ion has an incomplete d-orbital and cuprous ion has
(a) The amount of iron deposited in cells containing FeSO 4 and a complete d-orbital
Fe 2(SO 4 )3 are equal (b) both cuprous ion and cupric ion have half-filled p-and
(b) The amount of iron deposited in cells containing FeSO 4 is d-orbitals
1.5 times of the amount of iron deposited in Fe(NO 3 )3 (c) both cuprous and cupric ions have more than one unpaired
(c) The amount of iron deposited in cells containing Fe 2(SO 4 )3 electrons in d-orbitals
and Fe(NO 3 )3 are equal (d) cupric ion has a complete d-orbital and cuprous ion has an
(d) The same amount of gas is evolved in all three cases incomplete d-orbital
at the anode
37. If AgI crystallises in zinc blende structure with I − ions
30. Consider the first order reaction, at lattice points. What fractions of tetrahedral voids is
C2 H4 O(g) ⎯→ CH4 (g) + CO(g) occupied by Ag+ ions?
If the initial pressure of C2 H4 (g) is 80 mm and the total (a) 75%
pressure at the end of 20 min is 120 mm, then the (b) 50%
half-life is (c) 100%
(a) 120 min (b) 20 min (d) 25%
(c) 60 min (d) 45 min
38. Consider the following sequence of reactions,
31. The hormone that controls the process like burning of Mild (i) CH MgI
fats, proteins and carbohydrates to liberate energy in (CH3 )2 CHOH ⎯ ⎯⎯⎯⎯
⎯→ A ⎯ ⎯⎯⎯⎯⎯
3
→ B
Oxidation [O] (ii) Hydrolysis
the body is
(a) cortisone (b) thyroxine The product B is
(c) adrenalin (d) insulin (a) n-butyl alcohol
(b) isobutyl alcohol
32. A mineral consists of an equimolar mixture of the (c) isobutylene
carbonates of two bivalent metals. One metal is present (d) tertiary butyl alcohol
to the extent of 15.0% by weight, 3.0 g of mineral on
heating lost 1.10 g of CO2 . The per cent by weight of
39. Which of the following statements is incorrect about
formic acid?
other metal is
(a) It reduces Tollen’s reagent
(a) 35% (b) 75%
(b) It is a stronger acid than benzoic acid
(c) 25% (d) 65%
(c) It forms formyl chloride with PCl 5
33. The basicities of hypophosphorus acid, phosphorous (d) It gives CO and H 2O on heating with conc. H 2 SO 4
acid, hypophosphoric acid and pyrophosphoric acid
respectively, are 40. Select the correct statement(s).
(a) Cyanamide ion (CN 22− ) is isoelectronic with CO 2 and has the
(a) 1, 2, 4 and 4 (b) 1, 3, 4 and 2
(c) 2, 2, 3 and 4 (d) 1, 1, 2 and 3 same linear structure
(b) Mg 2 C 3 reacts with water to form propyne
34. The number of atoms in 100 g of a fcc crystal with (c) CaC 2 has NaCl type lattice
density and cell edge of 10 g cm− 3 and 200 pm,
(d) All of the above
respectively is
(a) 5 × 10 20 atoms (b) 5 × 10 24 atoms Directions (Q. Nos. 41 to 60) In the following questions
20
(c) 3 × 10 atoms (d) 3 × 10 24 atoms a statement of Assertion followed by a statement of
reason, is given. Choose the correct answer out of the
35. Which of the following statements are correct about the
given reaction? following choices.
O (a) Both Assertion and Reason are true and Reason is the
+ BF3 correct explanation of Assertion
P
H3C Cl H3C (b) Both Assertion and Reason are true but Reason is not the
O correct explanation of Assertion
(c) Assertion is true but Reason is false
(a) The product P is H3C CH3 which is (d) Both Assertion and Reason are false
proceeds via the formation of acylium ion
O
41. Assertion (A) The intermolecular forces of polymer
Nylon-6 6, buna-S, polythene increase in order.
I II III
(b) The product P is H3C
(c) The formation of P proceeds via the formation of II < III < I.
carbonation Reason (R) Nylon-6 6 is a fiber and buna-S is an
(d) All of the above elastomer.

41
42. Assertion (A) Among I, II and III, the molecules of II 53. Assertion (A) Softening of hard water is done using
are most suitable to disperse benzene in water. sodium aluminium silicate (zeolite).
O Reason (R) Al3 + ions replaces Ca2 + and Mg2 + ions from
– hard water.
ONa+
I 54. Assertion (A) The structure of glycogen is similar to
that of amylopectin.
Reason (R) Glycogen is hydrolysed by
CH3 enzyme β-amylase.
II
55. Assertion (A) Among Li, Na, K, Rb and Cs, lithium
salts exhibit the least electrical conductance and
Cs salts exhibits more electrical conductance in
Cl
III aqueous solutions.
Reason (R) II is non-polar covalent organic compound. Reason (R) Smaller the radius of the hydrated cation,
43. Assertion (A) A mixture of o- and p-aminosulphonic lower is the electrical conductance in aqueous solution.
acid is formed on heating aniline hydrogen sulphate. 56. Assertion (A) Towards nucleophilic attack,
Reason (R) Sulphonic acid shows electron benzaldehyde is less reactive than ethanol.
withdrawing nature. Reason (R) All the carbon atoms of benzaldehyde are
44. Assertion (A) The O—O bond length in H2 O2 is shorter sp2 -hybridised.
than that of O2 F2 .
57. Assertion (A) Zn, Cd and Hg are placed in d-block
Reason (R) H2 O2 is an ionic compound. elements but not regarded as transition elements.
45. Assertion (A) tert-butyl methyl ether on treatment with Reason (R) Zn, Cd and Hg have fully filled d-orbitals in
HI at 373 K gives a mixture of methyl iodide and their most common oxidation states.
tert-alcohol. 58. Assertion (A) When a solution becomes hot on mixing
Reason (R) The reaction takes place by SN 2 mechanism. two liquids it indicates that it shows negative deviation
46. Assertion (A) When rain is accompanied by a from Raoult’s law.
thunderstorm, the collecter rain water will have a Reason (R) Solutions which show negative deviation from
pH value slightly lower than that of rain water without Raoult’s law are accompanied by the decrease in volume.
thunderstorm. 59. Assertion (A) Sulphide ores generally roasted to oxide
Reason (R) There is formation of SO2 which ultimately for the extraction of metals instead of being directly
changes to H2 SO4 . reduced.
47. Assertion (A) p-methyl benzyl carbocation(I) is more Reason (R) For the reduction of ores if sulphide ores are
stable than benzyl carbocation (II). used, these will form carbon disulphide.
Reason (R) I is more stable than II due to
60. Assertion (A) C p − C V = R for an ideal gas.
hyperconjugation.
∂E
48. Assertion (A) NF3 is a weaker ligand than N(CH3 )3 . Reason (R) ⎡ ⎤ = 0 for an ideal gas.
⎢⎣ ∂V ⎥⎦ T
Reason (R) NF3 ionises to give F − ion in
aqueous solution.
49. Assertion (A) The electrolysis of NaCl solution gives
H2 (g) at cathode and Cl2 (g) at anode.
Reason (R) H2 O has lower oxidation potential than Cl2 .
50. Assertion (A) The order of bond angle
decreases as follows NH3 > NH4+ > NH2−
Reason (R) The repulsive force increases in the order
NH3 > NH4+ > NH2−
51. Assertion (A) On compressing a gas to half the
volume, the number of molecules is halved.
Reason (R) The number of moles of gas present
decreases with decrease in volume.
52. Assertion (A) Greater the value of K, more is the
fraction of initial concentration of reactants converted
to products at equilibrium.
Reason (R) The value of K depends on the initial
concentration of reactants.

42
Answers with Explanation
1. (a) de-Broglie wavelength is given as 9. (c) There will be a depression in the freezing point by the
h 6.6 × 10 −34 Js addition of glucose in water.
λ= =
2 m(KE ) 2 × 1 kg × ( 0.5 J) The degree of dissociation (α) increases as concentration
decreases.
= 6.626 × 10 −34 m
Δ sol H = Δ lattice H + Δ hyd H
2. (b) ΔG ° = ΔH ° − TΔS ° ( − ve) ( − ve)

3
= − 54.07 × 10 J − 298 × 10 J 10. (d) NO ⎯→ NO+ + e −
= − 57.05 × 10 3 J Fe 2 + + e − ⎯→ Fe +
ΔG ° = − 2.303RT log K 3d 6 3d7
− ΔG ° 57.05 × 10 3 ~ 4 unpaired 3 unpaired
log K = = − 10
2.303RT 5705.84 electrons electrons
3. (c) Zeolites are not used as such but are first heated in Magnetic moment, μ for Fe + = n( n + 2 ) BM
vacuum so that the water on hydration is lost. = 3 × 5 BM
Enzymes have maximum activity at pH of 7.4. = 3.87 BM
••
4. (b) Electron donating power of ⎯OMe > ⎯ Me 11. (b) For an organic compound to be aromatic, it should be
••
planar and contains ( 4n + 2 ) π-electrons (Huckel’s rule).
(3 hyperconjugated (HC) structures and + I-effect)
> ⎯CH 2CH 3 (2 HC structures and + I-effect) I. ; 8 π-electrons; Antiaromatic
> Me 2CH −(isopropyl) (1 HC structure and + I-effect)
> Me 3C −(t-butyl) (no HC only + I-effect)
5. (c) From Arrhenius equation, we know that slope II. ;
Ea
=−
2.303 × R
Tropylium cation is planar and has 6 π -electrons according to
Ea
=− Huckel’s rule, hence, it is an aromatic compound.
2.303 × 3
For the decomposition of acetaldehyde III. ; 6 π-electrons; Planar; Aromatic
Ea
− 7525 = −
6.909
∴ E a = 51990
, cal/mol IV. ; 4 π-electrons, Antiaromatic
Similarly, for the decomposition of acetone dicarboxylic acid
Ea
− 6045 = − 12. (c) For precipitation,
6.909
∴ E a = 41765
, cal/mol Ionic product > solubility product (Ksp )
For Ag 2CrO 4 ,
6. (c) Cs and F
Ionic product (IP) = [Ag+ ]2[CrO 4− ] = (10 −4 )2(10 −5 ) = 10 −13
There is a large electronegativity difference between the two
elements Cs and F, thus forces of attraction will be strongest Ksp for Ag 2CrO 4 = 4 × 10 −12
between them. Hence, they react violently. Ksp > IP
7. (c) Hydroboration reaction first gives R 3B and then with Thus, precipitation does not occur.
CH 3COOH, it gives cis-alkene For AgCl, Ionic product = [Ag+ ][Cl − ] = [10 −4 ][10 −5 ] = 10 −9
Ksp [AgCl] = 1 × 1010
B2H6, THF Ph Me
Ph—C C—Me C==C— IP > Ksp
H
—B So, percipition occurs
3

CH3COOH Thus, silver chloride gets precipitated first.


13. (a)
Ph Me 3d 4s 4p
Fe2+; [Ar]
H OH
Fe2+in K4[Fe(CN) 5O2] ;
8. (a) Ž Lead in water can cause kidney damage.
3d d2sp3
Ž Freon is responsible for depletion of the ozone layer in
the upper stratosphere of the atmosphere. [Ar] ×× ×× ×× ×× ××××

Ž Fishes do not grow in warm as well as in cold water. Complex is paramagnetic due to the presence of unpaired
electron of O 2− , i.e. superoxide ion acting as a ligand.

43
CH 3 1 × 10 −10
Ksp
[Ag + ] = = = 10 −10 M
⏐ Cr 2 O 27 − /H + [Cl ] −
1
14. (d) CH 3 ⎯ C ⎯ CH 2CH 2OH ⎯ ⎯ ⎯ ⎯⎯→
⏐ Ag+ + e − ⎯→ Ag
CH 3 0.0591 1
E = E° − log −10 = 0799 . − 0.591 = 0.208 V
CH 3 O CH 3 O n 10
⏐ ⏐⏐ SOCl 2 ⏐ ⏐⏐ 20. (b)
CH 3 ⎯ C ⎯ CH 2 ⎯ C ⎯OH ⎯ ⎯ ⎯ ⎯→ CH 3 ⎯ C ⎯ CH 2 ⎯ C ⎯Cl
⏐ A ⏐ H3C CH2 H3C CH2 H3C CH2
–CH3 Cl
CH 3 CH 3 C C CH3 C CH3
Shift +
B H3C H3C H3C Cl
CH 3 O CH3 (P)

(CH 3 ) 2 NH
⏐⏐ The formation of Q does not involve carbocation formation
⎯⎯⎯⎯
⎯→ CH 3 ⎯C ⎯ CH 2 ⎯ C ⎯N(CH 3 )2 and rearrangement. So the product Q is (H 3C)3CCH 2Cl.

CH 3
21. (d)
Cl
C
+
CH 3 ICl2– I sp3d ICl2+ I sp3
(i) LiAlH 4 ⏐
⎯ ⎯ ⎯⎯→ CH 3 ⎯ C ⎯ CH 2 CH 2 N(CH 3 )2 Cl Cl Cl
(ii) H 2 O
⏐ F F
CH 3
D IF7
F
II F
sp3d3 ICl4–
Cl
I
Cl
sp3d2
15. (c) Gas B turns the colour of acidified K 2 Cr2O 7 green thus it is F F Cl Cl
SO 2. SO 2 is obtained alongwith yellow precipitate when F
thiosulphate is treated with dilute acid. Thus, A, B and C are 22. (b)
Na 2 S 2 O 3, SO 2 and Cr2(S 2 O 4 )3 respectively.
Reactions involved C CH C
Na 2S 2O 3 + HCl ⎯→ SO 2 + NaCl + H 2O + S BH3·THF CH3
Suffocating Yellow I.
gas ( B ) H2O2/OH–
K 2Cr2O 7 + SO 2 + H2SO 4 ⎯→ K 2 SO 4 + Cr2(SO 4 )3
Green

16. (b) In CrO 2−


( C ( is an electron withdrawing group, thus electron
4 , Cr is in + 6 oxidation state and has
0
d configuration. The colour is due to charge transfer and not migration is takes place.
due to d-d transition. ⎯ C ≡≡CH, hydroboration-oxidation addition of H 2O is against
17. (c) The rate of Diels-Alder reaction increases, when normal addition.
dienophiles have electron-withdrawing groups and dienes II. By Li/NH 3, trans-addition is occurred.
have electron -donating group.
8RT T
Thus, the correct order is IV > III > II > I. 23. (a) u = , u ∝
πM M
18. (b) Isoelectronic means same number of electrons and
140 440
isostructural means same structure. (a) u ∝ = 35 (b) u ∝ = 10
NO −3 = 7 + 8 × 3 + 1 = 32 , CO −3 = 6 + 8 × 3 + 2 = 32 4 44
500 560
(c) u ∝ = 25 (d) u ∝ = 20

-
20 28
N C
24. (b) 2SO 3 2SO 2 + O 2
NO3 CO32 Initially 1 0 0
(a) In CaCl 2, Ca—Cl bonds are electrovalent bonds At equilibrium 1 − 2x 2x x
(c) Correct ∴ 2 x = 0.6 ∴ x = 0.3
(d) Peroxide ion is O 2− [SO 3 ] = 1 − 2 x = 1 − 0.6 = 0.4
2
[SO 2 ] = 2 x = 0.6
O 22− = σ 1s 2, *σ 1s 2, σ 2 s 2, *σ 2 s 2, σ2 pz2, π2 px2 ≈ π2 py2, [O 2 ] = x = 0.3
*π 2 p2 ≈ *π 2 p2 x × (2 x )2 0.3 × 0.6 × 0.6
x y K = = = 0.675 mol L−1
(1 − 2 x )2 0.4 × 0.4
Nb − Na 10 − 8
BO = = =1 25. (b) In chemisorption, adsorption first increases and then
2 2
decreases.
19. (c) Ag(I) ion collects as solid AgCl on the electrode itself. +7 2+
However, there is some Ag + in equilibrium with AgCl(s) in 26. (b) 2KMnO 4 + 3H 2 SO 4 + 5H 2O 2 ⎯→ K 2 SO 4 + 2 MnSO 4
solution.
AgCl( s ) -
Ag+ ( aq ) + Cl − ( aq )
+ − ∴ H 2 O 2 acts as a reducing agent. It reduces K MnO 4 to Mn2 +
+ 8H 2O + SO 2
+7

∴ Ksp = [Ag ][Cl ]


ion.

44
27. (b) Electron donating group (EDG) favours SN 1 reactivity 31. (d) Insulin maintains the blood level in glucose. It controls the
with HBr. processes of burning of fats, proteins and carbohydrates.
Electron withdrawing group (EWG) retards SN 1 reactivity 32. (a) MCO 3 ⎯→ MO + CO 2 ↑
with HBr. M ′ CO 3 ⎯→ M ′ O + CO 2 ↑
ED effect of ( ⎯ OCH 3 ) group (+ R, − I ) is greater than ( ⎯ CH 3 ) Equivalent of CO 2 = Equivalent of carbonates of metals
group (+ I and hyperconjugation). So, reactivity of III > II.
1 mol of CO 2 = 1 mol of CO 2−
3
In IV, ( ⎯ OCH 3 ) group is at m-position and exerts only
− I -effect, hence, reactivity of (IV) is less than (I). 44 g of CO 2 = 60 g of CO 2−
3

∴ The correct decreasing order is, III > II > I > IV. 60
1.1 g of CO 2 = × 1.1 = 1.5 g of CO 32−
44
28. (a) H 3 PO 3 is a dibasic reducing acid.
. × 100
15
% of CO 2−
3 = = 50%
O 3
⏐⏐ % of one metal = 15%
Its structure is given as, H ⎯ P ⎯ OH
⏐ % of another metal = 100 − ( 50 + 15) = 35%
OH 33. (a)
(b) The structure of S 3O 9 is
P P
S H OH
OH H
H OH
O O
Hypophosphorous acid Phosphorus acid
S S (H3PO2) (H3PO3)
Basicity=1 Basicity=2
O
No, S—S linkage is present in S 3O 9.
V 1
(c) Rate of diffusion, r = ∝ P OH
t M HO OH P
HO P P
(d) PbO 2 + 2HNO 3 ⎯→ Pb(NO 3 )2 + H 2O + O 2 OH
O
1 HO
29. (a) FeSO 4 : Fe 2+ + 2e − ⎯→ Fe; 1 F = mol of Fe Hypophosphoric acid
OH OH
2 (H4P2O6) Pyrophosphoric acid
1
Fe(NO 3 )3 : Fe 3+ + 3 e − ⎯→ Fe ; 1 F = mol of Fe Basicity=4 (H4P2O7)
3 Basicity=4
3+ − 1
Fe 2(SO 3 )3 : Fe + 3e ⎯→ Fe ; 1 F = mol of Fe Z × Mw
34. (b) Density, d =
3 a 3 × 10 − 3 × NA
Amount of Fe deposited in cell containing Fe(NO 3 )3 =
d × a 3 × 10 − 30 × NA
Amount of Fe deposited in cell containing Fe 2(SO 4 )3 Mw = [For fcc, Z = 4/unit cell]
Amount of Fe deposited in cell containing Fe 2(SO 4 )3 Z
1 1 10 g cm− 3 × (200)3 × 10 − 30 cm3 × 6 × 10 23 atoms
= : = 1.5 : 1 =
2 3 4

At anode : 4OH ⎯→ O 2 + 2H 2O + 4 e − = 12 g mol − 1
In all cases same amount of gas is evolved at the anode. Thus, 12 g mol − 1 contains, NA atoms = 6 × 10 23 atoms
30. (b) C2 H 4O( g ) ⎯→ CH 4 ( g ) + CO( g ) 6 × 10 23
∴ 100 g will contain = × 100 = 5 × 10 24 atoms
At t=0 p0 0 0 12
At t =t p0 − x x x 35. (a)
+ BF3 H3C—C=O
Total pressure at time t H3C Cl –Cl
( pt ) = p0 − x + x + x = p0 + x
p0 = 80 mm, p0 + x = 120 mm H3C—C ≡ O
∴ x = 120 − 80 = 40 mm Acylium ion
2.303 p0
k= log
20 p0 − x +
H3C CO
2.303 80 H3C
= log H3C CH3
20 80 − 40 –H
2.3 × 0.3
=
20
0.69 0.69 × 20
t 1/ 2 = =
k 0.69 H3 C CH3
= 20 min 36. (a) Option (a) is correct .

45
37. (b) In AgI crystal, number of Ag + ions is equal to I − ions. 49. (c) Correct Reason Cl 2 has higher reduction potential than
However, the number of tetrahedral voids are twice than the H 2O, but has lower discharge potential on account of over
number of atoms forming the cubic lattice. Therefore, number potential of O 2 . As a reference,
of tetrahedral voids occupied by Ag + ion = 50%. (i) Cl − ( aq ) ⎯→
1
Cl 2( g ) + e − ; Ered

= 1.36 V
2
38. (d) −
H3 C H3C H3C or Eoxide = − 1.36 V
[O] CH3MgI
CH—OH CH==O
H3O+
C—OH (ii) 2H 2O( l ) ⎯→ 4H ⊕( aq ) + 4 e − + O 2( g )
H3 C H3C H3C
A − −
CH3 Ered = 1.23 V or Eoxi = − 1.23 V
t-butyl alcohol
B 50. (d) The correct order of decreasing bond angle is
39. (c) Statements (a), (b) and (d) are correct, but statement (c) is NH 4+ > NH 3 > NH −2
O This is because all of them involves sp3 hybridisation. The
⏐⏐ number of lone pair of electron present on N-atoms of NH 4+ ,
incorrect, although HCOOH + PCl 5 gives H ⎯ C ⎯ Cl −
NH 3 and NH 2 are 0, 1 and 2 respectively. Greater the number
(formyl chloride) but it is unstable and does not exist. of lone pairs, greater will be the repulsion on the bond pairs
s s and hence, smaller is the bond angle.
40. (d) •• N ==C == N •• , O == C == O
51. (d) Number of molecules is independent of pressure, and
Thus, CN 22 − and CO 2 both are isoelectronic and isostructural. mole number is independent of volume.
Mg 2 C 3 + 3H 2O ⎯→ CH 3 ⎯ C ≡≡ CH + Mg(OH)2 52. (c) K depends only upon temperature.
Propyne
53. (c) Hardness of water is due to carbonate or sulphate of
Ca 2 +C 22 − has NaCl type lattice. Ca 2 + and Mg 2 + ions. When sodium aluminium silicate is
41. (b) Correct explanation Nylon-6,6 is a fiber and has added to the hard water. Na + replaces Ca 2 + and Mg 2 + by
hydrogen bonding as intermolecular force. Buna-S is an adsorption and hence, water becomes soft.
elastomer and has weak van der Waals’ force. Polythene is
54. (b) Like starch, glycogen is also a condensation polymer of
thermoplastic and has intermediate forces (i.e. stronger than
α-D-glucose. It is hydrolysed by the enzyme β-amylase to
elastomer but weaker than fiber). Thus, the given order of
form maltose. Glycogen is also called animal starch because
intermolecular force is correct.
its structure is similar to that of amylopectin. One main
42. (d) Benzene is non-polar, covalent organic compound. As we difference between glycogen and amylopectin is that
know that like dissolves like, i.e. polar compound is soluble in amylopectein chains consist of about 20-25 glucose units but
polar medium and non-polar is soluble in non-polar medium.
glycogen chains are much shorted i.e. they consist of about
Among the given compounds meta-methylnonyl benzene is
10-14 glucose units.
the only non-polar compound, hence it is soluble in benzene.
1
Rest all, i.e. I and III have polar ends. 55. (c) Electrical conductance ∝
size of hydrated cation
1
Hydration energy ∝
CH3 size
43. (b) Correct explanation Aniline hydrogen sulphate on As we move from Li to Cs, size of alkali metals increases and
heating decomposes to aniline and H 2 SO 4 . Sulphonation thus, hydration energy increases. Thus, lithium ion being
of aniline with H 2 SO 4 takes place (SE reaction) at smallest in size among alkali metals, is most extensively
o-and p-positions, since ⎯NH 2 group is o-and p-directing. hydrated and becomes largest in size. Thus, it has least
44. (d) Both Assertion and reason are false. electrical conductivity among alkali metals.
The O—O bond length is shorter in O 2 F2 than in H 2O 2 due to 56. (b) Due to electron withdrawing effect of carbonyl group of
higher electronegativity of F-atom. H 2O 2 is a covalent aldehyde, electron density of benzene ring decreases and
compound. electron density on carbonyl group increases. Hence,
45. (d) Correct Assertion tert-butyl methyl ether on treatment carbonyl carbon of benzaldehyde is less reactive towards
with HI at 373 K gives tert-butyl iodide and methyl alcohol. nucleophilic attack.
Correct Reason The reaction occurs via SN 1 mechanism. 57. (a) Zn, Cd and Hg are d-block elements but not regarded as
46. (c) Assertion is true but Reason is false. transition elements because these element do not have
Correct Reason During thunderstorm, there is the formation of partially filled d-orbitals in their most common oxidation states.
NO which changes to NO 2 and ultimately to HNO 3 (acid-rain). 58. (b) In case of solutions showing negative deviation, the forces
O2 H2O of interaction between the components are stronger than
N2+O2 NO NO2 N2O5 HNO3 those in the pure components.
(pH<7)
59. (a) Carbon is usually used for the reduction of ores and if
47. (a) (I) is more stable than (II) as (I) shows more sulphide ores are used, it will form carbon disulphide which is
hyperconjugative structures than (II). difficult to get rid off. Therefore, sulphide ores are first roasted
48. (c) The correct reason is that the lone pair of electron on the to get metal oxides which are then reduced by carbon to give
N-atom is strongly attracted by the three strongly metal and CO 2 escapes easily into the air.
electronegative F-atoms. As a result, not available for
donation and hence NF3 is a weak ligand. 60. (b) Both expression are true for an ideal gas.

46
1. A stream of electrons from a heated filament was
passed between two charged plates kept at a potential
difference V esu. If e and m are charge and mass of an
electron, respectively, then the value of h/ l (where, l is
wavelength associated with electron wave) is given by
(a) 2 meV (b) meV (c) 2 meV (d) meV
2. 2-chloro-2-methylpentane on reaction with sodium
methoxide in methanol yields
CH3
½
I. C2 H5 CH2 C ¾ OCH3 II. C2 H5 CH2 C == CH2
½ ½
CH3 CH3
III. C2 H5 CH== C ¾ CH3
½
CH3
(a) Both I and III (b) Only III (c) Both I and II (d) All of these
3. Which of the following compounds is metallic and
ferromagnetic?
(a) CrO 2 (b) VO 2 (c) MnO 2 (d) TiO 2
4. Which of the following statements about low density
polythene is false?
(a) It is a poor conductor of electricity
(b) Its synthesis required dioxygen or a peroxide initiator as a
catalyst
(c) It is used in the manufacture of buckets, dustbins etc
(d) Its synthesis requires high pressure
5. For a linear plot of log (x / m) versus log p in a
Freundlich adsorption isotherm, which of the
following statements is correct? (k and n are constants)
(a) 1/n appears as the intercept
(b) Only 1/n appears as the slope
(c) log æç ö÷ appears as the intercept
1
è nø
(d) Both k and 1/n appear in the slope term
6. The heats of combustion of carbon and carbon
monoxide are - 393.5 and - 283.5 kJ mol -1 ,
EXAM CRUX respectively. The heat of formation (in kJ) of carbon
monoxide per mole is
In chemistry, there are total 30 Questions asked in JEE Main (online)
2016 paper. Out of which 8 are numericals, while 22 questions are (a) 676.5 (b) -676.5 (c) -110.5 (d) 110.5
theory based. Overall paper was tricky and easy to solve. 7. The hottest region of Bunsen flame Region 4
Region 3
According to chapterwise distribution number of questions were shown in the figure given below is Region 2
asked as– (a) region 2 Region 1

• From Class XI Some Basic Principles of Organic Chemistry3-Qs, (b) region 3


Hydrocarbons-2Qs, s-block Elements-2Qs, Atomic Structure-2Qs, (c) region 4
States of Matter-1Q, Thermodynamics, Equilibrium, Hydrogen and (d) region 1
Chemical bonding-1Q from each. 8. Which of the following is an anionic detergent?
• From Class XII Metallurgy-3Qs, Chemistry in Everyday life-2Qs, (a) Sodium lauryl sulphate
p -block Elements-2Qs, Solutions, Solid State, Chemical Kinetics, (b) Cetyltrimethyl ammonium bromide
Surface Chemistry, Coordination Compounds, Alcohols, Phenols (c) Glyceryl oleate
and Ethers, Amines-1Q from each. (d) Sodium stearate
• Difficulty Level Around 40% of questions can be considered as easy, 9. 18 g of glucose (C6 H12 O6 ) is added to 178.2 g water.
30% are relatively tough and 30% are considered to be tough by The vapour pressure of water (in torr) for this aqueous
Arihant team. This year paper was little tough as compare to the solution is
previous year paper. (a) 76.0 (b) 752.4 (c) 759.0 (d) 7.6

50
10. The distillation technique most suited for separating 21. Which one of the following statements about water is
glycerol from spent lye in the soap industry is false?
(a) fractional distillation (b) steam distillation (a) Water can act both as an acid and as a base
(c) distillation under reduced pressure (b) There is extensive intramolecular hydrogen bonding in the
(d) simple distillation condensed phase
(c) Ice formed by heavy water sinks in normal water
11. The species in which the N-atom is in a state of (d) Water is oxidised to oxygen during photosynthesis
sp hybridisation is
(a) NO -2 (b) NO -3 (c) NO 2 (d) NO +2
22. The concentration of fluoride, lead, nitrate and iron in
a water sample from an underground lake was found to
12. Decomposition of H2 O2 follows a first order reaction. be 1000 ppb, 40 ppb, 100 ppm and 0.2 ppm,
In 50 min, the concentration of H2 O2 decreases from
respectively. This water is unsuitable for drinking due
0.5 to 0.125 M in one such decomposition. When the
to high concentration of
concentration of H2 O2 reaches 0.05 M, the rate of (a) lead (b) nitrate (c) iron (d) fluoride
formation of O2 will be
(a) 6.93 ´ 10 -4 mol min-1 (b) 2.66 L min-1 at STP 23. The main oxides formed on combustion of Li, Na and K
. ´ 10 -2 mol min-1
(c) 134 (d) 6.93 ´ 10 -2 mol min-1 in excess of air respectively are
(a) LiO 2, Na 2O 2 and K 2O (b) Li 2O 2, Na 2O 2 and KO 2
13. The pair having the same magnetic moment is (c) Li 2O, Na 2O 2 and KO 2 (d) Li 2O , Na 2O and KO 2
[at. no. Cr = 24, Mn = 25, Fe = 26 and Co = 27]
(a) [Cr(H 2O)6 ]2+ and [Fe(H 2O)6 ]2+ 24. Thiol group is present in
2+ 2+ (a) cystine (b) cysteine (c) methionine (d) cytosine
(b) [Mn(H 2O 6 )] and [Cr(H 2O)6 ]
(c) [CoCl 4 ]2- and [Fe(H 2O)6 ]2+ 25. Galvanisation is applying a coating of
(a) Cr (b) Cu (c) Zn (d) Pb
(d) [Cr(H 2O)6 ]2+ and [CoCl 4 ]2-
CO2H 26. Which of the following atoms has the highest first
ionisation energy?
14. The absolute configuration of H OH is
(a) Na (b) K (c) Sc (d) Rb
H Cl
CH3 27. In the Hofmann-bromamide degradation reaction, the
number of moles of NaOH and Br2 used per mole of
(a) (2S, 3R) (b) (2S, 3S) (c) (2R, 3R) (d) (2R, 3S)
amine produced are
15. The equilibrium constant at 298 K for a reaction,
A+ B - C + D is 100. If the initial concentrations of
(a) four moles of NaOH and two moles of Br2
(b) two moles of NaOH and two moles of Br2
all the four species were 1 M each, then equilibrium (c) four moles of NaOH and one mole of Br2
concentration of D (in mol L-1 ) will be (d) one mole of NaOH and one mole of Br2
(a) 0.818 (b) 1.818 (c) 1.182 (d) 0.182
28. Two closed bulbs of equal volume (V) containing an
16. Which one of the following ores is best concentrated by ideal gas initially at pressure pi and temperature T1 are
froth floatation method? connected through a narrow tube of negligible volume
(a) Siderite (b) Galena (c) Malachite (d) Magnetite as shown in the figure below. The temperature of one
17. At 300 K and 1 atm, 15 mL of a gaseous hydrocarbon of the bulbs is then raised to T2 . The final pressure pf is
requires 375 mL air containing 20% O2 by volume for
T1 T2 T1 T2
complete combustion. After combustion, the gases
pi, V pi, V pf , V pf, V
occupy 330 mL. Assuming that the water formed is in
liquid form and the volumes were measured at the
same temperature and pressure, the formula of the æ T1 ö æ T ö æ TT ö æ TT ö
hydrocarbon is (a) 2 pi ç ÷ (b) 2 pi ç 2 ÷ (c) 2 pi ç 1 2 ÷ (d) pi ç 1 2 ÷
(a) C 3H 8 (b) C 4H 8 (c) C 4H10 (d) C 3H 6 è T1 + T2 ø è T1 + T2 ø è T1 + T2 ø è T1 + T2 ø

18. The pair in which phosphorus atoms have a formal 29. The reaction of propene with HOCl (Cl2 + H2 O)
oxidation state of +3 is proceeds through the intermediate
+ +
(a) pyrophosphorous and hypophosphoric acids (a) CH 3 ¾ C H ¾ CH 2 ¾ Cl (b) CH 3 ¾ CH(OH) ¾ C H 2
(b) orthophosphorous and hypophosphoric acids + +
(c) pyrophosphorous and pyrophosphoric acids (c) CH 3 ¾ CHCl ¾ C H 2 (d) CH 3 ¾ C H ¾CH 2 ¾ OH
(d) orthophosphorous and pyrophosphorous acids 30. The product of the reaction given below is
19. Which one of the following complexes shows optical (i) NBS/hν
isomerism? X
(ii) H 2O/K2CO3
(a) cis [Co(en)2Cl 2 ]Cl (b) trans [Co(en)2Cl 2 ]Cl
(c) [Co(NH 3 )4 Cl 2 ]Cl (d) [Co(NH 3 )3Cl 3 ]
OH O
20. The reaction of zinc with dilute and concentrated nitric CO2H

acid, respectively, produces (a) (b) (c) (b)


(a) NO 2 and NO (b) NO and N 2O
(c) NO 2 and N 2O (d) N 2O and NO 2

51
Answers with Explanation
1. (c) Plan As you can see in options, energy term is mentioned On subtracting Eq. (ii) from Eq. (i), we get
h 1
hence, we have to find out relation between and energy. For C(s ) + O2 ( g ) ¾® CO( g );
l 2
this, we shall use de-Broglie wavelength and kinetic energy DH = ( - 393.5 + 283.5) kJ mol -1 = - 110 kJ mol -1
term in eV.
h 7. (a) Region 1 (Pre-heating zone)
de-Broglie wavelength for an electron ( l ) =
p Region 2 (Primary combustion zone, hottest zone)
h Region 3 (Internal zone)
Þ p= …(i)
l Region 4 (Secondary reaction zone)
Kinetic energy of an electron = eV 8. (a) Sodium lauryl sulphate [(CH 3(CH 2 )10CH 2OSO 3-Na + )]
p2 = Anionic detergent
As we know that, KE =
2m Cetyltrimethyl ammonium bromide
p2 +
\ eV = or p = 2meV …(ii) é CH 3 ù
2m ê ú
h ½
From equations (i) and (ii), we get = 2meV êCH 3(CH 2 )15 ¾N ¾ CH 3 ú Br - = Cationic detergent
l ê ½ ú
ê CH 3 ú
- ë û
2. (d) Key concept Strong nucleophile ( OMe) in polar solvent
(MeOH) gives elimination products over substitution Glyceryl oleate [(C 17 H 32COO)3C 3H 5 ] = Non-ionic detergent
products but all products are possible in different yields. Sodium stearate [C 17 H 35COO -Na + ] = Anionic soap
Cl OCH3
– +
9. (b) Key concept Vapour pressure of water ( p° ) = 760 torr
MeONa
CH3—C—CH 2CH2CH3 CH3—C—CH2—CH2—CH3 + Mass (g)
MeOH Number of moles of glucose =
CH3 CH3 Molecular mass (g mol-1)
(Less yield) 18 g
= = 0.1mol
CH3—C==CH—CH2—CH3+CH2 ==C—CH2—CH2—CH3 180 gmol -1
Number of moles of water = (1 - 01 . ) mol = 9.9 moles
CH3 CH3
(More yield)
Total number of moles = ( 01. + 9.9) moles = 10 moles
3. (a) Only three elements iron (Fe), cobalt (Co) and nickel (Ni) Now, mole fraction of glucose in solution = Change in
show ferromagnetism at room temperature. CrO 2 is also a pressure with respect to initial pressure
metallic and ferromagnetic compound which is used to make Dp 01 .
i.e. =
magnetic tapes for cassette recorders. p° 10
4. (c) High density polythene is used in the manufacture of or Dp = 0.01p° = 0.01 ´ 760 = 7.6 torr
buckets, dustbins etc. \ Vapour pressure of solution = (760 - 7.6) torr = 752.4 torr
x
5. (b) According to Freundlich adsorption isotherm, = kp1/ n 10. (c) Glycerol with high boiling point (290°C) can be separated
m
from spent lye by distillation under reduced pressure. This
On taking logarithm of both sides, we get
process is used to purify liquids having very high boiling
x x 1
log = log k + log p1/ n or log = log k + log p points. By this process, liquid is made to boil at lower
m m n temperature than its boiling point by lowering the pressure on
y = c + mx its surface.
11. (d) Species
1
Slope=
n
log x/m

θ
N sp2

O

log k N sp2

log p
x 1
y = log , c = intercept = log k, m = slope = and x = log p N sp2
m n
6. (c) C ( s ) + O 2( g ) ¾® CO 2( g ); DH = - 393.5 kJ mol -1 …(i) +
N sp
1
CO + O 2 ¾® CO 2( g ); DH = - 283.5 kJ mol -1 …(ii)
2

52
12. (a) For first order reaction, k =
2.303
t
log
a
a- x
15. (b) A + B - C + D
Initially at t = 0 1 1 1 1
Given, t = 50 min, a = 0.5 M, a - x = 0125 . M At equilibrium 1- x 1- x 1+ x 1+ x
2.303 0.5 -1 [C ][D ] (1 + x )(1 + x ) (1 + x )2
\ k= log = 0.0277 min Keq = = =
50 0125
. [ A ][B] (1 - x )(1 - x ) (1 - x )2
Now, as per reaction 2
2H 2O 2 ¾® 2H 2O + O 2 æ1 + x ö
or 100 = ç ÷
1 d [H 2O 2 ] 1 d [H 2O] d [O 2 ] è1 - x ø
- = =
2 dt 2 dt dt 1+ x
or 10 =
d [H 2O 2 ] 1- x
Rate of reaction, - = k[H 2O 2 ]
dt or 10 - 10 x = 1 + x
d [O 2 ] 1 d [H 2O 2 ] 1 10 - 1 = x + 10 x
\ =- = k[H 2O 2 ] …(i)
dt 2 dt 2 9 = 11 x
When the concentration of H 2O 2 reaches 0.05 M, 9
d [O 2 ] 1 x= = 0.818
= ´ 0.0277 ´ 0.05 [from Eq. (i)] 11
dt 2 \ [D ] = 1 + x = 1 + 0.818 = 1818
.
d [O 2 ]
or = 6.93 ´ 10 -4 mol min-1 16. (b) Sulphide ores are concentrated by froth floatation
dt
method.
ALTERNATE Galena (PbS)
In fifty minutes, the concentration of H 2O 2 decreases from 0.5 to
17. (None) C xH y ( g ) + æç x + ö÷ O 2( g ) ¾® xCO 2( g ) + H 2O( l )
y y
0.125 M or in one half-life, concentration of H 2O 2 decreases from è 4ø 2
0.5 to 0.25 M. In two half-lives, concentration of H 2O 2 decreases
15 mL 75 mL 30 mL
from 0.5 to 0.125 M or 2 t 1/ 2 = 50 min
O 2 used = 20% of 375 = 75 mL
t 1/ 2 = 25 min
Inert part of air = 80% of 375 = 300 mL
k = æç
0.693 ö -1
\ ÷ min Total volume of gases
è 25 ø
d [O 2 ] 1 d [H 2O 2 ] k[H 2O 2 ] = CO 2 + Inert part of air = 30 + 300 = 330 mL
or =- = = 6.93 ´ 10 -4 mol min-1 x 30
dt 2 dt 2 = Þ x=2
1 15
13. (a) y
x+
Electronic 4 = 75 Þ x + y = 5
Number of unpaired
Complex ion configuration of electrons (n) 1 15 4
metal ion
Þ x = 2, y = 12 Þ C 2 H12
[Cr(H 2O)6 ]2+ Cr 2+ ; [Ar] 3 d 4 ;4
O
½½
[Fe(H 2O)6 ]2+ Fe 2+ ; [Ar] 3 d 6 ;
18. (d) Orthophosphorous acid, H 3 PO 3 : HO ¾ P ¾ OH
½
[Mn(H 2O)6 ]2+ Mn2+ ; [Ar] 3 d 5 ; H
x
[CoCl 4 ]2- Co 2+ ; [Ar] 3 d 7 ; H3 PO3 = 3 + x + 3( -2 ) = 0 or x = + 3
O O
1
COOH ½½ ½½
Pyrophosphorous acid, H 4P2O 5 : HO ¾ P ¾O ¾ P ¾OH
14. (a) H 2
OH ½ ½
3
H Cl H H
4 x
CH3
H4 P2 O 5 = 4 + 2 x + 5 (- 2 ) = 0
For C-2, order of priority of substituents is
OH > CH(Cl) (CH 3 ) > COOH 4 + 2 x - 10 = 0
For C-3, order of priority of substituents is x=+ 3
Cl > CH(OH)COOH > CH 3 Cl Cl
Hence, according to CIP rules, Cl
3
COOH 19. (a) en Co en Co en
1
H OH 2S
2 1 en
H Cl 3R Cl
3 CH cis-[Co(en) 2Cl2]Cl trans-[Co(en)2Cl2]Cl
3 (optically active) (optically inactive due
to plane of symmetry)

53
[Co(NH 3 )4 Cl 2 ]Cl can exist in both cis and trans forms that are 25. (c) Zinc metal is the most stable metal to cover iron surfaces.
given below: The process of coating the iron surface by zinc is called
Cl + NH3 + galvanisation.
H3 N NH3 H3 N Cl 26. (c) Order of first ionisation energy is Sc > Na > K > Rb
Co Co Due to poor shielding effect, removal of one electron from 4s
orbital is difficult as compared to 3s-orbital.
H3 N NH3 H3 N Cl
Cl NH3 27. (c) Hofmann-bromamide degradation reaction is given as:
trans-[Co(NH 3)4Cl2]Cl cis-[Co(NH3)4Cl2]Cl RCONH 2 + 4NaOH + Br2 ¾® RNH 2
(optically inactive) (optically inactive) (1° amine)

[Co(NH 3 )3Cl 3 ] exists in fac and mer-isomeric forms and both + Na 2 CO 3 + 2NaBr + 2H 2O
are optically inactive. Hence, four moles of NaOH and one mole of Br2 are used.
NH3 NH3
28. (b) Initially,
Cl NH3 Cl NH3
pi V
Number of moles of gases in each container =
Co Co RT1
Cl NH3 Cl Cl pi V
Total number of moles of gases in both containers = 2
Cl NH3 RT1
fac-isomer mer-isomer
pf V
(optically inactive) (optically inactive) After mixing, number of moles in left chamber =
20. (d) Zn + 4HNO 3 ¾® Zn(NO 3 )2 + 2H 2O + 2NO 2 RT1
(Conc. ) Number of moles in right chamber
4Zn + 10HNO 3 ¾® 4Zn(NO 3 )2 + N 2O + 5H 2O pV
(Dil. ) = f
RT2
21. (b) There is extensive intermolecular H-bonding in the
condensed phase. Total number of moles
pV pV pV æ1 1ö
22. (b) This water is unsuitable for drinking due to high = f + f = f ç + ÷
concentration of nitrate. In drinking water, maximum RT1 RT2 R è T1 T2 ø
permissible concentration of As total number of moles remains constant.
Lead ~- 50 ppb 2 pi V pV pV
- 50 ppb
Nitrate ~ Hence, = f + f
RT1 RT1 RT2
- 0.2 ppm
Iron ~
- 1 ppm
Fluoride ~ æ T2 ö
Þ pf = 2 pi ç ÷
23.
1
(c) 2Li + O 2( g ) ¾® Li 2O è T1 + T2 ø
2
(Excess) 29. (a)
2Na + O 2( g ) ¾® Na 2O 2 δ– δ+
HO—Cl +
( Excess) — CH2
CH3—CH— CH3—CH—CH 2—Cl
K + O 2( g ) ¾® KO 2 (Electrophilic
(Intermediate)
(Excess) addition) –
OH
NH2
CH3—CH—CH2—Cl
24. (b) Cystine : HO S COOH
C S
OH
NH2 30. (a)
COOH NBS
Cysteine :
HS hν
NH2

S C Br Br
Methionine :
H3 C OH +
NH2

NH2
H2O/K2CO3
N HO OH
Cytosine :
+
N
H
Thiol group (SH) is present in cysteine.

54
1. Equal volumes of 0.1 M AgNO3 and 0.2 M NaCl (c) Oxidation-reduction electrode–
solutions are mixed. The concentration of NO3− ions in Pt,Co 3 + ( aq )| Co 2 + ( aq ); Co 3 + ( aq ) + e − ⎯→ Co 2 + ( aq )
the mixed solution will be (d) Amalgam electrode–
(a) 0.1 M (b) 0.05 M Pt, Cl 2( g )|Cl − ( aq ); Cl 2( g ) + 2e − ⎯→ 2Cl − ( aq )
(c) 0.2 M (d) 0.15 M
7. The correct order for the wavelengths of absorption in
2. Which among the following actinoids does not have the visible region of the following complexes
stable electronic configuration?
(a) [Ni(H 2O)6 ]2 + < [Ni(NO 2 )6 ]4 − < [Ni(NH 3 )6 ]2 +
(a) Protoactinium (b) Nobelium
(c) Americium (d) Lawrencium (b) [Ni(H 2O)6 ]2 + < [Ni(NH 3 )6 ]2+ < [Ni(NO 2 )6 ]4 –

3. Which one of the following enzymes is present in (c) [Ni(NO 2 )6 ]4 − < [Ni(NH 3 )6 ]2 + < [Ni(H 2O)6 ]2 +
animals like cow, buffaloes etc., to digest compounds (d) [Ni(NH 3 )6 ]2 + < [Ni(H 2O)6 ]2 + < [Ni(NO 2 )6 ]4 −
like paper, cloth etc? 8. Fehling’s solution can make distinction between
(a) Ureaze (b) Cellulase H3C
(a) CH 3CHO and
(c) Silicase (d) Sucrase OH
4. Which buffer solution has maximum pH?
(a) Mixture of 0.1 M CH 3COOH and 0.1 M (b) CH 3CHO and HCHO

CH 3 COONa + [pK a (CH 3 COOH) = 4.74]

(b) Mixture of 0.2 M CH 3 COOH and 0.2 M CH 3COONa + (c) H3C and HCHO
(c) Mixture of 0.1 M NH 4Cl and 0.1 M NH 4OH OH
[pK a(NH+4 ) = 9.26] OH
(d) All the solutions have equal pH which is equal to 4.74 (d) CH 3CHO and C 6H 5CHO
5. The ion which is not tetrahedral in shape is 9. The absolute configuration of the following
(a) BF4− (b) NH+4 compound is
(c) [Cu(NH 3 )4 ]2 + (d) [NiCl 4 ]2 − CH3
2
6. Which of the following electrodes is wrongly matched? H
3
Cl
Cl H
(a) Metal-metal ion electrode –
Zn / Zn2 + ; Zn2 + ( aq ) + 2e − ⎯→ Zn( s ) C2H5
(b) Gas electrode–Pt,H 2( g )/H + ( aq ); 2H + ( aq ) + 2e − ⎯→ H 2( g ) (a) 2S, 3R (b) 2S, 3S (c) 2R, 3S (d) 2R, 3R

55
10. 10 g of a sample of a mixture of CaCl2 and NaCl is 19. Conductivity of 0.01M NaCl solution is found to be
treated with sodium carbonate to precipitate all the 0.00147Ω −1 cm−1 . What happens to this conductivity if
calcium as CaCO3 . This CaCO3 is heated to convert all extra 100 mL of H2 O will be added to the above
the Ca to CaO and the final mass of CaO is found to be
solution?
1.62 g. The percent by mass of CaCl2 in the original
(a) Increases
mixture is
(b) Decreases
(a) 11.0% (b) 21.8%
(c) Remains unchanged
(c) 16.2% (d) 32.2%
(d) First increases then decreases
11. An unsaturated hydrocarbon X gives white precipitate
with Tollen’s reagent. If X is gaseous in nature, what is 20. If the positions of Na+ and Cl − are interchanged in
NaCl, having fcc arrangement of Cl − ions then in the
the molecular formula of it?
unit cell of NaCl.
(a) C 3 H 6 (b) C2 H 4 (c) C 2 H 2 (d) C4 H 8
(a) Na + ions will decrease by 1 while Cl − ions will increase by 1
12. Threshold Limit Value (TLV) of CO and CO2 in the (b) Na + ions will increase by 1 while Cl − ions will decrease by 1
atmosphere should be (c) Number of Na + and Cl − ions will remain same
(a) 40 ppm, 5000 ppm (b) 43 ppm, 56 ppm (d) The crystal structure of NaCl will change
(c) 56 ppm, 43 ppm (d) 5000 ppm, 40 ppm
21. Which one of the following is the correct order of acidic
13. At two stages of disintegration, disintegration strength of hypohalous acids?
constants are respectively 1 × 10 −2 s −1 and 1 × 10 −5 s −1 .
(a) HOCl > HOI > HOBr
At first stage 2000 atoms are disintegrating. At second
(b) HOI > HOBr > HOCl
stage number of atoms disintegrating would be (c) HOCl > HOBr > HOI
(a) 2 (b) 2 × 10 6 (d) HOI > HOCl > HOBr
−6
(c) 2 × 10 (d) 2 × 10 9 22. In lanthanoids, the decrease in the atomic radius for 14
14. The only cations present in a slightly acidic medium elements is only 11 pm. What is the reason?
are Fe3 + , Zn2 + and Cu2 + . The reagent added in excess (a) Lanthanoid contraction
to this solution would identify and separate Fe3 + in one (b) Stability of oxidation state +3
step is (c) Effective nuclear charge
(a) 2M HCl (b) 6 M NH 3 (d) None of the above
(c) 6M NaOH (d) H 2 S gas 23. Among the following complexes, which one has the
15. Which of the following is arranged in the increasing largest magnetic moment?
order of bond dissociation energy? (a) [Mn(CN)6 ]3 − (b) [MnCl 6 ]3 − (c) [Fe (CN)6 ]3 − (d) [FeF6 ]3 −
(a) F2 < Cl 2 < N 2 < O 2 (b) F2 < Cl 2 < O 2 < N 2
(c) F2 < O 2 < Cl 2 < N 2 (d) N 2 < Cl 2 < O 2 < F2 24. Which of the following is the correct order of
decreasing reactivity towards nucleophilic
16. Which one of the following statements is/are not substitution reaction?
correct for the fullerenes?
(a) Allyl chloride > vinyl chloride > propyl chloride
(a) They are large cage like spheroidal molecules with general
(b) Vinyl chloride > allyl chloride > propyl chloride
formula C 2 n
(c) Allyl chloride > propyl chloride > vinyl chloride
(b) Six membered rings are fused with both six membered as
well as five membered rings (d) Propyl chloride > vinyl chloride > allyl chloride
(c) This ball shaped molecule is a non-conducting substance 25. In the commercial manufacture of ethyl alcohol from
and has 600 vertices starchy substances by fermentation process, which
(d) All of the above are incorrect enzyme order completes the fermentation reaction?
17. On addition of 1 mL solution of 10% NaCl to 10 mL gold (a) Diastase, Maltase, Zymase
sol in the presence of 0.0250 g of starch, the (b) Maltase, Zymase, Invertase
coagulation is just prevented. Starch has the following (c) Diastase, Zymase, Lactase
gold number (d) Diastase, Invertase, Zymase
(a) 0.025 (b) 0.25 (c) 2.5 (d) 25
26. The end product in the following sequence of reaction,
18. Which one of the following statements is/are in 1% HgSO CH Mg X [ O]
accordance with the Arrhenius equations? is HC ≡≡ CH ⎯⎯⎯⎯→
4
A ⎯⎯⎯→
3
B ⎯⎯→ C.
20% H2SO4
(a) Rate of a reaction, increases with decrease in activation (a) Acetic acid (b) Isopropyl alcohol
energy and increase in temperature
(c) Acetone (d) Ethanol
(b) Rate constant value, decreases exponentially with increase
in temperature 27. Which test among the following is not used for the
(c) Rate of a reaction, decreases with decrease in activation distinction among 1°, 2° and 3° aliphatic amine?
energy (a) Hinsberg’s reagent test (b) Carbylamine reaction
(d) All of the above (c) Azo dye test (d) Action with nitrous acid

56
28. An organic compound with molecular formula 35. Extraction of gold from its ore is very beneficial
C6 H12 O6 forms a yellow crystalline solid with process. The reactions involved are
phenyl-hydrazine and gives a mixture of sorbitol and O2
mannitol when reduced with sodium. Which among Roasted gold ore + CN− + H2 O ⎯→ X + OH−
the following could be the compound? X + Zn ⎯→ Y + Au
(a) Fructose (b) Glucose (c) Mannose (d) Sucrose Identify the complexes X and Y,
29. Which one of the following can possibly be used as (a) X = [Au(CN)2 ]− , Y = [Zn (CN)4 ]2 −
analgesic without causing addiction and modification?
(b) X = [Au(CN)4 ]2 − , Y = [Zn(CN)4 ]2 −
(a) Acetyl salicylic acid (b) N-acetyl-para-amino phenol
(c) Phenyl salicylate (d) Methyl salicylate (c) X = [Au(CN)2 ]− , Y = [Zn(CN)6 ]4 −
30. The increasing order of the basic character of the (d) X = [Au(CN)4 ]− , Y = [Zn(CN)4 ]2 −
substituted aromatic amine is 36. An equilibrium mixture at 300K, contains N2 O4 and
NH2 NH2 NH2 NH2 NO 2 at 0.28 and 1.1 atm pressures, respectively. If the
Me volume of the container is doubled, the new
(a) > > > equilibrium pressure of these two gases respectively
Me
are
(a) 0.064 atm and 0.095 atm
Me
(b) 0.640 atm and 0.095 atm
NH2 NH2 NH2 NH2 (c) 0.095 atm and 0.632 atm
Me (d) 0.095 atm and 0.64 atm
(b) > > > 37. The concentration of H2 SO4 required to precipitate
Me BaSO4 from a solution of 0.01 M Ba2 + ions
(solubility product of BaSO4 at 25°C is 1.0 × 10 − 9 ) is
Me
(a) 10 −8 mol/L (b) 10 −2 mol/L
NH2 NH2 NH2 NH2
−7
Me
(c) 10 mol/L (d) 10 − 6 mol/L

(c) > > > 38. Which one of the following reactions does not show
Me
oxidising action of H2 O2 ?
(a) Na 3AsO 3 + H 2 O 2 ⎯→ Na3 AsO 4 + H 2O
Me
NH2 NH2 NH2 NH2 (b) 2 Fe 2 + + H 2O 2 ⎯→ 2 Fe 3 + + 2OH −
(c) PbS( s ) + 4H 2O 2( aq ) ⎯→ PbSO 4 ( s ) + 4H 2O( l )
Me
(d) 2K 3[Fe(CN)6 ] + 2KOH + H 2O 2 ⎯→ 2K 4 [Fe(CN)6 ]
(d) > > >
+ 2H 2O + O 2
Me
Me 39. Which of the following statements is/are correct?
(a) Washing soda is prepared by Solvay process and also
31. Triple ionised beryllium [Be ] has the same orbit
3+
called as soda ash
radius as that of the ground state of hydrogen atom.
(b) Solvay process is economical discontinuous
At which energy state, this situation arises?
(c) Washing soda when mixed with K 2CO 3, called as nitrating
(a) 2 (b) 4 (c) 3 (d) 1 mixture
32. The pair in which both species have iron is (d) All of the above
(a) nitrogenase, cytochromes
(b) carboxypeptidase, haemoglobin
40. An alkane has a molecular mass of 72. The possible
(c) haemocyanin, nitrogenase
chain isomers for this molecule are
(d) haemoglobin, cytochromes (a) 4 (b) 3
(c) 5 (d) 2
33. A 5L vessel contains 1.4 g of nitrogen, when heated to
1800 K, 30% of molecules are dissociated into atoms. Directions (Q. Nos. 41 to 60) In the following questions
Calculate the pressure of the gas at 1800 K. a statement of Assertion followed by a statement of
(a) 1.92 atm (b) 19.2 atm (c) 0.192 atm (d) 2.63 atm
Reason is given. Choose the correct answer out of the
34. Consider the following reaction at 298 K, following choices.
3
O2 (g) ⎯→ O3 (g) (a) Both Assertion and Reason are true and Reason is the
2 correct explanation of Assertion
The K p for this conversion is 2.47 × 10 − 29 . The Δ rG° for (b) Both Assertion and Reason are true but Reason is not the
this conversion is correct explanation of Assertion
(a) 163 kJ mol −1 (b) 206 kJ mol −1 (c) Assertion is true but Reason is false
(c) 183 kJ mol −1 (d) 136 kJ mol −1 (d) Both Assertion and Reason are false

57
41. Assertion (A) Water has a very high heat capacity. 53. Assertion (A) Boiling point of H5C2 C2H5
Reason (R) ΔH = C p ⋅ ΔT is high and there is extensive C C
H H
hydrogen bonding present among water molecules. I
H C2H5
42. Assertion (A) Anhydrous salts with smaller cations can is higher than C C
be prepared by heating the hydrated salts. H5C2 H
II
Reason (R) Smaller cations are inactive. Reason (R) Dipole moment of I is higher than II.
43. Assertion (A) Rise of sea levels is recorded by atleast 54. Assertion (A) Among [PbCl4 ]2 − , [Ni(CN)4 ]2 − and
25 meters (82 feet) by the year 2100. 2− 2−
[NiCl4 ] , only [NiCl4 ] shows tetrahedral geometry.
Reason (R) The warming effect of earth due to certain Reason (R) Cl − is a weak field ligand.
atmospheric gases such as CO2 and water vapour.
55. Assertion (A) The monomer of natural rubber is
44. Assertion (A) Al is obtained by high temperature isoprene.
reduction of alumina with carbon.
Reason (R) Natural rubber is formed through cationic
Reason (R) Alumina reacts with carbon to form addition polymerisation.
aluminium carbide which decomposes at high 56. Assertion (A) The ionic size of S2 − > Cl − .
temperature to form Al while carbon is oxidised to CO.
Reason (R) In the isoelectronic species, greater the
45. Assertion (A) Lead (IV) iodide is an unstable compound. nuclear charge, less is the size.
Reason (R) Iodine stabilises lower oxidation state. 57. Assertion (A) Ce4 + is stable.
46. Assertion (A) pH of 10 − 8 M HCl is not equal to 8. Reason (R) This is because of the half-filled d-orbitals.
Reason (R) HCl does not dissociate properly in very 58. Assertion (A) The boiling points of the chlorinated
dilute solution. derivatives of methane increase with number of
chlorine atoms.
47. Assertion (A) H2 O is dipolar, whereas BeF2 is not.
Reason (R) An increase in the induced dipole-dipole
Reason (R) The value of dipole moment is not zero in
attractive forces.
case of H2 O while it comes out to be zero in case of
BeF2 . 59. Assertion (A) Corrosion is an electrochemical
phenomenon.
48. Assertion (A) PhCOOH (I) is more acidic than Reason (R) A metal is oxidised by the loss of electrons
COOH to oxygen and forms metal oxide.
60. Assertion (A) The loss of a proton from an amine gives
an imide ion, while the loss of a proton from alcohol
gives an alkoxide ion.

II Reason (R) RO − is more stable than RNH.
Reason (R) The negative charge of the conjugate base
of (I) is delocalised through the benzene ring.
49. Assertion (A) Aqueous gold colloidal solution is
red in colour.
Reason (R) The colour arises due to scattering of light
by colloidal gold particles.
50. Assertion (A) In an atom, the velocity of an electron in
the higher orbits keeps on increasing.
Reason (R) Velocity of electron is directly proportional
to the radius of the orbit.
51. Assertion (R) If helium is allowed to expand in
vacuum, it liberates heat.
Reason (R) The inversion temperature of helium is
very low.
52. Assertion (A) Time taken for completion of 75% of first
order reaction is double of its t1 / 2 .
Reason (R) Time taken for completion of any fraction of
first order reaction is proportional to the extent of
completion of reaction.

58
Answers with Explanation
1. (b) When 1L of 0.1 M AgNO 3 is mixed with 1 L of 0.2 M NaCl If lowest priority substituent on asymmetric carbon occupies
solution. Then, 0.1 mole of AgNO 3 will react with 0.1 mole of either of horizontal position, change the configuration from R
NaCl to form 0.1 mole of NaNO 3 in the solution. This solution to S and vice-versa.
will ionise to produce 0.1 mole NO −3 ions, but new volume
3′
after mixing becomes 2 L. Hence, concentration of NO 3− ions CH3
0.1 4′ 2 1′
in the mixed solution = = 0.05 M. H Cl
2 1′′
2′
2′′ 4′′
2. (a) Protoactinium [Pa] is the element that does not have Cl H
3′′
stable electronic configuration C2H5
Protoactinium [Pa] = [Rn] 5f 2 6 d 17 s 2 (Atomic number = 91)
Nobelium (No) = [Rn] 5f 14 6 d 07 s 2 (Atomic number = 102) (2S, 3S)

Lawrencium [Lr] = [Rn] 5f 14 6 d 17 s 2 (Atomic number = 103) 10. (d) The reaction involved in this process are
CaCl 2 + NaCl + Na 2CO 3 ⎯→ CaCO 3 + 3NaCl
Americium [Am] = [Rn] 5f 7 6 d 07 s 2 (Atomic number = 95)
CaCO 3 ⎯→ CaO + CO 2
3. (b) According to above reactions, 1 mole of CaCl 2 gives 1 mole
of CaCO 3 which further give 1 mole of CaO after heating.
4. (d) pK a = pH
1.62
(a) pH = 4.74 (b) pH = 4.74 Q Mole of CaO formed = = 0.0289 mol
56
(c) pOH = 14 − pH ⇒ pH = 14 − pK a = 14 − 9.26 or pH = 474
. . ∴ Mole of CaCl 2 formed = 0.0289 mol
5. (c) [Cu(NH 3 )4 ]2 + is square planar in shape or mass of CaCl 2 formed = 0.0289 × 111 = 3.208 g
3d9 4s 4p [Q Molar mass = 111 g mol −1]
Cu2+=[Ar] Percentage of CaCl 2 in the original mixture
3.208
dsp2 4p = × 100 = 32.08%
2+
10
[Cu(NH3)4] = [Ar] xx xx xx xx
11. (c) With Tollen’s reagent (ammoniacal AgNO 3), a white
NH3 NH3 NH3 NH3 precipitate of silver salt is obtained.
H ⎯ C ≡≡ C ⎯ H + 2AgNO 3 + 2NH 4OH ⎯→
6. (d) Generally amalgam electrode used for active metals like Ag ⎯ C ≡≡ C ⎯ Ag + 2NH 4NO 3 + 2H 2O
Na, K etc. Silver acetylide
Zn(Hg) | Zn2 + , Zn2 + ( aq ) + 2e − ⎯→ Zn(Hg)
(white ppt. )
e.g.
12. (a) The permissible limit of a pollutant in the atmosphere to
7. (c) As metal ion is fixed, the increasing field strengths which a person is exposed for 8 h work day or 40 h work week
(CFSE values) of the ligands from the spectrochemical series or throughout his life, there is no adverse effect on him, is
are in the order : H 2O < NH 3 < NO −2 . called threshold limit value (TLV). TLV of CO is 40 ppm while
Thus, the energies absorbed for excitation will be in the order that of CO 2 is 5000 ppm. As the TLV decreases, poisonous
[Ni(H 2O)6 ]2 + < [Ni(NH 3 )6 ]2 + < [Ni(NO 2 )6 ]4 − effect increases.
As E =
hc
, the wavelengths absorbed will be in the 13. (b) N1λ1 = N2λ 2
λ λ1 1 × 10 −2
opposite order. N2 = N1 = 2000 × = 2 × 10 6 atoms
λ2 1 × 10 −5
8. (d) (a) Fehling’s solution reacts with both CH 3CHO and
14. (b) Fe 3+ , Zn2 + and Cu2 + ions are present in slightly acidic
α-hydroxy ketones H3 C α solution. On adding 6 M NH 3 solution, i.e. 6 M NH 4OH we get
OH the following reactions.
Fe 3 + + 3OH − ⎯→ Fe(OH)3
Dark brown ppt.
(b) Fehling’s solution reacts with both aliphatic aldehydes,
CH 3CHO and HCHO, respectively. Zn2 + + 4 NH 3 ⎯→ [Zn (NH 3 )4 ]2 +
Colourless solution
(c) Fehling’s solution reacts with both HCHO and
Cu2 + + 4 NH 3 ⎯→ [Cu (NH 3 )]24 +
Deep blue solution
α-hydroxy ketone H3C α In this way, dark brown precipitate of Fe(OH)3 can be separated
OH
from Cu2 + and Zn2 + amine complex solutions in a single step
OH by adding 6 M NH 3.
(d) Fehling’s solution reacts only with aliphatic aldehyde, 15. (b) F2 < Cl 2 < O 2 < N 2
not with aromatic aldehyde.
155 242 494 941 (kJ/mol)
9. (b) If lowest priority substituent on asymmetric carbon In general, as the size of atom or multiplicity of bond increases,
occupies vertically upward or downward position in Fischer bond dissociation energy increases. But bond dissociation
projection, the configuration obtained is equal to the actual energy of F2 is less than that of Cl 2 because of the small size of
configuration of the molecule. F. Thus, the correct order is F2 < Cl 2 < O 2 < N 2.

59
16. (b) This ball shaped molecule has 60 vertices and each one is Maltase
C 12 H 22O11 + H 2O ⎯⎯→ 2C 6 H12O 6
occupied by one carbon atom. Zymase
The alkali compounds of C 60 are used as superconducting C 6 H12O 6 ⎯⎯→ 2C 2 H 5OH + 2CO 2
substance at the temperature range of 10-40 K. 1% HgSO CH MgX
26. (c) HC ≡≡ CH 20% 4 3
⎯⎯⎯→ CH 3CHO ⎯⎯⎯→ CH 3CH(OH)CH 3
17. (d) Amount of starch in milligram that prevents coagulation by H SO 2 4 A H 2O B
1 mL of 10% NaCl solution = 0.025 × 1000 = 25 ↓ [O ]
Hence, gold number = 25 CH 3COCH 3
C
18. (a) Rate constant value increases exponentially with increase Acetone
in temperature. 27. (c) Azo dye test is not used for the distinction in 1°, 2° and 3°
Rate of a reaction increases with decrease in activation energy. aliphatic amines. While other three tests can be easily used to
19. (b) On dilution, number of ions/cm3 decreases. Hence, distinguish between 1°, 2° and 3° amines.
conductivity decreases.
20. (b) In NaCl with fcc arrangement of Cl − ions, number of Cl − ion
Secondary Tertiary
Test Primary amine
amine amine
= 14, Na + ions = 13. On interchanging their positions, Cl − ions
will be 13 and Na + ions will be 14. 1. Hinsberg’s Forms a Forms a No reaction
+ +1 +1 reagent test sulphonamide, sulphonami
21. (c) HOCl > HO Br > H OI soluble in de insoluble
Acidic strength of oxoacids of different halogens having alkali. in alkali.
same oxidation state, decreases with the increase in atomic 2. Carbylamine Forms a No action No reaction
number. reaction carbylamine
with
Oxygen is more electronegative than the halogen (Cl, Br or I).
unpleasant
Therefore, it attracts the electrons of the oxygen-halogen
smell.
bond towards itself. Now, as the electronegativity of the
halogen decreases from Cl to I, the shared pair of electrons of 3. Action with Forms primary Forms Forms an
the oxygen-halogen bond moves closer towards the oxygen nitrous acid alcohol and nitrosamine amine salt
evolves which gives
atom. As a result, electron-density of the oxygen atom
nitrogen with Libermann’s
increases from Cl to I. Consequently, O in H — O — Cl attracts effervescence. nitrosa urine
the electrons of O — H bond towards itself most strongly reaction.
followed by H — O — Br and least strongly by
H — O — I. O — H bond breaks most readily in H — OCl and 28. (a) As C 6 H12O 6 forms a crystalline osazone with phenyl
least easily in H — O — I. hydrazine, it must be an aldohexose or ketohexose. As the
22. (a) Lanthanoid contraction is the reason. As we move along the compound on reduction with Na gives sorbitol and mannitol,
lanthanoid series, the nuclear charge increases by one unit at therefore the given compound is fructose.
each successive element. The new electron is added into the CH2OH
same subshell. As as result, the attraction on the electrons by
the nucleus increases and this tends to decrease in the size. C O CH2OH CH2OH
Further, as the new electron is added into the f-subshell, there is
H C H HO C H H C OH
imperfect shielding of one electron by another in this subshell
+4 [H]
due to the shapes of these f-orbitals. This imperfect shielding is 2HO C H HO C H + HO C H
unable to counter balance the effect of the increased nuclear
charge. Hence, the net result is a contraction in the size though H C OH H C OH H C OH
the decrease is very small.
H C OH H C OH H C OH
23. (d) Magnetic moment of a complex is related to the presence
of number of unpaired electrons. CH2OH CH2OH CH2OH
[Mn(CN)6 ]3 − = two unpaired electrons Fructose Mannitol Sorbitol
[MnCl 6 ]3 − = four unpaired electrons 29. (b) N-acetyl-para-amino phenol can possibly be used as
[Fe(CN)6 ]3 − = one unpaired electron analgesic without causing addiction and modification.
[FeF6 ]3 − = five unpaired electrons 30. (b) In case of aniline, the lone pair of electron of nitrogen is
delocalised into benzene ring by resonance. As a result of
24. (c) Vinyl chloride is least reactive towards nucleophilic resonance the lone pair is less available for protonation.
substitution due to
The basicity of o-substituted anilines is generally lower as
(i) resonance stabilisation
compare to aniline and m-or p-substituted aniline. This is due
(ii) more s-character of carbon, attached to halogen.
to ortho-effect in which the substituent at ortho-position
Allyl chloride is more reactive, since, allyl carbocation formed
hinders the attack of the acid on the amino acid.
is resonance stabilised.
+ + In a case of p -methylaniline, + I and hyperconjugation effect
CH 2 == CH ⎯CH 2 ←→ CH 2 ⎯ CH == CH 2 of methyl group increases the electron density on nitrogen
25. (a) Diastase, Maltase, Zymase atom while in case of m-methylaniline only + I-effect works.
n Therefore, p-methylaniline is more basic than
(C 6H10O 5 )n + H 2O ⎯Diastase
⎯ ⎯⎯→ nC 12 H 22O11 m-methylaniline.
2

60
31. (a) For hydrogen like species (Be 3 + ) radius is given as, If volume of the container is doubled the pressure will reduced

rn =
n 2h 2
4 π 2mKZe 2
…(i)
to half
N 2O 4 - 2NO 2
⎛ 1.1 ⎞
New pressure ⎛⎜ − p⎞⎟
0.28
1 ⎜ + 2 p⎟
where, K = ,Z = 4 ⎝ 2 ⎠ ⎝ 2 ⎠
4 π ∈0 2
For hydrogen atom radius is given as, ⎛ 1.1 + 2 p⎞
⎜ ⎟
⎝ 2 ⎠
h2 Kp = = 4.32
rn = …(ii) ( n = 1, Z = 1) ⎛ 0.28 − p⎞
4 π mKZe 2
2
⎜ ⎟
⎝ 2 ⎠
Dividing Eq. (i) by Eq. (ii)
On solving, p = 0.045
rn n2 n2 ∴ p N 2O 4 = 0.14 − 0.045 = 0.095 atm
= =
r1 Z 4 p NO 2 = 0.55 + (2 × 0.045) = 0.64 atm
when rn = r1 37. (c) Precipitation will take place when ionic product is greater

-
⇒ n2 = 4 ⇒ or n = 2 than solubility product.
2+
Thus, the second orbit of Be 3 + has the same radius as the BaSO 4 Ba + SO 24 −
0.01 S
Bohr’s radius of hydrogen atom.
Q Ksp = [Ba 2 + ] [SO 24− ]
32. (d) Nitrogenase contains nitrogen
Cytochromes contains iron 1.0 × 10 − 9
S = = 10 − 7mol/L
Carboxypeptidase contains carbon and nitrogen 0.01
Haemoglobin contains iron 38. (d) 2K 3[Fe(CN)6 ] + 2KOH + H 2 O 2 ⎯→

-
Haemocyanin contains copper
2K 4 [Fe (CN)6 ] + 2H 2O + O 2
33. (a) N2 2N Reduction of potassium ferricyanide to potassium
14
. ferrocyanide. H 2O 2 has reducing property because of its
Initial moles = 0.05 0
28 electron donating property.
⎛ 30 ⎞
After dissociation 0.05 − ⎜ × 0.05⎟ 2 × 0.015 H 2 O 2 + 2OH − ⎯→ 2H 2O + O 2 + 2e − (in basic medium)
⎝ 100 ⎠
∴ Total number of moles after dissociation H 2O 2 ⎯→ O 2 + 2H + + 2e − (in acidic medium)
= 0.035 + 0.030 = 0.065, 39. (a) Solvay process is economical, continuous and self
i.e n = 0.065 mol, V = 5 L, T = 1800 K, p= ? contained. The raw material used for the process are only
nRT NaCl and CaCO 3 that are quite cheap and the biproduct of the
From pV = nRT , we get p =
V reaction is only calcium chloride, which has no large scale
0.065 mol × 0.0821 L atm K −1mol −1 × 1800 K industrial applications but NH 3 has large scale applications, is
= synthesise by other organic compounds or reagents.
5L
40. (b) General molecular formula of alkane is C n H 2n + 2
= 1.92 atm
Q Molecular mass = 72
34. (a) Δ rG°= − 2.303 RT log K p ∴ 12 n + 2 n + 2 = 72
Δ rG° = −2.303 ( 8.314 JK −1 mol −1 ) × (298 K ) (log 2.47 × 10 −29 ) n=5
The alkane is C 5 H12. The possible chain isomers are :
= 163229 J mol −1 ≈ 163 kJ mol −1
(i) CH 3 ⎯ CH 2 ⎯ CH 2 ⎯ CH 2 ⎯ CH 3
35. (a) X = [Au(CN)2 ]− , Y = [Zn(CN)4 ]2 − Pentane
Extraction of gold involves, leaching of the metals present in the (ii) CH 3 ⎯CH ⎯ CH 2 ⎯ CH 3
ore with CN − ions. This is also an oxidation reaction because ⏐
during the leaching process, Au oxidises to Au + which then CH 3
combines with CN − to form its respective soluble complexes. 2-methylbutane
4Au( s ) + 8 CN − ( aq ) + 2H 2O ( l ) + O 2( g ) CH 3
Gold

⎯→ 4 [Au(CN)2 ] ( aq ) + 4OH ( aq ) − ⏐
Soluble complex
(iii) CH 3 ⎯ C ⎯ CH 3
The metals are extracted from these complexes by reduction ⏐
CH 3
or displacement method by using a more electropositive
zinc metal. 2, 2-dimethylpropane
2[Au(CN)2 ]− ( aq ) + Zn( s ) ⎯→ 2 Au( s ) + [Zn(CN)4 ]2− ( aq ) 41. (a) Due to extensive hydrogen bonding in water, it has a very
36. (d) N 2O 4 ( g ) -
2NO 2 ( g ) high heat capacity. Because ΔH = C p ⋅ ΔT, ΔT =
ΔH
Cp
, which
Pressure at equilibrium 0.28 1.1
2 means that a large amount of heat is needed to raise the
p NO (1.1)2
Kp = 2
= = 4.32 atm temperature of an aqueous solution by 1 K. This property is
p N 2 O4 028
. important in regulating the temperature of a cell from the heat
generated by metabolic processes.

61
42. (d) Anhydrous salts with smaller cations cannot be prepared 50. (d) Correct Assertion In an atom, the velocity of electron in
by heating the hydrated salts as the tendency for the smaller the higher orbits keep on decreasing.
cations to get hydrated is so strong. Correct Reason Velocity of electron is inversely proportional to
Thus, anhydrous BaCl 2 may be prepared by heating the the radius of the orbit.
hydrated salts beyond 127°C Z
Velocity = 2.18 × 10 6 × .
Heat r
BaCl 2 ⋅ 2H 2O ⎯→ BaCl 2 + 2H 2O
However, analogous reaction cannot be carried out for 51. (a) He shows heating effect at room temperature as it
magnesium chloride. possesses very low values of inversion temperature.
Heat 52. (c) For a first order reaction t 1/ 2 does not depend upon the
MgCl 2 . 6H 2O ⎯→ MgCl(OH) + HCl + 5H 2O
initial concentration of reaction. t 1/ 2 for first order reaction is
MgCl(OH) ⎯→ MgO + HCl 0.693
given as t 1/ 2 = , k = rate constant.
43. (a) The increase of greenhouse gases concentrations k
(mainly carbon dioxide) led to a substantial warming of the 53. (a) Boiling point of isomeric compounds depends on
earth and the sea, called global warming. In other words, the dipole-dipole interactions and cis-isomers have greater
increase in the man-made emission of greenhouse gases is dipole moments (more polar) than trans -isomers (less polar).
the cause for global warming. There are many major effects of
global warming such as increase of temperature on the earth 54. (b) Correct explanation The complex, in which size of
by about 3° to 5°C and rise of sea levels by atleast 25 metres transition metal is small and ligand is weak field, has
by the year 2100. tetrahedral shape and sp3 hybridisation Ni 2+ = [Ar ] 3 d 8 4s 0

44. (d) Al is extracted by high temperature electrolytic reduction 3d 8 sp3


2+ 2–
of Al 2 O 3 to which fluorspar and cryolite have been added. Ni in [NiCl 4)] = [Ar] ×× ×× ×× ××
Al 2O 3 reacts with carbon at high temperatures to form Al 4C 3 Cl– Cl– Cl– Cl–
which does not decompose. 3
sp = Tetrahedral geometry
45. (a) Common oxidation state of IVA group elements are
+ 2 and + 4 but as we move down the group +2 oxidation state 55. (c) Correct Reason Natural rubber is formed through free
becomes more stable. Thus, Pb 4 + is not possible, i.e. PbI 4 is radical addition polymerisation.
highly unstable. 56. (a) Among isoelectronic species, ionic radii increase with
46. (c) Correct Reason In very dilute solutions both source of H ⊕ increase in negative charge.
ions from HCl and H 2O must be considered and also due to 57. (c) Correct Reason The electronic configuration of, Ce is
common ion (H + ) supression, ionisation occurs.
Ce 58 = [Xe ] 4f 1 5 d 2 6s 2 (predicted)
47. (a) The structure of H 2O is angular (V-shape) with = [Xe ] 4f 2 5 d 0 6s 2 (observed)
sp3-hybridisation and it shows bond angle equal to 104.5°. Its Ce 4 + = [Xe ] 4f 0 5 d 0 6s 0
dipole moment ( μ ) value is positive or more than zero. Since, in + 4 oxidation state, all, (i.e 4f, 5d and 6s) orbitals are
Hence, μ > 0 empty therefore, Ce ion gains the stable configuration of
O nearest inert gas, hence Ce 4 + is most stable.

H
104·5º
H
58. (a) The boiling points of the chlorinated derivatives of
methane increase with number of chlorine atoms because of
But in BeF2, structure is linear due to sp-hybridisation and its an increase in the induced dipole attractive forces.
dipole moment (μ ) value is zero. Thus, due to positive μ value
H 2O is dipolar and due to zero μ value BeF2 is non-polar. CH 3 Cl CH 2 Cl 2 CHCl 3 CCl 4
Boiling point 249 K 313 K 334 K 350 K
48. (c) PhCOOH (I) is more acidic than (II) due to −I and
+ R-effects of Ph group, while (II) is less acidic due to 59. (a) Corrosion is the oxidative deterioration of a metal, such as
presence of only + I-effect, so Assertion is true. Reason is conversion of iron to rust [Fe 2O 3 ⋅ xH 2O], the tarnishing of
false silver (due to formation of Ag 2O), development of a green
coating on copper and bronze. Corrosion of iron (rusting)
occurs in the presence of water and oxygen (air).
C
60. (a) The loss of a proton from an amine gives an imide ion while
O
the loss of a proton from alcohol gives an alkoxide ion as shown

Since O − of COO − can delocalise to > C == O group but not R ⎯ NH 2 ⎯→ R ⎯ NH + H +
with the benzene ring because O − is not conjugation with −
R ⎯ O ⎯ H ⎯→ R ⎯ O + H +
⎯ C ==C ⎯ bond of benzene ring.
Since, oxygen is more electronegative than nitrogen,
49. (a) The colour is due to scattering. It depends upon the size of therefore, ( RO −− ) can accommodate the negative charge more
gold sol particles. Finest gold sol has red colour. As the size of
easily

than RNH. In other words, RO − is more stable than
the particles increases, it becomes purple then blue and
RNH. Thus, alcohols are more acidic than amines.
finally golden yellow.
Conversely, amines are less acidic than alcohols.

62
COMPLETE
STUDY
PRACTICE &
ASSESSMENT

CLASS
XI

F246 `450 F244 `475 F240 `450 F245 `455

CLASS
XII

F206 `440 F225 `450 F208 `440 F211 `380

AVAILABLE IN
Physics, Chemistry, Mathematics, Biology, Accountancy, Business Studies,
Economics, Computer Science, Informatics Practices, English Core, Hindi Core

• •
1. β-elimination reaction is carried out with base (B) as Codes
shown below: A B C
H (a) I III V
(b) II III VI
B H2C—CH2 CH2=
=CH2+BH+X (c) I IV V
X (d) II IV VI
The following bases are used for above reaction. 5. 600 mL of ozonised oxygen at STP was found to weigh
I. RO – II. NO3– 1g. The volume of ozone in the ozonised oxygen is

III. RCOO –
IV. CN (a) 150 mL (b) 50 mL
– (c) 200 mL (d) 100 mL
V. OH
The decreasing order of reactivity for the above 6. The decreasing order of reactivity towards
elimination is electrophilic addition of the following is
(a) I > V > IV > III > II (b) IV > V > II > I > III I. CH ≡≡ CH II. CH2 == CH2
(c) V > I > II > III > IV (d) III > I > IV > V > II
III. H2 C == CH ⎯ Cl IV. H C == CH
2. An ester used as medicine is. ⏐ ⏐
(a) ethyl acetate (b) methyl acetate Cl Cl
(c) methyl salicylate (d) ethyl benzoate (a) II > I > III > IV
3. Which of the following statements is correct? (b) I > II > III > IV
(a) La(OH)3 is the least basic hydroxide among hydroxides of (c) IV > III > I > II
lanthanides (d) IV > III > II > I
(b) Ce 4 + can act as an oxidising agent 7. When 1g of an alloy of Al and Mg reacts with excess
(c) Ln (III) compounds are generally colourless HCl it forms AlCl3 , MgCl2 and H2 . The evolved
(d) None of the above H2 collected over mercury at 0°C, occupied 1200 mL at
4. Which compound in each of the following pairs will 699 mm Hg. The composition of alloy with respect to
− Al and Mg is, respectively.
react faster via SN 2 mechanism using O H nucleophile.
A. CH3 Br (I) and CH3 I (II) (a) 46.4% and 53.6%
(b) 58.2% and 41.8%
B. (CH3 )3 C ⎯ Cl (III) and CH3 Cl (IV)
(c) 54.2% and 45.8%
C. (V) and (VI)
Br Cl (d) 41.8% and 58.2%

64
O
14. Central chlorine atom in ClF3 is sp3d-hybridised and
– theoretically three structures are possible
OH
8. H2O
P. The product P is F
F F F
Cl F Cl F Cl
OH OH OH
F
F F
A B C
(a) (b) (c) (d)
The most probable structure is
(a) Structure A
OH OH
(b) Structure B
9. A mixture of volatile components A and B, has total (c) Structure C

-
vapour pressure (in torr) (d) All these three structures have same probability
p = 254 − 119χ A 15. Consider the reaction, A(g) B(g) + C(g)
where, χ A is the mole fraction of A in the mixture. If K P at 400°C is 1.5 × 10 − 4 and K P at 600°C is 6 × 10 − 3 .
Hence, p0A and p0B are (in torr) Choose the incorrect statement.
(a) 119, 254 (b) 154, 119 (c) 254, 119 (d) 135, 254 (a) The reaction is exothermic
10. Which one of the following statements is/are correct? (b) Increase in pressure, increases the formation of A
(a) AgCl with liquid ammonia forms a complex [Ag(NH 3 )2 ]+ (c) Increase in temperature, increases the formation of B
(b) Ammonium dichromate on heating, decomposes to give (d) Decrease in temperature and increase in pressure shift the
nitrogen and a green coloured compound equilibrium towards left
(c) CaNCN produces a white precipitate and a gas on 16. Consider the following statement(s).
hydrolysis. This gas turns, the moistened filter paper with I. Glucose exists in two different crystalline forms
copper sulphate solution, to deep blue α-D-glucose and β-D-glucose
(d) All of the above II. α-D-glucose and β-D-glucose are enantiomers
11. The brown haze of photochemical smog is largely III. Starch is a mixture of amylose and amylopectin,
attributed to both contain unbranched chain of α-D-glucose
(a) NO (b) NO 2 units
O IV. α-D-glucose and β-D-glucose are anomers
⏐⏐ (a) Both I and II (b) Only III
(c) CH 3 C OONO 2 (d) CH 2 == CHCHO (c) Both II and III (d) Both III and IV
12. Consider the following sequence of reaction 17. Which of the following statements is correct?
3+ Excess of SCN− (a) Blackened oil painting can be restored into original form by
Fe (aq) ⎯ ⎯ ⎯ ⎯ ⎯ ⎯ ⎯⎯→ Blood red colour the action of MnO 2
A
– (b) Pseudo-alum does not have Na + and K+
Colourless Excess of F (c) Zeolites have a more closed structure than feldspar
B (d) H 4 As 2O 7 is an ortho acid
The spin only magnetic moment of (B) is
18. The uncertainty in position of an electron is equal to its
(a) 35 BM (b) 48 BM de-Broglie wavelength. The minimum percentage
(c) 15 BM (d) 24 BM error in its measurement of velocity under this
13. Consider the following reaction, circumstance will be approximately.
(a) 18 (b) 22
(c) 8 (d) 4
+ C2H5 MgBr
(i) Ether, Δ
P
19. The chemical reaction,
(ii) H3O+ 2AgCl(s) + H2 (g) ⎯→ 2HCl(aq) + 2 Ag(s)
CH3
taking place in a galvanic cell, is represented by the
The product P is notation.
(a) Pt( s )| H 2( g ), 1 bar | 1M KCl( aq )| AgCl( s )| Ag( s )
CH3
HO CH3 HO (b) Pt( s )| H 2( g ), 1 bar | 1M HCl( aq )| 1M Ag+ ( aq )| Ag( s )
(c) Pt( s )| H 2( g ), 1 bar | 1M HCl( aq )| AgCl( s )| Ag( s )
(a) (b) (d) Pt( s )| H 2( g ), 1 bar | 1M HCl( aq )| Ag( s )| AgCl( s )
CH3
CH3 20. 0.3605 g of a metal is deposited on the electrode by
passing 1.2 A current for 15 minutes through its salt.
The valency of metal if its atomic weight is 96, will be
(c) (b) Both (a) and (b) (a) 2 (b) 3
CH3 (c) 6 (d) 4

65
21. Which of the following is the best method for the 28. 4% solution of sucrose C12 H22 O11 is isotonic with 3%
synthesis of ester, (H3 C)3 C ⎯ COOCH3 ? solution of an unknown organic substance. The
(a) (H 3C)3C ⎯ COCl + CH 3OH ⎯→ molecular mass of the unknown substance is
(b) (H 3C)3C ⎯ COOH + CH 2 N 2 ⎯→ (a) 282.2 (b) 261.8
(c) (H 3C)3C ⎯ COOH + CH 3OH ⎯→ (c) 256.5 (d) 232.7
(d) (H 3C)3C ⎯ COCl + CH 3ONa ⎯→ 29. The geometries of XeF4 , XeO4 and XeO2 F2
22. In the minerals, haematite and magnetite, the respectively, are
oxidation states of the metal respectively are (a) tetrahedral, square pyramidal, square planar
(a) + 2, + 3 in haematite and + 2 in magnetite (b) square pyramidal, square planar, tetrahedral
(b) + 3 in haematite and + 2, + 3 in magnetite (c) square planar, tetrahedral, trigonal bipyramidal
(c) + 2, + 3 in haematite and + 3 in magnetite. (d) tetrahedral, square planar, square pyramidal
(d) + 2 in haematite and + 2, + 3 in magnetite
30. Consider the following reaction sequence.
23. A metal crystallises in a face centred cubic unit HBr Hydrolysis
cell with edge length 0.560 nm. If it contains 0.1% CH2 == CH2 ⎯ ⎯⎯→ A ⎯ ⎯ ⎯ ⎯ ⎯⎯→ B
Na2 CO3
Schottky defects then the density of metal is ⎯ ⎯ ⎯ ⎯⎯→ C
(Atomic mass of metal = 40g / mol). I2 excess
(a) 1.92 g cm− 3 (b) 2.14 g cm− 3 (a) C 2 H 5 OH (b) CH 3 CHO
−3
(c) 1.51 g cm . g cm− 3
(d) 376 (c) CHI 3 (d) C 2 H 5 I
24. Which of the following boiling point orders is incorrect 31. The solubility product of a salt having formula M2 X3 is
for amines?
2.2 × 10 −20 . If the solubility of an another salt having
(a) For isomeric amines : 1° > 2 ° > 3°
formula M2 X is twice, molar solubility of M2 X is
(b) C 4H 9 NH 2 > (CH 3 )2N ⎯ C 2 H 5 > (C 2 H 5 )2 NH
(a) 3 × 10 −12 . × 10 −5
(b) 916
(c) NH2 H3C N H N CH3 (c) 4.58 × 10 −5 (d) 2.76 × 10 −18

(d) None of the above


32. Consider the following reaction and give the structure
of A.
25. Which of the following compounds is correctly IUPAC OH
O O
named? Ba(OH) 2
NO2 A
+ H
Cl
OH OH OH
(a) (b)
O2N NO2 O2N Cl OH OH OH OH

CH3 (a) (b)


(4-chloro-1,3-dinitrobenzene) (4-methyl-5- chloronitrobenzene)
OH OH OH OH OH

OH OH
(c) (d)
Cl H
(c) (d)
(3-ethyl-1,1-dimethylcyclohexane) (1-chloropropanal)
OH OH OH OH OH
26. In an adsorption experiment, a graph between log (x / m)
and log p was linear with a slope of 45° and intercept of 33. Which one of the following statements is correct?
0.310. Under a pressure of 0.5 atmosphere, the amount (a) Among HF, CH 4 , CH 3OH and N 2O 4 intermolecular hydrogen
of gas adsorbed per gram of the adsorbent is bonding is expected in all
(a) 2 (b) 1 (b) Among O 2+ , O 2, O −2 and O 22 − , O 22 − has the strongest
(c) 3 (d) 4 O ⎯ O bond
27. Which of the following complexes is correctly IUPAC (c) Hydrogen bonding is the dominant intermolecular force or
named? bond that must be overcome in converting liquid CH 3OH to a
(a) [CoCl(NH 3 )5 ] CO 3 gas
Chloropentaamminecobalt (III) carbonate (d) All of the above
(b) K 4 [Ni(CN)4 ]
34. At two stages of disintegration, constants are
Potassium tetracyanonickel (0)
2 × 10 − 5 s − 1 and 2 × 10 − 8 s − 1 respectively. At first stage
(c) K 2[Zn(OH)4 ]
200 s atoms are disintegrating. At second stage number
Potassium tetrahydroxozinc (III)
of atoms disintegrating will be
(d) Fe 4 [Fe(CN)6 ]3
(a) 2 (b) 2000
Iron (III) hexacyanoferrate (II)
(c) 2 × 10 6 (d) 2 × 10 − 6

66
35. Analysis shows that nickel oxide consists of nickel ions 41. Assertion (A) Buta-1, 3-diene is the monomer of Gutta
with 96% ions having d8 configuration and 4% having Percha.
d7 configuration. The formula of the oxide is best Reason (R) Gutta Percha is formed through anionic
represented by addition polymerisation.
(a) Ni 0.96 O1.00 (b) Ni 0.98 O 0.98
(c) Ni1.00O1.02 (d) Ni1.02 O1.21
42. Assertion (A) t-butyl bromide gives 2, 2, 3, 3-tetramethyl
butane on reaction with sodium metal in dry ether.
36. In an insulated container 1 mol of a liquid with molar Reason (R) t-alkyl halides readily undergo
volume 100 mL, is kept at 1 bar. When the volume of Wurtz reaction.
liquid decreases by 1mL, the liquid is steeply taken to
100 bar. The ΔH for the process is 43. Assertion (A) The molecular masses of polymers
(a) 8400 bar mL (b) 9900 bar mL cannot be calculated using the freezing point or
(c) 9240 bar mL (d) 7430 bar mL boiling point method.
37. The nature of bonds in the dichromate dianion, are Reason (R) The boiling point method for determining
(a) six equivalent Cr ⎯ O bonds and one Cr ⎯ Cr bond
the molecular masses is used for compounds stable at
(b) six equivalent Cr ⎯ O bonds and one Cr ⎯ O ⎯ Cr bond
higher temperature.
(c) four equivalent Cr ⎯ O bonds 44. Assertion (A) The dissociation of NH3 on hot platinum
(d) six non - equivalent Cr ⎯ O bonds surface may be of zero order or first order.
38. Which one of the following is not the example of first Reason (R) Pressure of the gas affects the order of
order reaction? reaction.
(a) Acid-catalysed hydrolysis of ethyl acetate 45. Assertion (A) The reaction between NH3 and MnO4−
(b) Inversion of sucrose in the presence of acid
occurs in an acidic medium
(c) Oxidation of I – ion by S 2O 28 − ion
NH3 + MnO4− ⎯→ MnO2 + NO2
(d) Hydrolysis of tertiary butyl halide using alkali
Reason (R) MnO4– is reduced to MnO2 in acidic
39. Consider the following reaction,
CH3 medium.
⏐ 46. Assertion (A) Using Pt electrodes, at the end of
HO
CH2 == CH ⎯ CH ⎯ Cl ⎯ ⎯2 ⎯→ electrolysis, an aqueous solution of CuSO4 turns
Which of the following statement is incorrect? colourless.
CH 3 Reason (R) CuSO4 changes to Cu(OH)2 during
⏐ electrolysis.
(a) The product is a mixture of CH 2 == CH ⎯ C H ⎯ OH (I) and
47. Assertion (A) According to Werner's theory, ligands
HO ⎯ CH 2 ⎯ CH == CH ⎯ CH 3 (II) (major)
are connected to the metal ions by covalent bonds and
(b) The product is CH 2 == CH ⎯ C H ⎯ OH (I) (major) secondary valencies have non directional properties.

CH 3 Reason (R) Secondary valencies are ionisable.
(c) Allyl chloride is reactive towards both SN 1 and SN 2 48. Assertion (A) The actinoids exhibit more number of
mechanisms but more reactive towards SN 1 mechanism oxidation states in general than the lanthanoids.
(d) Formation of (II) takes place by an allylic rearrangement Reason (R) The 5f orbitals extend farther from the
40. For the preparation of sodium thiosulphate by Spring’s nucleus than the 4f orbitals.
reaction, the reactants used are 49. Assertion (A) RS– is a stronger nucleophile and a better
(a) Na 2 S + Na 2 SO 3 + Cl 2 leaving group than RO − .
(b) Na 2 S + SO 2 Reason (R) RS– is a weaker base than RO − .
(c) Na 2 SO 3 + S
50. Assertion (A) The heat of neutralisation of a strong
(d) Na 2 S + Na 2 SO 3 + I 2
acid with a strong base is equal to heat of ionisation of
water.
Directions (Q. Nos. 41-60) In the following questions a
Reason (R) Water ionises to a very small extent while
statement of Assertion followed by a statement of
H+ ions form an acid which combines very rapidly with
Reason, is given. Choose the correct answer out of the −
OH from a base to form H2 O.
following choice.
51. Assertion (A) HNO3 renders iron passive.
(a) Both Assertion and Reason are true and Reason is the
correct explanation of Assertion Reason (R) Iron reacts with HNO3 to form ferric nitrate.
H C
(b) Both Assertion and Reason are true but Reason is not the 52. Assertion (A) 3 CHO undergoes Aldol condens
correct explanation of Assertion H3C
(c) Assertion is true but Reason is false -ation reaction.
(d) Both Assertion and Reason are false Reason (R) Compounds containing α - H atom undergo
Cannizzaro reaction.

67
53. Assertion (A) The boiling point of hexane (341K) is Reason (R) Carbon atom in ethene is sp hybridised while
greater than CF4 (144K). it is sp2 hybridised in ethyne.
Reason (R) Hexane and CF4 have the same molecular 58. Assertion (A) Two different bond lengths are observed
in PF5 molecule but only one bond length is observed in
mass and are non-polar in nature.
SF6 .
54. Assertion (A) Electronaffinity is positive, when O −
Reason (R) PF5 has trigonal bipyramidal structure and
changes to O2 − .
SF6 has octahedral structure.
Reason (R) O − repels the incoming electron due to
similar charge and needs energy to accept the 59. Assertion (A) Precisely 1 mole of helium and 1 mole of
electron. neon are placed in a container, then the molecules of
55. Assertion (A) PhNH3 Br is more acidic than NH4 Br. helium strike the wall more frequently.
+
Reason (R) PhNH3 (anilinium ion) is resonance Reason (R) Molecules of helium have lesser average
stabilised. molecular speed.

CaCO3 (s) -
56. Assertion (A) For a reaction,
CaO(s) + CO2 (g); ΔH > ΔE
Reason (R) Enthalpy change is always greater than
60. Assertion (A) The decreasing order of the anti- knocking
value of octane number is
C4 H10 > C3 H8 > C2 H6 > CH4
internal energy change. Reason (R) As the number of carbon-atoms increases,
57. Assertion (A) C⎯H bond in ethyne is shorter than knocking power decreases.
C ⎯H bond in ethene.

Answers with Explanation


1. (a) The reagent should be a strong Bronsted base. 6. (a) The order of electrophilic addition is alkene > alkyne. But if
Acidic order HNO 3 > RCOOH > HCN > H 2O > ROH alkene has electron withdrawing group, reactivity of alkene
− becomes less than that of alkyne.
Basic order NO –3 – –
< RCOO < CN < OH < RO –
Due to EWG, electron density at (C == C ) or nucleophilicity
Hence, decreasing order of basicities for β-elimination is decreases, so addition of electrophile to C == C becomes slow.
I > V > IV > III > II. So, the decreasing order of electrophilic addition is
2. (b) Methyl salicylate is used to relive muscular aches, pains alkene (II) > alkyne (I) > alkene with one EWG (III)
and rheumatic conditions. > alkene with two EWGs(IV).
3. (b) La(OH)3, is the most basic hydroxide among hydroxides 7. (c) The reactions involved are as follow:
3
of lanthanides. Ln(III) compounds are generally coloured due Al + 3HCl ⎯→ AlCl 3 + H 2
to the presence of unpaired f-electrons that undergo 2
f-f transition. Mg + 2HCl ⎯→ MgCl 2 + H 2
Let, the mass of Al = x g
4. (d) A. (II), I – is a better leaving group than Br –
∴ Mass of Mg = (1 − x ) g
B. (IV), 1° alkyl halide undergoes faster SN 2 reaction than
699
3° alkyl halide × 1.2
PV
C. (VI), Vinyl halide (V) does not undergo SN 1 or SN 2 Mole of H 2( n) = = 760 = 0.0492
RT 0.0821 × 273
reaction as nucleophile is repelled by the π-electron
3 x (1 − x )
cloud of alkene. Since (VI) is 1° alkyl halide therefore it Mole of H 2 = × + 1× = 0.0492
undergoes via SN 2 mechanism. 2 27 24
0.0555 x + 0.042 (1 − x ) = 0.0492
5. (c) Let, the volume of O 3 in ozonised oxygen = x mL
⇒ 0.0135 x = 7.24 × 10 − 3
Volume of O 2 present in ozonised oxygen = ( 600 − x ) mL
x = 0.542 g
22400 mL of O 3 and O 2 at STP will weigh 48 g and 32 g,
respectively. ⇒ % Al = 542 . %, % Mg = (100 − 54.2 ) % = 45.8% .
x × 48
The weight for x mL of O 3 = g 8. (a) The reaction takes place as follows
22400
( 600 − x ) O OH
The weight for ( 600 − x ) mL of O 2 = × 32
22400
OH H2O
Total weight of ozonised O 2 ( 600 mL ) is
48 x ( 600 − x ) × 32
+ =1
22400 22400 OH
H H
48 x + 19200 − 32 x = 22400 Acidic
16 x = 3200 hydrogen atoms
x = 200 mL

68
9. (d) pTotal = pA0 χ A + pB0 χ B OH
= pA0 χ A + pB0 (1 − χ A ) (i) Ether, Δ
+ H3C MgBr
= pB0 − ( pB0 − pA0 ) χ A … (i) (ii) H3O+
CH3 CH3
p = 254 − 119 χ A [Given] CH3
Tautomerisation
On comparing with the equation (i), we get
∴ pB0 = 254
pB0 − pA0 = 119
CH3
⇒ pA0 = 254 − 119 = 135
P CH3
10. (d) AgCl + 2NH 3 ⎯→ [Ag(NH 3 )2 ] + + Cl −
14. (c) The most stable structure will be the one with lowest
Δ
(NH 4 )2Cr2O 7 ⎯⎯→ N 2 + Cr2O 3 + 4H 2O energy or with minimum steric repulsion. The order of
repulsion between different kinds of electron pairs is
CaNCN + 3H 2O ⎯→ CaCO 3 + 2NH 3 lp − lp > lp − bp > bp − bp.
White ppt. Also, the less is separation, more is repulsion. Thus,
11. (b) Photochemical smog is mainly composed of ozone and repulsion between two electron pairs at 90° will be more than
nitrogen dioxide (NO 2 ). NO 2 is introduced in atmosphere via when they are at 120°.
vehicular emissions and industrial processes. This NO 2 is (i) In structure A, there are 6 repulsions between lp − bp
photolysed by solar radiations to produce nitrogen oxide at 90°
• •
(NO) and an unpaired oxygen atom (O). O combines with O 2 (ii) In structure B, there is one 90°, lp − lp repulsion, and
molecule to produce ozone (O 3 ). three 90°, lp − bp repulsion.
However, when volatile organic compounds (VOCs) are
In structure (C ), there are four 90° lp-bp repulsions and no
present in the atmosphere, these oxidise NO to NO 2 without

-
90° lp-lp repulsion. Therefore, structure (C ) is most probable.
breaking down any ozone molecule in this process. These
NO 2 molecules form brown haze of photochemical smog in 15. (a) For A( g ) B( g ) + C ( g )
atmosphere. (a) With increase of temperature, K P increases, i.e. with
12. (a) The given reaction sequence is increase of temperature, the reaction is favoured in
Excess of F − forward direction and hence, reaction is endothermic
Fe 3 + ( aq ) + SCN − ⎯→ Fe(SCN)3 ⎯ ⎯ ⎯ ⎯ ⎯ ⎯ ⎯→ [FeF6 ]3 −
(b) Δng = 1 + 1 − 1 = 1
Excess Blood red Colourless
Δng = positive, i.e. with the increase of pressure,
colour (B)
reaction is favoured in backward direction and hence,
(A)
the formation of A( g ) increases
In [FeF6 ]3 − , Fe 3 + has electronic configuration 3d 5, F − being
(c) Increase of temperature favours forward reaction and
weak field ligand, cannot pair up the electrons of 3d-orbital.
hence, the formation of B increases
Hence, [FeF6 ]3 − shows sp3d 2 hybridisation to possess
(d) From the statements (a) and (b), the reaction is favoured
octahedral shape with five unpaired 3d electrons.
in backward direction with decrease of temperature and
3d 4s 4p increase of pressure
Fe3+(Free state)= 16. (c) II is incorrect because α-D-glucose and β-D-glucose are
not enantiomers but are anomers.
3d III is incorrect because starch is a mixture of amylose and
Fe3+(in hybridised state) =
amylopectin. Amylose is a linear polymer of α-D-glucose but
i.e. in [FeF6]3–
amylopectin is highly branched polymer of α-D-glucose.
sp3d2 4d
xx xx xx xx xx xx 17. (b) (a) Old oil paintings becomes black due to the formation of
black PbS, H 2O 2 converts it into white PbSO 4 .
Thus, spin magnetic moment, PbO + H 2 S ⎯→ PbS + H 2O
White Black
μ = n ( n + 2 ) BM
PbS + 4H 2 O 2 ⎯→ PbSO 4 + 4H 2O
= 5 ( 5 + 2 ) BM White
= 35 BM (b) Pseudo-alum constitutes a class in which monovalent
atom of true alum is replaced by a divalent atom. Hence,
13. (c) α, β-unsaturated ketone gives predominantly 1,4-addition
Na + and K + are not present in pseudo-alums.
product with Grignard’s reagent.
(c) Feldspar has more closed structure than zeolites
(d) H 4 As 2 O 7 is a pyro acid.

69
h 25. (c)
18. (c) We know de-Broglie wavelength ( λ ) = 6
1 Cl
p 5
(a) 4
According to Heisenberg uncertainty principle,
2
h O2N NO2
Δ x ⋅ Δp ≥ 3
4π 1-chloro-2,4-dinitrobenzene
On neglecting ‘>’ sign NO2
h h
× Δp = 4
p 4π 3
h (b)
(As uncertainty in position ( Δx ) = ) 2
p 1 Cl
Δp 1
= CH3
p 4π 2-chloro-1-methyl-4- nitorobenzene
m × Δv 1
=
m×v 4π (d) 3
Δv 1 1
= Cl 2 H
v 4π 3-chloropropanal
Δv 1
∴ % (uncertainty in velocity) = × 100 = × 100 = 8%.
v 4π 26. (b) According to Freundlich adsorption isotherm,
x
19. (b) The cell representation can be represented as = k p1/ n
m
Pt( s )| H 2( g ) (1 bar) | 1MHCl( aq )| 1 M Ag+ ( aq )| Ag( s )
Taking logarithm on both the sides
Anode Cathode 1
log ( x / m) = log k + log p
20. (b) Amount of electricity passed, n
Q = it = 1.2 × 15 × 60 = 1080 C Plot of log ( x / m) vs log p is linear with slope = 1 / n and
Mass of metal deposited by passing intercept = log k
1080 C electricity = 0.3605 1
Mass of metal deposited by passing Slope = = tan 45° = 1
n
0.3605
96500 C electricity = × 96500 = 32.2g ∴n=1
1080
Intercept, log k = 0.3010
∴ Equivalent mass of metal = 32.2g mol −1
or k=2
Atomic mass = 96 g mol −1 x
Atomic mass 96 = kp1/ n = 2 × ( 0.5)1 = 10
.
Valency = = =3 m
Equivalent mass 32.2
27. (d) Correct names of (a), (b) and (c) respectively, are
21. (b) The esterification of an alcohol with RCOCl or with RCOOH (a) Pentaamminechlorocobalt (III) carbonate.
is affected by steric hindrance. Bulky group on either alcohol (b) Potassium tetracyanonickelate (0)
or acid derivatives slow down the reaction.
(c) Potassium tetrahydroxozincate (II)
So, the esterification with diazomethane (CH 2 N 2 ) is the best
method. 28. (c) Since, the two solutions are isotonic, they must have same
concentrations in moles/litre for sucrose solution,
22. (b) In haematite, i.e. Fe 2O 3 oxidation state of Fe is Concentration of sucrose solution
2 x + 3 × (− 2 ) = 0 40
= 40g / L = mol L− 1
∴ x=3 342
Magnetite, i.e. Fe 3 O 4 is an equimolar mixture of FeO and For unknown substance, suppose m is the molecular mass,
Fe 2O 3 then
∴ Oxidation states of iron in FeO is + 2 and in Fe 2 O 3 is + 3. Concentration of unknown substance
30
23. (c) Due to Schottky defect, the vacant spaces will increase = 30g /L = mol L− 1
Mw
resulting decrease in number of atoms per unit cell. As 0.1%
30 40
Schottky defects are present, Hence, decrease in number of ∴ =
atoms in fcc unit cell is, Mw 342
4 × 01 . 30 × 342
Z =4− = 3.996 Mw = = 256.5 g mol −1
100 40
Z ×M
Density, ρ = 29. (c)
N0 × a 3 F
3.996 × 40 g mol − 1
= F F O
(6.022 × 1023 mol − 1 ) (0.560 × 10 − 7 cm)3 Xe
−3
Xe Xe
= 1.51g cm . O
F F O O F
24. (b) The correct boiling point order for option (b) is O Hybridisation-sp3d
C 4H 9 NH 2 > (C 2 H 5 )2 NH > (CH 3 )2 N ⎯ C 2 H 5 Hybridisation-sp3d2 Hybridisation-sp3 Geometry-Trigonal
(As for amines boiling point follows the order 1° > 2° > 3°). Geometry-square planar Geometry-Tetrahedral bipyramidal or see-saw

70
30. (c) CH2==CH2 HBr
CH3 CH2 Br
Hydrolysis
CH3 CH2 OH Let, the number of O 2 − ions present in the crystal = x
A B Applying electroneutrality rule,
Na2CO3 I2 excess Total positive charge = Total negative charge
0.96 × 2 (positive charge) + 0.04 × 3 (positive charge)

-
CHI3 + HCOO
C = x × 2 (negative charge)
31. (a) M 2 X 3 2 M 3 + + 3 X 2− ∴( 0.96 × 2 ) + ( 0.04 × 3) − 2 x = 0
2S 3S ∴ x = 102
.
Therefore, formula of crystal is
Solubility product Ksp = (2S )2 ( 3S )3
Ni1.00 O1.02
= 108S 5 ( S = Solubility)
36. (b) At pressure 1 bar
−20
or 2 ⋅ 2 × 10 = 108S 5 Volume of 1 mol of liquid = 100 mL
or S = 4.59 × 10 −5 At pressure 100 bar
Again for M 2 X
M2X - 2 M + + 3 X 2−
2S ′ S′
Volume of 100 mol liquid = 99 mL
Since, the process is steeply changed from
1 bar to 100 bar, it is irreversible process.
′ = (2 S ′)2 ⋅ (S ′) = 4S ′ 3
Ksp ∴ w = − p( V2 − V1 ) = − 100 × ( 99 − 100) = 100 bar mL
or ′ = 4 (2 × 4.59 × 10 −5 )3
Ksp (QS ′ = 4 × S ) ΔU = q + w
⇒ ΔU = w
∴ ′ = 3 × 10 −12
Ksp
[Qq = 0, adiabatic nature due to insulation]
32. (b) Aldehydes react faster than ketones as they are good ΔU = 100 bar mL
electron acceptors. Therefore, carbanion formed by the ΔH = ΔU + pΔV = ΔU + ( p2V2 − p1V1 )
removal of α − H-atom of ketone with base, adds to the
= 100 + (100 × 99 − 1 × 100)
C-atom of (CH == O ) group to give β-hydroxy ketone.
= 9900 bar mL
OH

OH OH
37. (b) In Cr2O 27 − ion, there are six equivalent Cr ⎯ O bonds and
Ba(OH)2
H
one Cr ⎯ O ⎯ Cr bond
α H – OH O O
O
H O Cr Cr O
OH OH OH OH OH OH OH –
O O–
Required product
38. (c) It is a second order reaction, first order with respect to both
S 2O 28 − and l –
33. (c) Intermolecular hydrogen bond is present in HF and
CH 3 OH ∴ r = k[S 2 O 28 − ] [l – ]
O 2( 8 + 8 = 16 electrons ) All other options except(c) are the examples of first order
= σ 1s 2, σ *1s 2, σ 2 s 2, σ *2 s 2, π 2 pz2, reaction.
π2 px2 ≈ π 2 py2, π *2 p1x ≈ π *2 p1y 39. (b) The reaction takes place as follows:
10 − 6 CH3
Bond order = =2
2
10 − 5 CH2==CH—CH—Cl CH2==CH—CH ⎯CH 3
O 2+ (electrons = 8 + 8 − 1) = 15; BO = = 2.5
2 Allylic carbocation
10 − 7
O –2 (electrons = 8 + 8 + 1) = 17 ; BO = = 15
.
2 OH
10 − 8 CH2 —CH==CH—CH—CH3
O 22 − (electrons = 8 + 8 + 2 ) = 18; BO = =1
2 CH3 OH OH
1
Bond order ∝ ; ∝ Bond strength
Bond length CH2==CH—CH—OH CH2 —CH==CH—CH3
Q O 2+ has the minimum bond length as its bond order is (I) (II)
Less stable due More stable due
maximum to less-substituted to more-substituted
The O ⎯ O bond is strongest alkenol alkenol
The molecules of CH 3OH are associated by H-bonding. 40. (d) Spring’s method is used for the preparation of sodium
34. (c) N1λ1 = N2λ 2 thiosulphate (hypo).
λ1 2 × 10 −5 I 2 + Na 2 S + Na 2 SO 3 ⎯→ 2NaI + Na 2 S 2 O 3
N2 = N1 = 2000 ×
λ2 2 × 10 − 8 Sodium Sodium Sodium thiosulphate
sulphide sulphite
= 2 × 10 6 atoms
41. (d) Gutta Percha is synthetic rubber and its monomer is
35. (c) Ni (Atomic number = 28) = 3 d 8 4s 2 isoprene.
∴ Ni 2 + = 3d 8 and Ni 3 + = 3d 7 Since, isoprene contains electron donating group, therefore it
Hence, 96% ions of Ni 2 + and 4% ions of Ni 3 + are present. is prepared by cationic addition polymerisation.

71
42. (d) t -BuBr gives isobutene on reaction with strong base. Which is reverse of the heat of neutralisation and the value
t -alkyl halides undergo dehydrohalogenation in the presence of of heat is equal but sign is reverse.
strong base such as alc. KOH rather than Wurtz reaction. 51. (c) Correct reason is HNO 3 reacts with Fe to form a thin
β CH3 CH3
impervious layer of ferrosoferric oxide
OH H —CH2—C—Br H2C= =C—CH3 (FeO ⋅ Fe 2O 3 ≡≡ Fe 3O 4 ) on the surface of iron which
Isobutene protects it from further reaction.
Acidic H-atom
CH3
52. (d) Compounds containing α - H-atom undergo aldol
∴ 1° and 2° RX undergo Wurtz reaction, while 3° RX undergoes
dehydro halogenation to give alkenes. condensation but the α H-atom of compound H3C CHO.
H 3C
43. (a) The molecular masses of large molecules are determined
using the osmotic pressure technique. The molecular masses does not undergo aldol condensation. It undergoes
of smaller molecules are determined using the freezing point Cannizzaro reaction because the mobility of α -H-atom is
lowering method. arrested by two sterically hindered methyl groups.
44. (a) At low pressure, it is first order reaction while at high 53. (b) Hexane is a straight-chain alkane (or unbranched), so it
pressure, it is a zero order reaction. is less compact or has more surface area and hence has
The expression for the rate of decomposition of NH 3 on Pt higher boiling point than CF4 . Whereas, CF4 is spherical and
surface is has less approachable area for intermolecular attractive
k1 pNH 3 force and hence, has lower boiling point.
Rate = , where k1 and k 2 are rate constants for
1 + k 2 pNH 3 54. (a) When O − changes into O 2 − , change is endothermic. The
different steps involved during reaction. reason is that O − repels the incoming electron due to similar
When pressure of ammonia is very small then charge, hence it needs energy to accept the electron.
k 2 pNH 3 < < 1 Hence, electronaffinity is positive.
Rate = k1 pNH 3 , i.e. first order 55. (c) NH 3 is a stronger base than aniline; therefore, the
+
When pressure of ammonia is high then k 2 pNH 3 > > 1 conjugate acid of aniline, i.e. PhNH 3 is more acidic than the
k1 pNH 3 k +
Rate = = 1 =k conjugate acid of NH 3, i.e. NH 4 ion.
k 2 pNH 3 k2
56. (c) Δng = 1 − 0 = 1
i.e. zero order.
ΔH = ΔE + Δng RT = Δ E + RT
45. (d) MnO –4 is reduced to MnO 2 in a mild basic medium or neutral
ΔH > ΔE
medium whereas in an acidic medium, MnO 4– is reduced to
Mn2 + and in a strong basic medium, it is reduced to MnO 24 − . 57. (c) The carbon atom is sp2 hybridised in ethene while
2+
46. (c) Assertion is correct. Since, Cu ions are reduced sp hybridised in ethyne.
to Cu and deposited at cathode. H 2O is oxidised at anode 58. (a) Trigonal bipyramidal structures
(SO 24 − is not oxidised)
(PF5, P has sp3 d -hybridisation) have two different type of
1
H 2 O ⎯→ 2H + + 2e − + O 2 ; 2H + + SO 24 − ⎯→ H 2 SO 4 bonding two are axial 90° away from a plane and three are
2
equatorial in the plane. The equatorial bonds are 120° away
The solution becomes colourless due to the formation of from each other. Since, the bonding is different, the bond
H 2 SO 4 in solution. lengths are not expected to be the exactly same. In SF6,
47. (d) Ligands are attached to the metal ion by coordinate bonds, S-atom has sp3d 2-hybridisation showing octahedral
which are directional and are called secondary valencies in arrangement with each bond angle at 90° to the other and
case of complex ion. Secondary valencies are non-ionisable.
thus, bond lengths are same.
48. (a) As 5f orbitals extend farther from the nucleus than
4f orbitals, the attraction on the 5f electrons in actinoid series 59. (c) Helium gas has lower molecular mass and hence it
decreases. Hence, they can easily participate in bonding and, moves faster than neon and strikes the wall more frequently.
therefore exhibit more number of oxidation states. 8RT
μ av =
49. (a) RSH has more acidity as compare to ROH and RS − has less πM
basicity than RO − . 1/ 2
μ av ∝ ⎛⎜ ⎞⎟
1
When nucleophilic centres are different and belong to the ⎝M⎠
same group, nucleophilicity is antiparallel to basicity. Hence, helium has higher speed than neon.

Nucleophilicity R S > RO −
Weaker the base, better is the leaving group.
60. (d) The correct decreasing order of anti-knocking value of
So, the leaving group ability of RS − is greater than RO − . octane number is CH 4 > C 2 H 6 > C 3 H 8 > C 4H10

-
50. (b) Dissociation of water is Because as the number of C-atoms increases, knocking
– power increases or anti-knocking power decreases.
H 2O H + + OH

72
General Instructions
● All questions are compulsory.

● Question number 1 to 5 are very short

answer questions and carry 1 mark each.


● Question number 6 to 10 are short

answer questions and carry 2 marks


each.
● Question number 11 to 22 are also short

answer questions and carry 3 marks


each.
● Question number 23 is a value based

question and carry 4 marks.


● Question number 24 to 26 are long

answer questions and carry 5 marks


each.
● Use log tables, if necessary, use of

calculators is not allowed.

1. Write the IUPAC name of the given compound, 9. When a coordination compound CoCl3 ⋅6NH3 is
CH2—CH2—OH mixed with AgNO3 , 3 moles of AgCl are precipitated
per mole of the compound. Write
(i) structural formula of the complex.
2. Write the structure of an isomer of compound (ii) IUPAC name of the complex.
C4H9Br, which is the most reactive towards
SN1 reaction.
10. Write the structures of the following compounds:
(i) BrF3 (ii) XeF4
3. What is the reason for the stability of colloidal sols?
or
4. Give an example each of a molecular solid and an What happens when
ionic solid.
(i) SO2 gas is passed through an aqueous solution
5. Pb(NO3 )2 on heating gives a brown gas which of Fe3 + salt?
undergoes dimerisation on cooling. Identify the
gas. (ii) XeF4 reacts with SbF5 ?
6. For a reaction, H2 + Cl2 ⎯ ⎯ hν
⎯→ 2HCl 11. Write the final product(s) in each of the following
Rate = k reactions:
CH3
(i) Write the order and molecularity of this reaction.

(ii) Write the unit of k. (i) CH3 ⎯ C ⎯ O ⎯ CH3 + HI ⎯→
7. Write the chemical equations involved in the ⏐
following reactions: CH3
(i) Hofmann-bromamide degradation reaction (ii) CH3 ⎯ CH2 ⎯ C H ⎯ CH3 ⎯Cu/573
⎯⎯⎯⎯ K

(ii) Carbylamine reaction ⏐
8. (i) Gas (A) is more soluble in water than gas (B) at OH
(i) CHCl + aq. NaOH
the same temperature. Which one of the two (iii) C6H5 ⎯ OH ⎯ ⎯⎯⎯⎯⎯⎯⎯⎯⎯
3
⎯→
gases will have the higher value of KH (ii) H+
(Henry’s constant) and why?
12. How do you convert
(ii) In non-ideal solution, what type of deviation
(i) chlorobenzene to biphenyl?
shows the formation of maximum boiling
azeotropes? (ii) propene to 1-iodopropane?

73
(iii) 2-bromobutane to but-2-ene? 19. For the first order thermal decomposition reaction,
or the following data were obtained.
Write the major product(s) in the following C2H5Cl(g) ⎯→ C2H4 (g) + HCl(g)
reactions: Time/s Total pressure/atm
CH2—CH3 0 0.30
(i) Br2, UV light 300 0.50
?
Calculate the rate constant.
O 2N (Given, log 2 = 0.301, log 3 = 0.4771, log 4 = 0.6021)
20. Give reasons for the following.
Na (i) Aniline does not undergo Friedal-Crafts
(ii) 2CH3—CH—CH3 ? reaction.
Dry ether

Cl (ii) (CH3 )2 NH is more basic than (CH3 )3 N in an


aqueous solution.
AgCN (iii) Primary amines have higher boiling point than
(iii) CH3—CH2—Br ?
tertiary amines.
13. (i) Write the structure difference between starch 21. Define the following terms.
and cellulose. (i) Lyophilic colloid (ii) Zeta potential
(ii) What type of linkage is present in nucleic acids? (iii) Associated colloids
(iii) Give one example each for fibrous protein and 22. Calculate the boiling point of solution when 4 g of
globular protein. MgSO4 (M = 120 g mol − 1) was dissolved in 100 g of
14. (i) Name the method of refining of nickel. water, assuming MgSO4 undergoes complete
(ii) What is the role of cryolite in the extraction of ionisation. (Kb for water = 0.52 K kg mol − 1)
aluminium?
23. Due to hectic and busy schedule, Mr Singh started
(iii) What is the role of limestone in the extraction of taking junk food in the lunch break and slowly
iron from its oxides? became habitual of eating food irregularly to excel
15. Give reasons. in his field. One day during meeting, he felt severe
(i) SO2 is a reducing while TeO2 is an oxidising chest pain and fell down. Mr Khanna, a close friend
agent. of Mr Singh, took him to doctor immediately. The
(ii) Nitrogen does not form pentahalides. doctor diagnosed that Mr Singh was suffering
(iii) ICl is more reactive than I2 . from acidity and prescribed some medicines.
3+ Mr. Khanna advised him to eat home-made food
16. (i) For the complex [Fe(H2O)6 ] , write the and change his lifestyle by doing yoga, meditation
hybridisation, magnetic character and spin of
and some physical exercise. Mr Singh followed his
the complex. (Atomic number of Fe = 26)
friend’s advice and after few days, he started feeling
(ii) Draw one of the geometrical isomers of the better.
complex [Pt(en)2 Cl2 ] 2 + which is optically
After reading the above passage, answer the
inactive. following.
17. An element crystallises in a bcc lattice with (i) What are the values (atleast two) displayed by
edge length of 500 pm. The density of the element is Mr Khanna?
7.5 g cm− 3 . How many atoms are present in 300 g of (ii) What are antacids? Give one example.
the element? (iii) Would it be advisable to take antacids for a long
18. (i) What is the role of sulphur in the vulcanisation period of time? Give reason.
of rubber?
24. (a) Write the structures of A and B in the following
(ii) Identify the monomers in the following polymer. reactions.
O O H , Pd- BaSO H N ⎯ OH
(i) CH3COCl ⎯ ⎯⎯⎯⎯⎯
2 4
→ A ⎯ ⎯⎯⎯⎯
2
→B
(i) CO2 PCl
—O—CH2—CH2—O—C— —C— (ii) CH3MgBr ⎯ ⎯⎯
⎯+→ A ⎯ ⎯
⎯5→ B
n (ii) H3 O

(iii) Arrange the following polymers in the (b) Distinguish between


increasing order of their intermolecular forces. (i) C6H5 ⎯ COCH3 and C6H5 ⎯ CHO
Terylene, Polythene, Neoprene (ii) CH3COOH and HCOOH

74
(c) Arrange the following in the increasing order of (a) The conductivity of 0.001 mol L−1 solution of
their boiling points. CH3COOH is 3.905 × 10−5 S cm− 1. Calculate its
CH3CHO, CH3COOH, CH3CH2OH molar conductivity and degree of dissociation (α).
or Given, λ ° (H + ) = 349.6 S cm2 mol − 1
(a) Write the chemical reaction involved in and λ ° (CH3COO − ) = 40.9 S cm2 mol − 1
Wolff-Kishner reduction.
(b) Define electrochemical cell. What happens if
(b) Arrange the following in the increasing order of external potential applied becomes greater than
their reactivity towards nucleophilic addition °
Ecell of electrochemical cell?
reaction.
C6H5COCH3 , CH3 ⎯ CHO, CH3COCH3 26. (a) Account for the following.
(i) Mn shows the highest oxidation state of + 7 with
(c) Why carboxylic acids do not give reactions of
oxygen but with fluorine, it shows the highest
carbonyl group?
oxidation state of + 4.
(d) Write the product in the following reaction
(i) (i-Bu) AlH (ii) Cr 2 + is a strong reducing agent.
CH3CH2CH==CH ⎯ CH2CN ⎯ ⎯⎯⎯⎯⎯
2 ⎯→ (iii) Cu 2 + salts are coloured while Zn 2 + salts are
(ii) H2 O
white.
(e) A and B are two functional isomers of compound
C3H6O. On heating with NaOH and I2 , isomer B (b) Complete the following equations.
Δ
forms yellow precipitate of iodoform whereas (i) 2MnO2 + 4KOH + O2 ⎯⎯ →
isomer A does not form any precipitate. Write (ii) Cr2O27 − + 14H + + 6I− ⎯→
the formulae of A and B.
or
25. (a) Calculate Ecell
° for the following reaction at 298 K.
The elements of 3d transition series are given as
2Al(s) + 3Cu2 + (0.01 M) ⎯→ 2Al3 + (0.01M) + 3Cu(s)
Sc Ti V Cr Mn Fe Co Ni Cu Zn
Given: Ecell = 1.98 V
Answer the following.
(b) Using the E° values of A and B, predict which is (i) Write the element which shows maximum
better for coating the surface of iron number of oxidation states. Give reason.
[ E°( Fe2 + / Fe) = − 0.44 V] to prevent corrosion and (ii) Which element has the highest melting point?
why? (iii) Which element shows only + 3 oxidation state?
Given, E °( A2 + / A ) = − 2.37 V : E °( B2 + / B ) = 0.14 V (iv) Which element is a strong oxidising agent in
or + 3 oxidation state and why?

CHEMISTRY

Water Molecules Break Bonds Through Quantum Tunneling


Water, one of the most common substances on Earth, has served up yet another scientific surprise. In its
liquid state, water molecules cling to one another through so-called hydrogen bonding, constantly
making and breaking bonds as they jumble about. Within the smallest possible 3D droplet of water, which
consists of just six molecules, those molecules can rearrange themselves not just one at a time, but in
sets of two. These two molecules can simultaneously break their hydrogen bonds with their neighbors
and rotate off one another like gears.
The reconfiguration takes place through a subtle effect called quantum tunneling, in which the droplet
does not have enough energy to wriggle from one configuration to the other, but simply pops from one to
the other as if burrowing through an energy barrier, a team of chemists reports today in Science. And
because the dual bond–breaking could play a role in how water behaves in cells and on mineral
interfaces, the substance continues to be a font of insight.

75
Answers with Explanation
1. 2 1
CH2—CH2 —OH
10. (i) F (ii)
F F
F Br Xe
2-phenylethan-1-ol F F
CH3
2. F
BrF3 XeF4
CH3 C CH3 (Square planar)
(T-shaped)
Br
or
3. The reasons for stability of colloidal particles are : (i) SO 2 gas reduces Fe 3 + to Fe 2 + .
I The presence of equal and similar charges on the colloidal
SO 2 + 2Fe 3 + + 2H2O ⎯→ 2Fe 2 + + SO 24 − + 4H +
particles that prevents coagulation.
I Colloidal sols are covered by a sheath of liquid due to which (ii) XeF4 reacts with covalent pentafluoride (SbF5 ) to form an
these are extensively solvated. adduct.
The constant rapid zig-zag motion of colloidal particles, known XeF4 + SbF5 ⎯→ XeF4 ⋅ SbF5 or [XeF3+ ] [SbF6− ]
as Brownian movement. This movement of particles prevents
them from settling down due to gravity. 11. CH3

4. Molecular solid — Solid CO 2 (i) CH3 —C—I


+
CH 3OH
+
Ionic solid — NaCl − CH3

5. NO 2 gas, reactions involved are given as below: (ii) CH3 —CH2—C—CH3 + H2


Δ
2Pb(NO 3 )2 ⎯⎯ → 2PbO + 4NO 2 + O2 O
(Brown gas)
On cooling OH
2NO 2 ⎯ ⎯⎯⎯⎯→ N2O 4
CHO
6. (i) Since, rate does not depend on concentration of any of the (iii)
reactants, therefore order of reaction is zero.
Molecularity = 2 (As two molecules of reactants take part in the
reaction) 12. (i) Chlorobenzene to biphenyl
(ii) Unit of k = Unit of rate = mol L−1 s −1
Dry ether + –
O CI + 2Na + CI + 2NaCl
⏐⏐
7. (i) R ⎯C ⎯ NH2 + Br2 + 4NaOH ⎯→ R ⎯ NH2 + Na 2CO 3 +2NaBr Biphenyl
Amide 1° amine
+2H2O
(ii) Propene to 1-iodopropane
Here, R = Alkyl or aryl group
Heat CH3CH == CH2 + HBr ⎯Peroxide
⎯⎯ ⎯→ CH3CH2CH2Br
(ii) RNH2 + CHCl 3 + 3KOH ⎯ ⎯
⎯→ R ⎯ NC + 3KCl + 3H2O Dry Acetone
Aliphatic or Aliphatic or CH3CH2CH2Br + NaI ⎯ ⎯⎯⎯⎯→ CH3CH2CH2 I + NaBr
aromatic aromatic (Reflux)
amine isocyanide
(iii) 2-bromobutane to but-2-ene
8. (i) Gas (B) will have higher value of KH . CH3 ⎯ C H ⎯ CH2 ⎯ CH3 ⎯Alc.
⎯⎯ KOH
⎯→ CH3 ⎯ CH == CH ⎯ CH3

Reason According to Henry’s law, Br
Solubility (S ) ∝ Partial pressure ( p)
1 or
and p = KH X ... (i) (where, KH = ) CH2CH3 CH2BrCH3
K′
where, KH = Henry’s constant
Br2, UV light
X = Mole fraction of gas (i)
From eq (i), the gas with lower value of KH will show more
solubility than the gas with higher value of KH .
NO2 NO2
(ii) In non-ideal solution, a large negative deviation from Raoult’s
law shows the formation of maximum boiling azeotropes. Na
(ii) 2CH3—CH—CH3 H3C—CH—CH—CH3
Dry ether
9. (i) Structural formula of the complex =[Co(NH3 )6 ]Cl 3
3 AgNO Cl H3C CH3
[Co(NH3 )6 ]Cl 3 ⎯ ⎯⎯⎯
⎯3
→ [Co(NH3 )6 ]3 + + 3NO −3 + 3AgCl
AgCN
(ii) [Co(NH3 )6 ]Cl 3 (iii) CH3—CH2—Br CH3CH2NC
Hexaamminecobalt (III) chloride.

76
13. (i) Cellulose has β -1, 4-linkages while starch has α-1, 4-linkages Density (ρ ) = 7.5 g cm − 3
between glucose units. Cellulose is mostly found in linear Z×M
chain of glucose molecules while starch is found in both linear Density of unit cell (ρ ) =
a 3 × N0
and branched chains.
(ii) The linkage between nucleotide units in nucleic acids is a where, Z = number of atoms present per unit cell
phosphodiester linkage which connects the 5′-hydroxyl group (For bcc lattice, Z = 2)
of one nucleotide to 3′-hydroxyl group of next nucleotide. M = Atomic mass of the element
(iii) Fibrous protein—Keratin N 0 = Avogadro’s number
Globular protein—Haemoglobin 2 ×M
∴ 7.5 =
( 500 × 10 −10 )3 × 6.023 × 10 23
14. (i) Mond’s process
(ii) Cryolite improves the electrical conductivity of the cell as Al 2O 3 M = 282.328 g mol − 1
is the poor conductor of electricity. It is also added as impurity Number of moles present in 300 g of element
to lower down the melting point of the mixture to about 950°C. 300g
=
(iii) In slag formation zone of blast furnace, limestone 282.328g mol −1
decomposes into lime (CaO) and carbon dioxide (CO 2 ). CaO
= 1.062 mol
acts as a flux and combines with silica to form slag (CaSiO 3 ).
∴ Number of atoms present in 300 g of element
15. (i) SO 2 is a reducing agent because it has empty d -orbitals, so it = 1.062 × 6.023 × 10 23
can expands into + 4 and + 6 oxidation states and acts as a
reducing agent. In TeO 2, Te is a heavier element, its electrons = 6.4 × 10 23 atoms
opposed to take part in the bond formation due to poor 18. (i) In order to improve the physical properties of natural rubber, it
shielding of d and f-orbital electrons. Thus, + 2 and − 2 is heated with sulphur and a suitable additive at a temperature
oxidation states are more stable in case of TeO 2 or it acts as an range between 373 K to 415 K. S-atoms form cross-links at
oxidising agent. reactive sites of double bond making it stiffer.
(ii) Nitrogen does not have empty d -orbitals to expand its octet. (ii)
Hence, it does not form pentahalides.
(iii) ICl is more reactive than I 2 because interhalogen bonds are Polymer = —O—CH2—CH2—O—C— —C—
weaker than diatomic bonds (except F2) due to their different n
electronegativity. This electronegativity difference polarises Dacron or Terylene
the bond between iodine and chlorine and therefore breaks
more easily as compared to I 2.
16. (i) Fe atom : [Ar] 3d 6, 4 s 2 : Monomers=HO—CH 2—CH2—OH+ HO—C— —C—OH

3d 4s 4p 4d Ethylene glycol or
Ethane-1,2-diol Terephthalic acid or
Benzene-1,4-dicarboxylic acid

Fe3+ion:3 d5,4 s0: (iii) The order of intermolecular forces of different classes of
3d 4s 4p 4d polymers is,
Elastomers < Thermoplastics < Fibres.
Thus, for given polymers increasing order of intermolecular forces
Fe3+ ion in [Fe(H2O)6]3+ : sp3d2 is,
3d 4s 4p 4d
Neoprene < Polythene < Terylene .
xx xx xx xx xx xx
19. C 2 H5Cl( g ) ⎯→ C 2 H4 ( g ) + HCl( g )
H2 O H2 O H2 O Initial pressure pi 0 0
H2 O H2 O H 2O After time t pi − p p p
Hybridisation-sp3d2 Total pressure after time t, i.e.
As there are five unpaired electrons present, hence, it is pt = pi − p + p + p = pi + p
paramagnetic in nature and high spin complex. so, a = pi
a − x = p i − ( pt − p i ) = p i − pt + p i = 2 p i − pt
(ii) Cl 2+
Rate constant for first order reaction is given as,
2 .303 a
k= log
en Pt en t a−x
2.303 pi
= log
Cl 300 2 pi − pt
trans-isomer 2.303 0.30
(opticallyinactive) = log
300 [2(0.30) − 0.50]
17. Given, edge length ( a ) = 500 pm 2.303 0.30
= log = 3.66 × 10 − 3 s −1
− 10 300 0.10
= 500 × 10 cm

77
20. (i) Aniline behaves as a catalyst as a Lewis base and forms a (ii) Silver mirror test
salt with Lewis acid (AlCl 3 ) used in Friedel-Crafts reaction. HCOOH gives silver mirror test with Tollen’s reagent, whereas
Due to this, nitrogen atom of aniline acquires a positive acetic acid does not give this test.
charge and hence acts as a strong deactivating group and
HCOOH + 2[Ag(NH3 )2 ] OH ⎯→ 2Ag + 2H2O + CO 2 + 4NH3
thus, does not allow the reaction to take place. Silver
mirror
(ii) In aqueous solution, with increase in number of methyl
group, hydrogen bonding and stabilisation by solvation CH3COOH + 2[Ag(NH3 )2 ]OH ⎯→ No reaction
decreases. Hence, (CH3 )2NH is more basic than (CH3 )3N .
(c) The extent of hydrogen bonding for given compounds is,
(iii) In 1° amines, two hydrogen atoms are present to the nitrogen Acetic acid > Ethanol > Acetaldehyde
atom. Due to the presence of hydrogen atoms, 1° amine
As, hydrogen bonding ∝ Boiling point
undergoes extensive intermolecular hydrogen bonding. In
3° amine, no hydrogen atom is present to the nitrogen atom. Hence, increasing order of boiling point for given compounds is
Hence, 1° amines have higher boiling point than 3° amines. CH3CHO < CH3CH2OH < CH3COOH
21. (i) Lyophilic colloid The colloidal solution in which the particles or
of the dispersed phase have a great affinity for the dispersion (a) H3C
medium is known as lyophilic colloid. These solutions are C CH2—CH3
easily formed and shows reversible nature, e.g. gum, starch
and rubber etc. NH2—NH2+KOH
+ 4[H] + H2O +N2 ↑
(ii) Zeta potential The potential difference that exists between 472 K, glycol
the stationary layer of compensating charges and the diffuse
layer (present in the body of the solution) is called zeta Acetophenone Ethyl
benzene
potential. It is involved in many kinds of non-static electrical
properties of solid-liquid interfaces and with the help of it, we (b) C 6 H5COCH3 < CH3 COCH3 < CH3CHO
observe electrical effects in colloids. ••
(c) In carboxylic acids, lone pairs on oxygen atom of ⎯ O H group
(iii) Associated colloids There are some substances which at ••

low concentrations behave as normal strong electrolytes, but involved in resonance and make the carbon atom of carbonyl
at higher concentrations exhibit colloidal behaviour due to group less electrophilic. Hence, carboxylic acids do not give
the formation of aggregates. The aggregated particles thus reactions of carbonyl group.
(i) ( i − Bu ) AlH
formed are called micelles. These are also known as (d) CH3CH2CH == CH ⎯ CH2CN ⎯ ⎯⎯⎯⎯⎯⎯
4 2

associated colloids. (ii) H 2O

22. Given, WA = 100 g, WB = 4g, M B = 120 g mol − 1 CH3CH2CH == CH ⎯ CH2CHO


i × K b × 1000 × WB (e) I2+NaOH
Elevation in boiling point ,ΔTb = A (CH3CH2CHO) No ppt.
WA × M B
(As MgSO 4 undergoes complete ionisation and gives 2 moles C3H6O
of its constituent ions for each mole).
I2+NaOH
2 × 0.52 × 1000 × 4 B (CH3COCH3) CHI3
= = 0.3466 K
100 × 120 (Yellow ppt.)
Q Boiling point of water = 373.15 K A = CH3CH2CHO
∴ Boiling point of solution = (373.15 + 0.34) K = 373.49 K B = CH3COCH3
3+ 2
23. (i) Mr Khanna shows concern for his friend’s health. 25. (a) Ecell = Ecell
°

2 .303 RT
log
[Al ]
nF 2+ 3
Mr Khanna is a responsible person. He take responsibility of [Cu ]
his friend’s health and take him to the doctor. or
(ii) Antacids Some chemical substances which remove the ° 0.059 (0.01)2
excess acid from the stomach and raise the pH at an 1.98 = Ecell − log
6 (0.01)3
appropriate level are called antacids.
or
e.g. sodium hydrogen carbonate (NaHCO 3 ).
° 0.059
(iii) No it is not advisable to take antacids for a long period of Ecell = 1.98 + log 100
6
time. Use of antacids for a long period of time makes the
0.059
medium alkaline in stomach and trigger the overproduction = 1.98 + × 2 = 1.99 V
of acid. 6
° ~
H , Pd BaSO H N ⎯ OH or Ecell − 2 V
24. (a) (i) CH3COCl ⎯ ⎯⎯⎯⎯⎯
2 -

4
→ CH3CHO ⎯ ⎯⎯⎯
2
⎯→
(Rosenmund reaction)
A
CH3CH == N ⎯ OH (b) In case of A, the protecting film will be effective as long as it is
B intact. When scratches occur at the coating surface, both the
(i) CO PCl metals are exposed to oxygen and iron is preferably oxidised
(ii) CH3MgBr ⎯ ⎯⎯⎯
2
→ CH3COOH ⎯ ⎯⎯
5
→ CH3COCl
(ii) H 3O + A B and rusted. This is because reducing potential of A is more than
that of iron.
(b) (i) Iodoform test
In case of B, the article of iron is connected with more active
C6 H5COCH3 + 4I 2 + 3Na 2CO 3 ⎯→ CHI 3
(Yellow ppt. ) metal like B. The active metal has lower reduction potential than
iron and will lose electrons in preference to iron. Hence, B is
+ C6 H5COONa + 3NaI + 2CO 2 + H2O better for coating the surface of iron.
C 6H5CHO + 4I 2 + 3Na 2CO 3 ⎯→ Do not react

78
or F
O O
(a) Given, κ = 3.905 × 10 − 5 S cm − 1 O
+7
Mn Mn
+7
O Mn
M = 0.001 mol L− 1 O O F F
F
κ × 1000
Molar conductivity ( Λcm ) = Mn2O7 MnF4
M
−5
3.905 × 10 × 1000 (ii) Cr 2 + gets readily convert into Cr 3 + because in + 3 oxidation
= = 39.05 S cm 2 mol − 1 state, it has stable half-filled t 2g orbitals.
0.001
(iii) Cu 2 + shows 3 d 9 electronic configuration while Zn 2 + shows
At infinite dilution molar conductivity
3 d 10 electronic configuration. Due to the presence of one
Λ∞CH 3COOH = Λ∞CH COO − + Λ∞H + unpaired electron, Cu 2 + salts are coloured while Zn 2 + salts
3

= 349.6 + 40.9 = 390.5 S cm 2 mol −1 are white.


Δ
(b) (i) 2MnO 2 + 4KOH + O 2 ⎯⎯ → 2KMnO 4 + 2KCl + 2H2O
Λcm 39.05
Degree of dissociation = = = 0.1
Λ∞m 390.5 (ii) Cr2O 27 − +14H + + 6I − ⎯→ 2Cr 3 + + 3I 2 + 7H2O

(b) Electrochemical cell The device through which chemical or


energy produced in a redox reaction converts into electrical (i) The electronic configuration of Mn is [Ar] 3 d 5 4 s 2. Thus, it
energy is called electrochemical cell. has 5 unpaired electrons and likely to exhibit maximum
The chemical reaction of the cell is reversed and current flows number of oxidation states.
in opposite direction when external potential applied becomes (ii) Cr, due to availability of maximum number of unpaired
°
greater than Ecell of the electrochemical cell. electrons (6 es), which results increases in metallic bond
strength.
26. (a) (i) Mn has the highest oxidation state of +7 with oxygen atoms
Mn forms multiple bonds with oxygen atoms involving two (iii) Sc (Scandium), as it has one 3d and two 4s electrons. So, it
2p-orbitals per oxygen atom and 3d-orbitals of Mn. On the can lose maximum three electrons.
other hand, Mn shows the highest oxidation state of (iv) Mn, because in + 3 oxidation state, it has [Ar]3 d 4 electronic
+4 with F-atoms because it forms single bonds involving configuration. Hence, it readily gives its electrons to gain
one 2p-orbital per fluorine atom and 3d-orbitals of Mn. stable electronic configuration.

CHEMISTRY

New Super Cloth to Mop up Oil Spills


Oil spills at sea, on the land and in your own kitchen could one day easily be mopped up with a new multipurpose
Super cloth (fabric) covered with semi-conducting nanostructures, developed by a team of researchers
The fabric could also potentially degrade organic matter when exposed to light thanks to these semi-conducting
properties. This fabric repels water and attracts oil. Scientists have tested it and found it effective at cleaning up
crude oil, and separating organic solvents, ordinary olive and peanut oil from water. All steps in its production are
easy to carry out and, in principle, production of this fabric could be scaled up to be used on massive oil spills that
threaten land and marine ecosystems. To produce the fabric nylon was used, but in principle any fabric could
work. The commercially available nylon that already had a seed layer of silver woven into it, makes it easier to
carry out the next part of the process i.e., addition of the copper. This fabric is then dipped into a vat where a
copper layer was electrochemically deposited onto it. Now with a copper coating, the fabric is converted into a
semiconducting material with the addition of another solution that causes nanostructures to grow on the fabric's
surface i.e., the key to its enhanced properties. The nanostructures are like tiny rods that cover the surface of the
fabric. Water just runs straight off it but the rods attract and hold oil also, when the fabric is saturated it allows the
oil to permeate where it then acts like a sieve to separate oil and water.
What is particularly exciting is that it is multifunctional and can separate water from other liquids like a sieve, it is
self-cleaning, antibacterial, and being a semiconductor opens up further applicability, Its antibacterial properties
arising from the presence of copper could be used to kill bugs while also separating water from industrial waste
in waterways or decontaminate water in remote and poor communities where water contamination is an issue.
Because it is also a semi-conductor it can interact with visible light to degrade organic pollutants such as those
found in waste water streams. The testing has shown the material is chemically robust but still it is needed to
investigate whether the nanostructures can withstand tough wear conditions or not.

79
THERMODYNAMICS
Thermodynamics is more important topic from engineering entrance exams rather than medical entrance exams
point of view. In engineering, problems related to different types of enthalpy change or entropy change while in
medical, direct formula, spontaneity or phase change related problems are asked. Problem solving strategies for
some important topics are given below.

Enthalpy Change (DH) Note Except bond dissociation, enthalpy change for any other
process is given as,
When problems related to enthalpy change are asked, you can go
Enthalpy change = Sum of enthalpies of all products
through the following steps:
- sum of enthalpies of all reactants
Step 1 Write down all the balanced chemical reactions including But for bond dissociation,
their enthalpy change value.
Enthalpy change = Sum of enthalpies of all reactants
Step 2 Mark these reactions in terms of equation number. - sum of enthalpies of all products
Step 3 Write down the chemical reaction for which we have to
find out enthalpy change value. CHEMISTRY CONCENTRATE
Step 4 Arrange the given equations in such a manner that we PRACTICE PROBLEMS
could get the required equation. ■ The standard heats of formation for CCl4 (g ), H2O(g ), CO2 (g ) and
Step 5 For this purpose, you can apply operations such as HCl(g ) are -25.5, - 57.8, - 94.1 and -22.1 kcal mol-1 respectively at
addition or subtraction of equations and multiplying or dividing by 298 K. Calculate DH ° for the following reaction at 298 K.
an integer to the equation.
CCl4 (g )+ 2H2O(g ) ¾®CO2 (g )+ 4HCl(g ) [-41.4 kcal]
Step 6 Same operations should be applied for given enthalpy ■ The standard enthalpies of formation of CO2 (g ), H2O(l ) and
change values as well. glucose(s) at 25° C are -400 kJ mol-1, - 300 kJ mol-1 and
Step 7 Finally, you can get the required enthalpy change value. - 1300 kJ mol-1, respectively. The standard enthalpy of combustion
per gram of glucose at 25°C is [- 16.11 kJ g -1]
Alternate ■ Using the data provided, calculate the multiple bond energy
(kJ mol-1) of a C ºº C bond in C 2H2 (take the bond energy of a
From Step 4 You can go through the following steps :
C ¾ H bond as 350 kJ mol-1),
Step 4 Apply the formula for enthalpy change ( DH ) of required
2C(s ) + H2 ( g ) ¾® C 2H2 (g ); DH = 225 kJ mol-1
reaction, say
2C(s ) ¾® 2C(g ); DH = 1410 kJ mol-1
aA + bB ¾® cC + dD
H2 (g ) ¾® 2H(g ); DH = 330 kJ mol-1 [815 kJ mol -1]
DH = Sum of enthalpies of all products
- sum of enthalpies of all reactants Entropy Change (DS)
Step 5 Multiply the enthalpies of products and reactants Type 1 When phase transitions occur during a process and entropy
according to the stoichiometric coefficients of required reaction change is asked for overall process. Use the following method.
DH = ( cDHC + dDHD ) - ( aDHA + bDHB ) First write down all the reactions involved in the overall process,
Step 7 Finally, you will get the required enthalpy change value. keeping in mind that during phase transition, temperature does not
change.

80
Suppose a compound A( s ) is heated from temperature T1° C to T4° C
and changes itself into A( g ), the reactions involve are, CHEMISTRY CONCENTRATE
A( s ) ¾® A( s ); DS1 PRACTICE PROBLEMS
T1 T2 ■ Calculate entropy change for the transition of liquid water to steam,
DH fus
A( s ) ¾¾® A(l ); DS 2 DH vap is 40.8 kJ mol-1 at 373 K. [109.38 JK -1 mol-1]
T2 T2 ■ Calculate entropy change when 10 moles of an ideal gas expands
reversibly and isothermally from an initial volume of 10 L to 1000 L
A(l ) ¾® A(l ); DS3 at 300 K. [191.24 JK -1 mol-1]
T2 T3
DH ■ Consider the following reaction,
A(l ) ¾®
vap
A( g ); DS 4 1
T3 T3 Ag 2O(s ) ¾® 2Ag(s ) + O2 (g )
2
A( g ) ¾® A( g ); DS 5 DH and DS for this reaction are 30.56 kJ mol-1 and 66.0 JK -1 mol-1,
T3 T4 respectively. Calculate the temperature at which free energy change
DS for, A( s ) ¾® A( g ); will be
(DG ) will be zero. [463 K]
DS = DS1 + DS 2 + DS3 + DS 4 + DS 5

Type 2 When one quantity changes during a process and DS is


Work Done (W )
asked keeping the other quantity constant, apply Type 1 When reversible isothermal process is mentioned in the
Variable Constant
problem and work done is asked to find out, apply
quantity(ies) quantity(ies) DS
æV ö
In case of expansion, W = - nRT ln ç 2 ÷
Temperature Pressure T
2.303 nC p log 2 è V1 ø
T1
æV ö
T2 In case of compression, W = nRT ln ç 2 ÷
Temperature Volume 2.303 nC V log è V1 ø
T1
Type 2 When irreversible isothermal process is mentioned and
Volume Temperature V2
2.303nR log work done is asked to find out, apply
V1
In case of expansion, W = - p2 (V2 - V1 )
Temperature and Volume æ T p ö In case of compression, W = p2 (V2 - V1 )
2.303 ç nC P log 2 + nR log 1 ÷
pressure è T1 p2 ø Type 3 When reversible adiabatic process is mentioned, work
done is asked, apply
Temperature and Pressure æ T V ö
2.303 ç nC V log 2 + nR log 1 ÷ For both expansion and compression,
volume è T1 V2 ø
1
W= ( p2V2 - p1V1 )
Type 3 When phase change occurs during the process and DS is g -1
asked, apply Type 4 When irreversible adiabatic process is mentioned, work
DH
DS = done is asked, apply
T
For both expansion and compression,
Type 4 When a system at higher temperature T1 and its æ p T - p2T1 ö
W = - pextR ç 1 2 ÷
surroundings at lower temperature T2 , q amount of heat goes è p1p2 ø
irreversibly from system to surroundings and DS is asked, apply
éT -T ù
DS = q ê 1 2 ú CHEMISTRY CONCENTRATE
ë T1T2 û
Q T1 - T2 = + ve, DS > 0
PRACTICE PROBLEMS
Hence, process is spontaneous.
■ Calculate the work done by 2 moles of an ideal gas at 298 K in
reversible isothermal expansion from 10 L to 20 L. [-3434.9 J]
Type 5 When Gibbs free energy change ( DG ), enthalpy change ■ Two moles of an ideal monoatomic gas are compressed
( DH ) and temperature are given, DS is asked, apply adiabatically and reversibly to occupy a volume of 4.48 dm 3 at NTP.
DG = DH - TDS Calculate work done value. (CV = 12.45 JK -1mol-1) [24.775 kJ ]

81
Amazing facts with proper explanation.

• Do you know, a glass containing virtually transparent • Do you know, penta-graphene is the fourth allotrope of
coating of titanium dioxide (15 nm thick), has the ability carbon?
to clean itself? Till now, we know that carbon exists in many allotropic forms such as
Pilkington activ TM , launched in 2001, was the first type of glass to be graphite, diamond and buckminsterfullerene. In these allotropic forms,
produced that has the ability to clean itself. This glass contains a virtually hexagons are the primary building blocks except for C 20 fullerene. Carbon
transparent coating of titanium dioxide, 15 nm thick, which is deposited structures made exclusively of pentagons were not known because of
many exotic properties of carbon are associated with their unique
during the manufacturing process. TiO 2 coating is durable, since it is
structures.
bonded to the glass surface and has two functions that allow it to act as a
self-cleaning glass. In 2014, the researchers at Virginia Commonwealth University and
Universities in China and Japan have discovered a new allotrope of carbon
Mechanism of self-cleaning glasses First TiO 2 absorbs ultraviolet named as penta-graphene, on the basis of analysis and simulations.
photons from sunlight. On absorption of a photon, an electron is
A 2D penta-graphene is metastable carbon allotrope, composed entirely of
promoted from the filled valence band to empty conduction band. This
carbon pentagons. Theoretical calculations confirm that this new allotrope
promoted electron reacts with oxygen adsorbed on the surface, to
of carbon is not only dynamically and mechanically stable, but also can
produce a superoxide ion (O −2 ). withstand temperature as high as 1000 K. Due to its unique configuration,
O 2 + e − ⎯→ O 2− penta-graphene shows an unusual negative Poisson’s ratio and ultrahigh
Once it has been activated in this way, the photoactive form of TiO 2 acts ideal strength than can even outperform graphene.
as an oxidising agent by accepting electrons into the vacancies in the
valence band. It obtains these electrons by oxidation of water, converting

H2O into very reactive hydroxyl radicals (O H).

H2O ⎯→ O H + H + + e −
This also takes the TiO 2 back to ground state and completes the catalytic
cycle. The hydroxyl radicals and superoxide ions both are strong
oxidising agents and are able to oxidise most of the organic molecules Penta-graphene
present in dirt, converting them eventually to CO 2 and H2O .
The second way in which the TiO 2 coating leads to self cleaning relates • Do you know, the percentage of solids in a sample of human
interaction with water molecules. On the surface of the coating, oxygen blood is normally about 45%?
atoms of TiO 2 are protonated, forming hydrophilic OH groups. These Our blood in living tissues is made up of liquids and solids. The liquid part
groups interact with water molecules through hydrogen bonds. As a called plasma is made up of water, salts and proteins. On an
result, rain water spread out onto a thin film on the glass surface and dirt approximation, half of our blood is plasma. The solid part (~45%) of our
is washed off the window in a sheet of water. blood contains red blood cells, white blood cells and platelets.
• Do you know, memory metal exists in two different solid This can be easily demonstrated by placing a specimen of whole blood in a
phases and these phases will remain in their respective test tube with a small amount of oxalate to prevent clotting. If the sample is
phases indefinitely unless these are warmed to 50-60°C? centrifuged or allowed to stand for a sufficient length of time, it will be found
Memory metal (NiTi or nitinol), illustrates the importance of deformations. that the blood cells will settle towards the bottom of the test tube while
If a straight piece of NiTi wire is wound into a spiral, it will remain in the plasma remains on the top. Plasma has pale yellow colour.
spiral shape indefinitely, unless it is warmed to 50-60°C, at this Centrifuged Blood Sample
temperature, it will spontaneously straighten out again.
Composition
The high temperature phase has the cubic calcium chloride structure, in 1. Blood plasma non-cellular
which a Ti-atom is embedded in the centre of a cube of Ni-atoms portion of the blood-55%
(or vice-versa). The low temperature phase has a related but kinked
White
structure, in which one of the angles of the unit cell is no longer 90°. Buffy blood cells
Bending an object at low temperature, creates defects that change the 2. Formed elements (45%) : coat
pattern of kinks within the structure. (i) Red blood cells, RBCs Platelets
(erythrocytes) -99% of
If the object is heated to a temperature greater than 50°C, the material formed elements Red blood
undergoes a transition to the cubic high temperature phase, causing the (ii) Platelets (thrombocytes) cells
object to return into its original shape. The shape of the object above (iii) White blood cells, WBCs
50°C is controlled by a complex set of defects and dislocations that can (leukocytes)
be relaxed by the thermal motion of the atoms.

82
1. Which pair of the following chlorides does not impart 6. BF3 behaves as a Lewis acid and on treatment with
colour to the flame? NH3 , gives an adduct. The hybridisation of N and B in
(a) MgCl 2 and CaCl 2 (b) BeCl 2 and SrCl 2 this adduct
(c) BeCl 2 and MgCl 2 (d) CaCl 2 and BaCl 2 (a) changes from sp2 to sp3 for both N and B
2. Both Be and Al become passive on treatment with (b) changes from sp2 to sp3 for N and remains same for B
conc. HNO3 due to (c) remains same for N but changes from sp2 to sp3 for B
(a) non-oxidising nature of HNO 3 (d) changes from sp3 to sp2 for N and sp2 to sp3 for B
(b) the formation of an inert layer of the oxide on the surface of
the metals 7. Which of the following relation is correct according to
(c) non-reacting nature of the metals Charles’ law?
δV δT
I. ⎛⎜ ⎞⎟ = K II. ⎛⎜ ⎞⎟ =
(d) None of the above 1
3. 0.15 g of an organic compound containing phosphorus, ⎝ δT ⎠ p ⎝ δV ⎠ p V
δT
IV. ⎛⎜ ⎟⎞ = K
gave 0.32 g of Mg2 P2 O7 by the usual analysis. The 1
III. V ∝
percentage of phosphorus in the compound is T ⎝ δV ⎠ p
(a) 61.20% (b) 60.20%
Choose the correct option.
(c) 59.58% (d) 58.59%
(a) Only III (b) I, II and IV
4. The structure of ethyne shows (c) I and IV (d) I, II and III
I. σ-overlaps 8. Total volume of atoms present in a face-centred cubic
II. π-overlaps
unit cell of a metal is (r is atomic radius)
III. cylindrical nature of electron cloud
20 3 24 3 12 16 3
(a) Only II (b) I and II (a) πr (b) πr (c) πr 3 (d) πr
3 3 3 3
(c) Only III (d) All of these
9. For a cell reaction, involving a two-electron change,
5. Which of the following systems are not aromatic?
the standard emf of the cell is found to be 0.295 V at
25°C. The equilibrium constant of the reaction at 25°C
will be
I II III (a) 10 × 1010 (b) 1 × 1010
−10
(a) I and II (b) II and III (c) 1 × 10 (d) 10 × 10 −10
(c) I and III (d) I, II and III

83
10. For an elementary reaction, 2 A + B ⎯→ 3C, the rate of 14. Consider the following Ellingham diagram for carbon.
appearance of C at time t is 1.3 × 10 − 4 mol L−1s −1 . The
rate of reaction at this time is 0
(a) 3.44 × 10 5 mol L−1 s −1 (b) 3.44 × 10 −5 mol L−1 s −1
–100
(c) 4.33 × 10 −5 mol L−1 s −1 (d) 4.33 × 10 5 mol L−1 s −1 C+
1
O2 CO
–200 2
C + O2 CO2
11. The order of dehydration of the following alcohols with –300
H2 SO4 is
–400
CH3 ΔG°
⏐ –500
I. CH3 CH2 ⎯ CH ⎯ OH II. CH3 CH2 ⎯ C ⎯ OH –600
⏐ ⏐
CH3 CH3 –700

III. CH3 CH2 ⎯ CH2 ⎯ OH –800 710


(a) I > II > III (b) II > I > III 500 1000 1500 2000 2500
(c) II > III > I (d) III > II > I T(ºC)

12. Identify the final product from the following reaction: Which of the following statements is incorrect about the
NH3 KOH/Br2
Cyclopropane carboxylic acid A B Ellingham diagram?
Δ
CHCl3/KOH (a) Upto 710°C, the reaction of formation of CO 2 is energetically
C more favourable but above 710°C, the formation of CO is
(a) CHO (b) OH preferred
(b) Carbon can be used to reduce any metal oxide at a
sufficiently high pressure
(c) Carbon reduces many oxides at elevated temperature
(c) NC (d) CN because ΔG° versus temperature line has a negative slope

(d) ΔS ° ⎡C ( s ) + O 2( g ) ⎯→ CO( g )⎤ < ΔS °


1
13. NaNO2 /HCl causes diazotisation of aniline and ⎣⎢ 2 ⎦⎥
results in the formation of diazonium salt. Identify X [C ( s ) + O 2( g ) ⎯→ CO 2( g )]
and Y in the following reaction sequence. 15. Identify the correct product formed during the
following reaction,
Phenol CH3
X Benzene diazonium Y Conc. H 2SO4
chloride
NaOH CH3 A
OH OH
(a) X= CH3 CH3
(a) (b)
O CH3 CH3

Y= N N CH3 CH3
(c) (d)
C—CH3 CH3
(b) X= N N

16. A swimmer coming out from pool is covered with a film


Y= of water weighing about 18 g. Calculate the internal
energy of vaporisation at 100°C.
(c) X= N N OH [Δ vap H° for water at 373 K = 40.66 kJ mol −1 ]
(a) 35.67 kJ mol −1 (b) 37.56 kJ mol −1
(c) 36.57 kJ mol −1 (d) 38.75 kJ mol −1
Y=
17. The major achievement of Bohr atomic model is that it
N N OH explains
(d) X= (a) finer details of atomic spectra
HO (b) splitting of spectral lines in the presence of magnetic and
electric fields
(c) spectra of hydrogen or hydrogen like species
Y= (d) intensity of various spectral lines

84
18. A person adds 1.71 g of sugar (C12 H22 O11 ) in order to 26. 0.5 F of electricity is passed through 500 mL of copper
sweeten his tea. The number of carbon atoms added sulphate solution. The amount of copper which can be
are (molecular mass of sugar = 342) deposited will be
(a) 3.6 × 10 22 (b) 7.2 × 10 21 (c) 0.05 (d) 6.6 × 10 22 (a) 63.5 g (b) 31.75 g

19. Consider the two gaseous equilibria at 298 K (c) 15.80 g (d) unpredictable

1
SO2 (g) + O2 (g) - SO3 (g); K1
27. Which of the following aromatic compounds is least

-
2 reactive towards electrophilic substitution reaction?
2SO3 (g) 2SO2 (g) + O2 (g); K2
(a) (b) (c) (d)
The equilibrium constants are related by N
N O
(a) K 2 = K1 (b) K 2 = K12
1 1 H
(c) K 2 = 2 (d) K 2 =
K1 K1
28. The reagents used to convert
⎛⎜ − Ea ⎞⎟ O
20. In the Arrhenius equation, k = A ⋅ exp ⎝ RT ⎠
, the rate
are
constant C CH CH3
(a) decreases with increasing activation energy and increases
with increasing temperature (a) (i) O 3/Red P (ii) AlCl 3 (iii) MeCOOH
(b) increases with increasing activation energy and (b) (i) H 2SO 4 + HgSO 4 (ii) H 2O, Heat
temperature (c) O 3 / Zn-AcOH (ii) H 2SO 4 + HgSO 4 (iii) H 2O, Heat
(c) decreases with decreasing activation energy and −
(d) (i) CH 3COOH (ii) H 2O 2 + OH / H 2O
temperature
(d) increases with increasing activation energy and decreasing 29. In the Hofmann-bromamide reaction, CO is lost as
(b) HCO È
temperature
(a) CO 2 3
21. Which of the following statements is wrong? (c) CO 2−
3 (d) CO
(a) P 33 is used in the treatment of leukemia
(b) I131 is used in the treatment of thyroid gland cancer 30. Which of the following statements is correct?
(c) Co 59 cannot be used in the treatment of cancer (a) Benzoic acid is a stronger acid than formic acid
(d) Excessive use of radioactive elements is responsible for (b) Benzoic acid is a weaker acid than peroxy benzoic acid
cancerous growth (c) Acetic acid is a weaker acid than p-nitrophenol
22. General configuration of ultimate and penultimate (d) α-methyl butyric acid is a stronger acid than β-methyl butyric
acid
shell is (n − 1)s2 (n − 1)p6 (n − 1)d x ns2 . If n = 4 and x = 5,
the number of protons in the nucleus would be 31. The compound 1-(N-ethyl-N-methyl) propanamine
(a) less than 24 (b) 25 forms non-superimposable mirror images. But this
(c) 24 (d) more than 25 compound does not show optical activity because of
the
23. When hydrogen sulphide is passed through acidified (a) absence of a chiral N-atom
solution of KMnO4 , the permanganate solution is (b) presence of a chiral N-atom
decolourised. In this reaction, change of oxidation (c) presence of lone pair on N-atom
state of manganese is found to be from (d) rapid flipping of one form into the other
(a) + 7 to + 2 (b) + 7 to + 4 32. 1 mg radium has 3.20 × 1018 atoms. Its half-life period
(c) + 2 to + 7 (d) + 4 to + 7 is 1620 yr. How many radium atoms will disintegrate
24. 0.5% aqueous solution of KCl was found to freeze at from 1 milligram of pure radium in 4860 yr?
− 0.24 °C. Calculate the degree of dissociation of the (a) 2.0 × 1018
solute at this concentration. (b) 2.5 × 1018
(Given, K f for water = 1.86 K kg mol −1 ) (c) 2.8 × 1018
(a) 0.92 (b) 0.86 (d) 2.3 × 1018
(c) 0.25 (d) 0.63
33. Which one of the following reactions of xenon
25. The stabilisation of a dispersed phase in a lyophobic compounds is not feasible?
colloid is due to
(a) XeO 3 + 6HF ⎯→ XeF6 + 3H 2O
(a) the adsorption of charged substances on dispersed phase
(b) 3XeF4 + 6H 2O ⎯→ 2Xe + XeO 3 + 12HF + 1.5O 2
(b) the large electro-kinetic potential developed in the colloid
(c) 2XeF2 + 2H 2O ⎯→ 2Xe + 4HF + O 2
(c) the formation of an electrical layer between two phases
(d) the viscosity of the medium (d) XeF6 + RbF ⎯→ Rb[XeF7 ]

85
34. Across the lanthanide series, the basicity of lanthanide 37. Heating of an aqueous solution of aluminium chloride
hydroxides to dryness will give
(a) increases (a) Al(OH)Cl 2 (b) Al 2O 3 (c) Al 2Cl 6 (d) AlCl 3
(b) decreases
38. Which one of the following substances is used in the
(c) first increases and then decreases
laboratory for fast drying of neutral gases?
(d) first decreases and then increases
(a) Sodium sulphate
35. Which one of the following ionic species will impart (b) Phosphorus pentoxide
colour to an aqueous solution? (c) Sodium phosphate
(a) Cu + (b) Zn2 + (c) Cr 3 + (d) Ti 4 + (d) Anhydrous calcium chloride
36. Ammonia forms the complex ion [Cu(NH3 )4 ]2 + with 39. Which base is found only in nucleotides of RNA?
copper ions in alkaline solution but not in acidic (a) Adenine (b) Uracil
solution. What is the reason for it? (c) Guanine (d) Cytosine
(a) In acidic solution, hydration protects copper ions 40. Artificial silk is a
(b) In alkaline solution, insoluble Cu(OH 2 ) is precipitated
(a) polypeptide (b) polysaccharide
(c) Copper hydroxide is an amphoteric substance
(c) polythene (d) polyvinyl chloride
(d) In acidic solution, protons coordinate with ammonia
+
molecules forming NH 4 ions and NH 3 molecules are not
available

Answers with Explanation


1. (c) Be and Mg atoms have very high ionisation enthalpies and In the plane of the paper Below the plan
2px 2px
their valence electrons are not excited to higher energy states π of the paper
by the energy of flame. The chlorides of these elements do not 2py
π
give any colour to the flame.
2. (b) HNO 3 being a strong oxidising agent oxidises both Al and Be
C C
to thin layers of Al 2O 3 and BeO, respectively. These layers
π
protect Al and Be metals from further attack of HNO 3. The II.
reactions involved are π
2py
2HNO 3 ⎯→ 2NO 2 + H 2 O + [O]
Above the plane
Be + [O] ⎯→ BeO In the plane of the paper
of the paper (π-overlaps)
2Al + 3[O] ⎯→ Al 2O 3
3. (c) Given, the mass of the organic compound = 0.15 g
Mass of Mg 2 P2 O 7 formed = 0.32 g III. H C C H
1 mole of Mg 2 P2 O 7 molecules contain 2 moles of P atoms
or 222 g of Mg 2P2O 7 molecules contain 62 g of atoms of Cylindrical nature of electron cloud
P [Molecular weight of
5. (c) I. sp3
Mg 2 P2 O 7 = (2 × 24 + 2 × 31 + 16 × 7) g] = 222 g
Atomic weight of P = 31 g ; Non-aromatic
∴ 0.32 g of Mg 2 P2 O 7 will contain P
Due to the presence of one sp3-hybridised carbon, the system
= ⎛⎜ × 0.32 ⎞⎟ g
62
is not planar. It contains 6 π-electrons but the system is not fully
⎝ 222 ⎠
conjugated as all the 6 π -electrons do not form cyclic electron
But this is the amount of phosphorus present in 0.15 g of the cloud that should surround all the atoms of the ring.
organic compound.
62 0.32 II. ; Aromatic
∴ % of phosphorus = × × 100 = 59.58%
222 0.15
4. (d) Structure of ethyne H ⎯ C ≡≡ C ⎯ H This system contains ( 4n + 2 ) π-electrons where, n = 2, i.e. it
contains 10 π-electrons and hence, according to Huckel’s rule,
sp-s, sp-s, it is an aromatic compound.
C⎯H, σ-bond C⎯H, σ-bond
sp 1s III. ; Non-aromatic
1s sp sp sp

I. H C C H It is not planar but is tub-shaped. Further, it has only


σ σ 8 π-electrons. Therefore, the system is not aromatic since,
sp-sp, C—C it does not contain a planar cyclic electron cloud having
σ-bond ( 4n + 2 ) π-electrons.
(σ-overlaps)

86
6. (c) Due to incomplete octet of B in BF3, it behaves as a Lewis CH 3 CH 3
acid. When it reacts with electron rich species like NH 3, it gives ⏐ H 2 SO 4 ⏐
an addition product. II. CH 3CH 2 ⎯ C ⎯ OH ⎯ ⎯ ⎯ ⎯→ CH 3 CH 2 ⎯ C ⊕
+
+ − ⏐ (H ), − H 2O ⏐
NH 3 + BF3 ⎯→ H 3 N → BF3 ≡≡ H 3 N ⎯BF3 CH 3 CH 3
sp 3 sp 2 Adduct sp 3 sp 3 (3° carbocation)
+ ⊕
H
Hence, the hybridisation of N remains same ( sp3 ) but III. CH 3CH 2CH 2 ⎯ OH ⎯ ⎯→ CH 3 CH 2 C H 2
−H 2O
hybridisation of B changes from sp2 to sp3. (1° carbocation)
7. (b) According to Charles’ law, Since, stability of carbocation follows the order: 3° > 2 ° > 1°.
V ∝ T at constant p Therefore, the correct order of dehydration is II > I > III.
⇒ V = KT 12. (c)
NH3
∴ ⎛ δV ⎞ = K COOH CONH2
⎜ ⎟ Δ
⎝ δT ⎠ p
A
T ∝ V at constant p Cyclopropane
carboxylic acid KOH/Br2
⇒ T = KV
∴ ⎛ δT ⎞ = K
⎜ ⎟
⎝ δV ⎠ p NH2
V B
Also, =K
T CHCl3/KOH
⇒ ⎛ 1 ⎞ (V ) = K
⎜ ⎟
⎝T⎠
NC
Differentiating with respect to V at constant pressure, we get
C
+ V ⎛⎜ − 2 ⎞⎟
1 1 dT
=0
T ⎝ T ⎠ dV 13. (c) r s
1 V dT NH2 N NCl
− =0
T T 2 dV
dT 1 NaNO2
or = HCl
dV V
δT
At constant pressure, ⎛⎜ ⎞⎟ =
1 Benzene diazonium chloride
⎝ δV ⎠ p V Phenol
H (Coupling reaction)
8. (d) Total number of atoms in face-centred cubic unit cell = 4 NaO
1 1
(At corners = 8 × = 1; On faces = 6 × = 3) N N OH
8 2
4 (Y ) (X )
Q Volume of one sphere = πr 3
3 Biphenyl Azo dye
∴ Volume of four spheres
4 16 3 14. (d) Since, ΔG ° = ΔH ° − TΔS °
= 4 × πr 3 = πr
3 3 where, ΔG° = Standard Gibbs’ free energy of the reaction
E ° × n 0.295 × 2 ΔS ° = Standard entropy of the reaction
9. (b) log Keq = cell = = 10
ΔH° = Standard enthalpy of the reaction
0.0591 0.059
∴ Keq = 1 × 1010 T = Temperature
From the above equation, it is clear that ΔG° will be more
10. (c) 2 A + B ⎯→ 3C negative when ΔS ° is less negative (or ΔS ° is high).
1 d[ A] d [ B] 1 d [C ] 1
Rate = − =− =+ In the diagram, ΔG° value for C + O 2 ⎯→ CO is less
2 dt dt 3 dt 2
d [C ] negative than that for
Q = 1.3 × 10 mol L−1 s −1 (Given)
−4
dt C + O 2 ⎯→ CO 2.
1 d [C ] Therefore, ΔS ° would be higher for
∴ Rate of reaction =
3 dt 1
1 C + O 2 ⎯→ CO
= × 1.3 × 10 −4 mol L−1 s −1 2
3
than that for C + O 2 ⎯→ CO 2
= 4.33 × 10 − 5 mol L− 1s − 1
Hence, ΔS ° ⎡C ( s ) + O 2( g ) ⎯→ CO( g )⎤ >
+ ⊕
1
H
11. (b) I. CH 3 CH 2 ⎯ CH ⎯ OH ⎯ ⎯→ CH 3CH 2 ⎯ C H ⎯ CH 3 ⎢⎣ 2 ⎥⎦
− H 2O
⏐ (2 ° carbocation)
CH 3 ΔS ° [C( s ) + O 2( g ) ⎯→ CO 2( g )].

87
15. (d) Pinacol-pinacolone rearrangement The diol is converted 21. (a) I131 is used in the treatment of thyroid gland cancer. Co 59 is
into α-hydroxy ketone, when reacted in the presence of acid, a stable isotope and is not radioactive. Co 60 is used in the
is believed to proceed through rearrangement of carbocation treatment of cancer. Similarly, it is not P 33 but P 32 which is used
as shown, in the treatment of leukemia.
CH3
H2SO4
CH3 22. (b) The element having outer electronic configuration
CH3 CH3 3s 2 3 p6 3d 5 4s 2, has atomic number 25. Therefore, the
OH OH OH OH2 number of protons is 25.
+ +7 +2
23. (a) 2K MnO 4 + 3H 2 SO 4 + 5H 2 S ⎯→ K 2 SO 4 + 2 MnSO 4
CH
O CH3 O⎯H 3 CH3 + H 2 O + 5S
+
CH3
–H 1000 × K f × W 2
CH3 CH3 24. (a) Observed molecular mass of KCl =
OH ΔTf × W1
. K kg mol −1, W 2 = 0.5 g, W1 = 100 g, ΔTf = 0.24
Given, K f = 186
CH3
So, observed molecular mass of KCl
CH3 1000 × 1.86 × 0.5
=
0.24 × 100
The carbocationic rearrangement is due to relief from angle = 38.75 g mol −1
strain. Normal molecular mass of KCl = 39 + 35.5 = 74.5 g mol −1
16. (b) We can represent the process of evaporation as Normal molecular mass 74.5
van’t Hoff factor = = = 192
.

-
Vaporisation
18 g H 2 O( l ) ⎯ ⎯ ⎯ ⎯⎯→ 18 g H 2O( g ) Observed molecular mass 3875.
+ −
18 g KCl K + Cl
Number of moles in 18 g H 2O( l ) = = 1 mol (1 − α ) α α
18 g mol −1 Total number of particles = 1 − α + α + α = 1 + α
Δ vapU ° = Δ vap H ° − pΔV = Δ vap H ° − Δ ng RT i = 1+ α
Assume steam behaves as an ideal gas . = 1+ α
192
Δ vapU ° = ( 40.66) − (1)( 8.314 × 10 −3 )( 373) So, α = 192
. − 1 = 0.92 , i.e. 92% dissociated.
= 40.66 − 3.10 = 37.56 kJ mol −1 25. (c) The stabilisation of a dispersed phase in a lyophobic
17. (c) Spectra of hydrogen or hydrogen like species such as colloid is due to the formation of an electrical layer between
He + , Li 2 + were successfully explained by Bohr model. two phases.
171
. 26. (c) Cu2+ + 2e − ⎯→ Cu …(i)
18. (a) Number of moles of sugar = = 0.005 mol
342 According to Faraday’s first law,
Q1 mole of sugar contains = 12 × 6.02 × 10 23 atoms of carbon when an electric current is passed through an electrolyte,
0.005 mole of sugar will contain = 12 × 0.005 × 6.02 × 10 23 amount of substance deposited is directly proportional to the
quantity of electric charge passed through the electrolyte.
= 0.36 × 10 23 = 3.6 × 10 22 C-atoms
From Eq. (i),
19. (c) Given reactions,
1
SO 2( g ) + O 2( g ) -SO 3( g ); K1 …(i)
Q 1 mol or 63.5 g of Cu is obtained by passing 2 F of electricity.
Mass of Cu obtained by passing 1 F of electricity =
63.5

-
2 g
2
2SO 3( g ) 2SO 2( g ) + O 2( g ); K 2 …(ii) ∴ Mass of Cu obtained by passing 0.5 F of electricity
We can get equation (ii) if we reversed equation (i) and also 63.5
= × 0.5 = 15.80 g
multiplying it by 2. 2
If the reaction (i) is reversed, the equilibrium constant becomes 27. (a) Pyridine is least reactive towards electrophilic substitution
1 1 (SE ) because pyridine is resonance stabilised.
inversed, i.e. . If the equation (having equilibrium constant )
K1 K1 r
is multiplied by 2, the equilibrium constant for the new equation
1 1
becomes the square of , i.e. 2 r r
K1 K1 N N N N
s s s
1 Because of withdrawal of electrons from the ring by N-atom,
∴ K2 = 2
K1 the ring is deactivated, thereby resembling the benzene ring
⎛ Ea ⎞
⎜− ⎟ with nitrobenzene. So, it is least reactive towards SE reactions.
⎝ RT ⎠
20. (a) k = A ⋅ exp (b) and (d) are resonance structures. So, SE reaction takes
Ea Ea
ln k = ln A − or log K = log A − place easily.
RT 2.303RT 28. (b) R ⎯ C ≡≡CH is converted to ketone by catalytic hydration
From the above relation, it can be concluded that the rate
with reagents
constant decreases with increasing activation energy. It
(i) H 2 SO 4 + HgSO 4 and (ii) H 2 O, heat.
increases with the increase in temperature.

88
33. (a) XeF6 has much tendency to hydrolyse. The reverse
(i) H2SO4+HgSO4 reaction is more spontaneous.
C CH
(ii) H2O, Heat
CH3 XeF6 + 3H 2O ⎯→ XeO 3 + 6HF
34. (b) Basicity of lanthanide hydroxides decreases along the
29. (c) In Hofmann-bromamide reaction, CO is lost as CO 32− . lanthanide series from left to right.
RCONH 2 + Br2 + 4KOH ⎯→ RNH 2 + 2KBr + K 2CO 3 + H 2O 35. (c)
30. (b) Benzoic acid is a weaker acid than peroxy benzoic acid. Metal ion Outer electronic configuration
The conjugate base of peroxy acid is peroxy anion Cu + 3 d 10, 4s 0
⎛ O ⎞
⎜ È ⎟ Zn2 + 3 d 10, 4s 0
⏐⏐ ••
⎜ R ⎯ C ⎯O ⎯O • ⎟ , in which negative charge is not resonance
• 3+
⎜ •• ⎟ Cr 3 d 3, 4 s 0
⎜ ⎟
⎝ ⎠ Ti 4 + 3 d 0, 4 s 0
stabilised, as it is resonance stabilised in RCOO È. The peroxy
anion is much more basic and less acidic than carboxylic As only Cr 3 + has unpaired electrons. Hence, it will impart
acids. colour to the aqueous solution.
+
31. (d) Due to nitrogen, inversion or rapid flipping of one form into 36. (d) NH 4 does not act as a ligand due to lack of electron pair on
other. nitrogen atom.
Me
s Me 37. (b) Aqueous solution of AlCl 3 is acidic due to hydrolysis
Et
N

Pr
N

Pr
Et AlCl 3 + 3H 2O - Al(OH)3 + 3HCl
On strongly heating, Al(OH)3 is converted into Al 2O 3.
4860 Δ
32. (c) Number of half-lives in 4860 yr = =3 2Al(OH)3 ⎯⎯→ Al 2O 3 + 3H 2O
1620
38. (d) Anhydrous calcium chloride is used in the laboratory for
Amount of radium left after three half-lives
3
fast drying of neutral gases.
= 1 × ⎛⎜ ⎞⎟ = 0125
1
. mg 39. (b) RNA contains four bases. Three (adenine, guanine, and
⎝2⎠ cytosine) are the same as those in DNA, but the fourth base in
Amount of radium disintegrated = (1 − 0125
. ) g = 0.875 g RNA is uracil instead of thymine (present in DNA).
Number of atoms which have disintegrated 40. (b) Chemically treated cellulose (a polysaccharide) is known
= 0.875 × 3.20 × 1018 = 2.8 × 1018 atoms as silk.

CHEMISTRY

Researchers Take Small Step Toward Silicon-based Life


For life on Earth, carbon is king. All organisms build their cells from carbon-based molecules. Scientists
and science fiction authors have long speculated that because silicon atoms bond to other atoms in a
manner similar to carbon, silicon could form the basis of an alternative biochemistry of life. Yet even
though silicon is widely available on Earth and makes up 28% of the planet’s crust (versus 0.03% for
carbon), the element is almost entirely absent from life’s chemistry. Researchers reported that they have
evolved a bacterial enzyme that efficiently incorporates silicon into simple hydrocarbons—a first for life.
Down the road, organisms able to incorporate silicon into their cells could lead to a novel biochemistry for
life, although for now creating actual silicon-based creatures remains a long way off.
The bacterium is so-called thermophilic bacterium, which grows in hot springs. Like many organisms, the
bacterium contains an enzyme called cytochrome c, which shuttles electrons to other proteins, making it
widely useful in biochemistry. In some cases, however, enzymes in thermophilic bacteria expand their
roles to carry out other reactions on the side. So the Caltech researchers tested their microbe and found
that in rare cases its cytochrome c also added silicon to hydrocarbons.
For now, the silicon-spiked hydrocarbon compounds, called organosilanes, probably aren’t useful either
to the bacteria or to industry. They’re short and stubby, unlike the long chainlike versions that chemical
companies make for uses such as adhesives, caulks, and sealants.

89
1. Which of the following changes are accompanied 6. Derivatives of barbituric acid act as
during the process of adsorption? (a) tranquilisers (b) antibiotics
(a) Decrease in entropy (c) antipyretics (d) analgesics
(b) Decrease in enthalpy 7. Which of the following, on reaction with nitrous acid,
(c) Decrease in free energy followed by treatment with NaOH produces a
(d) All of the above blood-red colouration?
2. What is the energy change for the following nuclear (a) RCH 2 NO 2 (b) R3 CNO 2
reaction? (c) R 2 CHNO 2 (d) PhNO 2
0 n ¾® p + +1 e
1 0
8. An organic compound gives a fruity smell, this
If mp = 100728
. u, mn = 100867
. u compound on distillation in presence of dilute alkali,
. ´ 1010 J
(a) 1676 (b) 01242
. ´ 1012 J yields a product capable of giving positive iodoform
(c) 15.36 ´ 10 8 J (d) 12.86 ´ 1012 J test. Hence, the compound is
(a) methyl formate (b) isopropyl acetate
3. The four quantum numbers of the 19th electron for (c) methyl propionate (d) n-propyl formate
Ti (Z = 22) are
9. Which of the following statements is/are correct about
(a) n = 4, l = 1, m = 0, s = + 1 / 2
s-and p-molecular orbitals?
(b) n = 4, l = 0, m = 0, s = + 1 / 2
I. p-bonding orbitals are ungerade.
(c) n = 3, l = 2, m = + 2, s = + 1 / 2
II. p-antibonding orbitals are ungerade.
(d) n = 4, l = 2, m = - 1, s = + 1 / 2
III. s-antibonding orbitals are gerade.
4. An ore contains 1.34% of the mineral argentite, Ag2 S, Choose the correct option.
by mass. How many grams of this ore would have to be
(a) Only I (b) II and III
processed in order to obtain 1.00 g of pure solid silver,
Ag? (c) Only III (d) Only II
(a) 74.6 g (b) 85.7 g 10. In 21.6 carat gold, the percentage of gold is
(c) 107.9 g (d) 134.0 g (a) 70% (b) 90% (c) 21.8% (d) 10%
5. Which of the following statements is not correct? 11. The E°red of Sn, Fe and Zn are -014
. V, - 0.44 V and
(a) In an exothermic reaction, the enthalpy of products is less - 0.76 V, respectively. Which one of the following
than that of the reactants statements is correct?
(b) DHfus = DHsub - DHvap (a) Corrosion of zinc can be prevented by coating with iron
(c) A reaction for which DH° < 0 and DS > 0, is possible at all (b) Corrosion of zinc can be prevented by tin coating
temperatures (c) Corrosion of tin can be prevented by coating with zinc
(d) DH is less than DE for combustion of carbon to carbon (d) Corrosion of iron can be prevented by coating with tin
dioxide

90
12. Glycerol on treatment with excess HI gives 21. When equal volumes of the following solutions are
(a) 1, 2, 3-triiodopropane mixed, precipitation of AgCl (K sp = 1.8 ´ 10 -10 ) will
(b) 1, 3-diiodopropane occur only with
(c) 2-iodopropane (a) 10 -4 M (Ag+ ) and 10 -4 M (Cl - )
(d) 3-iodopropane (b) 10 -5 M (Ag+ ) and 10 -5 M (Cl - )
13. Which of the following is the correct reactivity order for (c) 10 -5 M (Ag+ ) and 10 -6 M (Cl - )
substitution reaction?
(d) 10 -10 M (Ag+ ) and 10 -10 M (Cl - )
(a) Vinyl chloride > allyl chloride > isopropyl chloride
(b) Allyl chloride > isopropyl chloride > vinyl chloride 22. In which of the following isolations, no reducing agent
(c) Vinyl chloride > isopropyl chloride > allyl chloride is required?
(d) Isopropyl chloride > vinyl chloride > allyl chloride (a) Aluminium from bauxite (b) Mercury from cinnabar
14. The latent heat of vaporisation of water is 9700 cal/mol. (c) Zinc from zinc blende (d) Iron from haematite
If the boiling point of water is 100°C, find out the 23. When gold is dissolved in aqua-regia, which of the
ebullioscopic constant of water. following compounds is formed?
(a) 0.516 K (b) 1.026 K (a) Aurous chloride (b) Chloroauric acid
(c) 10.26 K (d) 1.832 K (c) Auric chloride (d) None of these
15. The rate of a reaction becomes four times when 24. The oxide that gives hydrogen peroxide on treatment
temperature is raised from 293 K to 313 K. The with dilute acid is
activation energy for such a reaction would be (a) PbO 2 (b) Na 2O 2 (c) MnO 2 (d) TiO 2
(a) 50.855 kJ mol -1 25. The shape of XeF4 is
(b) 52.876 kJ mol -1 (a) tetrahedral (b) square planar
(c) 54.855 kJ mol -1 (c) octahedral (d) trigonal planar
(d) 56.855 kJ mol -1
26. Which method of purification is represented by the

-
16. Consider the reaction, following equation?
CaCO3 (s) CaO(s) + CO2 (g) 773 K 1075 K
in a closed container at equilibrium. What would be the Ti + 2I2 ¾¾® TiI4 ¾¾® Ti + 2I2
effect of addition of CaCO3 on the equilibrium (a) Poling (b) van-Arkel
concentration of CO2 ? (c) Cupellation (d) Zone refining
(a) Increases 27. In a closed vessel, the following equilibrium exists at

-
(b) Decreases 85°C,
(c) Data is not sufficient to predict H2 O(l) H2 O(g)
(d) Remains unaffected For the above reaction to proceed in forward direction,
17. Which of the following compounds will exhibit which condition is correct?
geometrical isomerism? (a) DS > 0 (b) DH > 0
(a) C 6H 5CH==NOH (c) DH - TDS = 0 (d) DH - TDS > 0
(b) CH 3CH==CHCH 3 28. The bond angle in water is 104.5° instead of 109.5°, this
(c) HOOC COOH is because of the presence of
(a) two lone pairs of electrons
(d) All of the above
(b) two hydrogen bonds
18. Toluene, on oxidation with KMnO4 gives (c) hydrogen bonding in the molecule
(a) benzaldehyde (d) two covalent bonds
(b) phenol 29. Caesium is the most reactive metal among alkali
(c) nitrotoluene metals because
(d) benzoic acid (a) it exerts considerable attractive force on valence electrons
19. A compound of carbon, hydrogen and nitrogen (b) it is a heavier metal
contains three elements in the ratio of 9 : 1 : 3.5. The (c) its incomplete shell is nearest to the nucleus
empirical formula of the compound is (d) its valence electron has the highest principal quantum
(a) C 2H 4N (b) C 3H4 N number than the valence electron of any of the others
(c) C 3H 6 N (d) C 2H6 N 30. Which of the following is the correct relationship for
20. IUPAC name of the isomer of CH3 ¾ CH==NOH the same gas at two different temperatures with two
would be different rms velocities?
v1 T1 v2 T1
(a) methanamide (a) = (b) =
(b) 1-amino-2-propanone v2 T2 v1 T2
(c) ethanamide v1 T v2 T
(c) = 1 (d) = 1
(d) None of the above v 2 T2 v1 T2

91
31. 10 g of sample of mixture of CaCl2 and NaCl is treated 36. The lowest rate of esterification of acetic acid catalysed
to precipitate all the calcium as CaCO3 . This CaCO3 is by acid will be observed with the alcohol
heated to convert all the Ca to CaO and the final mass (a) CH 3 OH (b) CH 3 ¾ C H ¾ CH 3
of CaO is found to be 1.62 g. The per cent by mass of |
CaCl2 in the original mixture is OH
(a) 32.1% (b) 16.2% (c) 21.8% (d) 12.0% (c) C 2H 5 ¾ C H ¾ CH 3 (d) (CH 3 )2CH ¾ C H ¾ C 2H 5
32. The number of a and b-particles emitted in the | |
OH OH
following nuclear reaction are
90 Th
228
¾® 83 Bi212 37. The correct increasing order for the polarisability of
(a) 4 a and 1 b (b) 3 a and 7b halide ions is
(c) 8 a and 1 b (d) 4 a and 7b (a) F - < I - < Br - < Cl - (b) Cl - < Br - < I - < F -
33. For the half-cell reaction, Cu + 2 e ¾® Cu, graph
2+ - (c) I - < Br - < Cl - < F - (d) F - < Cl - < Br - < I -
between Ered versus log[Cu2+] is a straight line with 38. What will be the temperature when the rms velocity is
intercept 0.34 V. The electrode oxidation potential of four times of that at 300 K?
the half-cell Cu/ Cu2+(01
. M) will be (a) 300 K (b) 900 K (c) 4800 K (d) 1200 K
(a) 0.34 (b) 0.34 + 0.0591 / 2 39. Which one of the following pairs is not correctly
(c) - 0.34 - 0.0591 / 2 (d) - 0.34 + 0.0591 / 2 matched?
34. Following is the graph between logt1 / 2 and log a (a) Terylene ® Condensation polymer of terephthalic acid and
(a = initial concentration) for a given reaction at 27°C. ethylene glycol
Hence, order of reaction is (b) Perspex ® A homopolymer of methyl methacrylate
(c) Teflon ® Thermally stable cross-linked polymer of phenol
and formaldehyde
45º (d) Synthetic rubber ® A copolymer of butadiene and styrene
log t1/2

40. Which among the following is the wrong statement for


Cannizzaro reaction?
log a
(a) In Cannizzaro reaction, the oxidation number of carbon of
(a) 1 (b) 2 (c) 3 (d) 0 ¾ CHO increases as as well as decreases
35. For a reaction, (b) Cannizzaro reaction is a disproportionation process
A(s) + 2 B+ (aq) ¾® A2+(aq) + 2 B (c) Cannizzaro reaction is responded only by the first member
K c has been found to be 1012 . The E°cell is of alkanal series
(a) 0.354 V (b) 0708
. V (c) 0.0098 V (d) 136
. V (d) Aldehydes without a-hydrogen give Cannizzaro reaction

Answers with Explanation


1. (d) For adsorption, DH = - ve (always exothermic), DS = - ve But, the ore contains only 1.34% Ag 2 S.
248 100
(as the randomness decreases in the process). Hence, at low Thus, 1 g of Ag is obtained from ore = ´ g = 85.68 g
temperatures, adsorption is spontaneous as DG is negative. 216 1.34
When,|TDS| > | DH| then DG = - ve. 5. (d) For any reaction to be spontaneous, the value of DH
2. (b) Total mass of the reactant, 0 n1 = 100867
. u should be negative. Combustion of carbon to carbon dioxide
is a spontaneous process for which DH = - ve.
Total mass of the products = 100728
. u
Loss in mass = (100867
. - 100728
. ) u = 0.00139 u 6. (a) The drugs that are given to the patients suffering from
Energy released by the loss of mass of 0.00139 u anxiety and mental tension are called tranquilisers. They help
= 0.00139 ´ 931 MeV the individual to work according to the capacity by removing
= 0.00139 ´ 931 ´ 9.6 ´ 1010 J symptoms of emotional distress.
= 12.42 ´ 1010 J or = 01242
. ´ 1012 J
3. (b) Electronic configuration of Ti (Z = 22 ) is N H
Barbituric acid
1s 2 2 s 2 2 p6 3s 2 3 p6 4s 2 3 d 2
N
Since, 19 th electron lies in 4s-orbital, therefore
n = 4, l = 0, m = 0, s = + 1 / 2 H

4. (b) Ag 2S ºº 2Ag 7. (a) RCH 2 NO 2 + O == N ¾ OH ¾® R ¾ C == N ¾ OH


Nitrous |
248 g 2 ´ 108 g acid
2 ´ 108 g of Ag is obtained from Ag 2 S = 248 g NO 2
Nitrolic acid
248 ´ 1 248
1 g of Ag will be obtained from Ag 2 S = = g Nitrolic acid dissolves in NaOH to give blood-red colouration.
2 ´ 108 216

92
R 16. (d) This is a heterogeneous equilibrium. Hence, equilibrium
|
constant for this is
R ¾C ¾ R + HNO 2 ¾® No reaction
| KC = [CO 2 ]
NO 2 Hence, CaCO 3 has no effect on equilibrium concentration.
R R 17. (d) All the compounds that have restricted rotation about a
| |
R ¾ C ¾ H + HO ¾ N == O ¾® R ¾ C ¾ N == O + H 2O bond, show geometrical isomerism.
| | Oximes of the type
NO 2 NO 2 R ¾ CH==N ¾ OH; C 6 H 5 ¾ C == N ¾ OH
Pseudo nitrole |
Pseudo nitrole does not dissolve in NaOH, but give blue CH 3
colouration. and cyclic compounds like HOOC COOH
PhNO 2 + HNO 2 ¾® No reaction
show geometrical isomerism.
8. (b)
18. (d) CH3 COOH
9. (a) According to molecular orbital theory, p-bonding orbitals
are ungerade. KMnO4
10. (b) 21.6 carat gold contains 21.6 parts of gold in 24 parts of
weight of alloy. Percentage of gold in 21.6 carat gold is Toluene Benzoic ac
21.6
= ´ 100 = 90% 19. (b) The percentage (or mass) ratio = 9 : 1 : 3.5
24
9 1 3.5
11. (c) Ered
° values can be used in protecting the corroded metal by \ Mole ratio = : :
12 1 14
plating the metal with a thin layer of a less easily oxidised metal.
So, corrosion of tin can be easily prevented by coating with Zn. = 0.75 : 1 : 0.25
or = 3: 4:1
12. (c) CH2OH CH2I CH2 Hence, empirical formula = C 3 H 4N
+ 3HI – I2 HI
CHOH CHI CH 20. (c) CH 3 CONH 2 is the isomer of the given structure and its

CH2OH CH2I CH2I


IUPAC name is ethanamide.
Glycerol 21. (a) When ionic product at a time is greater than Ksp , electrolyte
CH3 CH3 CH3
– I2 HI
precipitates.
CHI CH CHI Ionic product is greater than Ksp , only in condition mentioned

-
in option (a), hence, precipitation occurs.
CH2I CH2 CH3
2-iodopropane AgCl Ag+ + Cl -
K = [Ag ][Cl - ]
+
13. (b) Vinyl chloride (most stable) is relatively less reactive due to
resonance. K = 10 -4 ´ 10 -4 = 10 -8
Cl 10 -8 M > 10 -10 ( Ksp )
| _ +
CH 2 ==C H ¬¾® C H 2 ¾ CH==Cl 22. (b) Isolation of mercury from cinnabar is an auto-reduction
While allyl chloride is very reactive due to stabilisation of process. In this process, no external reducing agent is
allyl carbonium ion by resonance. required. The sulphide ores of less electropositive metals like,
+ + Hg, Pb, Cu etc., are heated in air to convert sulphide part of
CH 2==CH ¾ C H 2 ¬¾® C H 2 ¾ CH==CH 2
the ore into oxide or sulphate which then reacts with the
Isopropyl chloride is in between them in reactivity. remaining sulphide ore to give the metal and sulphur dioxide.
R × Tb2 M 2 ´ 373 ´ 373 ´ 18 2HgS + 3O 2 ¾® 2HgO + 2SO 2
14. (a) K b = = = 0.516K
1000 × LV 1000 ´ 9700 2HgO + HgS ¾® 3Hg + SO 2

15. (b) From Arrhenius equation, 23. (b) Reaction of gold with in aqua-regia is
Au + 3HNO 3 + 4HCl ¾® HAuCl 4 + 3NO 2 + 3H 2O
k2 E a æ T2 - T1 ö
log = ç ÷ Chloroauric acid
k1 2.303R è T1 × T2 ø
24. (b) Peroxides, on treatment with dilute acids, give hydrogen
Hence, peroxide.
Ea æ 313 - 293 ö
log 4 = ç ÷ Na 2O 2 + 2HCl ¾® H 2O 2 + 2NaCl
2.303 ´ 8.314 è 293 ´ 313 ø Sodium Hydrogen
peroxide peroxide
2 ´ 0.3010 ´ 2.303 ´ 8.314 = E a æç
20 ö
or ÷
è 91709 ø 25. (b) 5s 5p 5d
11.527 Xe in excited
or Ea = = 52876.1 J mol -1 state
0.000218 sp3d2-hybridisation
= 52.876 kJ mol -1

93
n-1
Due to the presence of two lone pair of electrons, it is square
34. (d) t 1/ 2 µ æç ö÷
1
planar in shape. or t 1/ 2 = k( a )1 - n
è aø
F F log t 1/ 2 = log k + (1 - n) log a
Xe (It represents straight line equation, y = c + mx)
F F Slope = (1 - n) = tan 45° = 1
\ (1 - n) = 1
26. (b) van-Arkel method is used for the purification of Þ n=0
titanium (Ti). 35. (a) We know that,
27. (a) For the reaction mentioned in the problem to proceed in DG° = - 2.303 RT log KC
forward direction, it has to be spontaneous. = - 2.303 RT log 1012
DG = DH - TDS = - 12 ´ 2.303 RT …(i)
For spontaneity, DG = -ve; DH = - ve °
Also, DG° = - nFEcell …(ii)
DS = + ve
or °
-12 ´ 2.303 RT = - nFEcell [from (i) and (ii)]
28. (a) The bond angle in water is 104.5° instead of 109.5°, this is
because of the presence of two lone pairs of electrons. or °
Ecell =çæ 2.303RT ö
´ 12 ÷ V
Lone pair-lone pair repulsion is much greater than lone è nF ø
pair-bond pair repulsion and hence, this repulsion causes the æ

0.059 ö
´ 12 ÷ V (n = 2 )
change in shape. è 2 ø
29. (d) When valence electron(s) is in the highest quantum shell, = 0.354 V
the ionisation energy is lowest and hence, reactivity is highest.
36. (d) In esterification reaction, the oxygen of alcohol donates its
30. (a) For the same gas at two different temperatures, the ratio of lone pair to positively charged C-atom of carbonium ion. But
v T1 this attack does not occur readily in bulky alcohol such as
rms velocities will be, 1 = .
v2 T2 (CH 3 )2CH ¾ C H ¾ C 2 H 5.
|
31. (a) CaCl 2 + NaCl = 10g OH
Let, weight of CaCl 2 = x g Hence, the rate of esterification of acetic acid catalysed by
CaCl 2 ¾® CaCO 3 ¾® CaO acid get lowered.
1 mol 1 mol 1 mol
x x x 37. (d) The increasing order for the polarisability of halide ions is
mol mol mol F - < Cl - < Br - < I -
111 100 56
The size of I - is greater than all the halide anions. As a result,
162
.
Moles of CaO = the electrons are loosely packed and can be easily polarisable.
56
x 162
. 3RT
\ = 38. (c) m (rms velocity) =
111 56 M
x = 3.21 g Hence, mµ T
3.21 For case I, at temperature T,
% of CaCl 2 = ´ 100 = 32.1%
10 3RT
m= …(i)
32. (a) Number of a-particles M
Mass number of emitter - mass number of end - product For case II, at temperature T ¢,
= 3RT ¢
4 4m = …(ii)
228 - 212 16 M
= = =4
4 4 From Eqs. (i) and (ii),
Number of b-particles m T
=
= 2 ´ a-particles - (difference in atomic number) 4m T¢
= 2 ´ 4 - ( 90 - 83) = 8 - 7 = 1
Hence, a - and b-particles emitted are 4 and 1 respectively. 1 300
or = (T = 300 K )
- 4 T¢
33. (d) Cu 2+
+ 2e ¾® Cu
0.059 1 or T ¢ = 4800 K
E = E° - log
Cu 2+ /Cu Cu 2+ /Cu
2 [Cu2+ ] 39. (c) Teflon is thermally stable polymer of tetrafluoroethylene
= E° +
0.059
log[Cu2+ ] (F2 C == CF2 ).
2+
Cu /Cu
2 Polymerisation
nF2C == CF2 ¾¾¾¾¾® ¾[ F2 C ¾ CF2 ¾
]n
Intercept = 0.34 Þ E° = 0.34
Cu 2+ /Cu
40. (c) Cannizzaro reaction is shown by aldehydes lacking of
log 0.1 = æç 0.34 -
0.059 0.059 ö
E 2+ = 0.34 + ÷V a-hydrogen (non-enolisable aldehydes) and seen as
Cu / Cu
2 è 2 ø self oxidation-reduction, i.e. disproportionation. Thus, the
= æç -0.34 +
0.059 ö reaction is given by aromatic aldehydes, HCHO, R 3 CCHO
E = - E 2+ ÷V
Cu /Cu 2+ è
Cu /Cu
2 ø and heterocyclic aldehydes etc.

94
1. A compound B (C10 H12 O3 ) was insoluble in water, 4. Gold metal is extracted from its ore by treating the
dil. HCl and dil. NaHCO3 (aq), it was soluble in crushed rock with an aerated cyanide solution.
dil.NaOH. A solution of B in dil. NaOH was boiled Use any of the following data at 25°C and calculate
and the distillate was collected in a solution of NaOI, DG° for this reaction, Kf for [Au(CN)2 ]- = 6.2 ´10 38 ,
where a yellow precipitate formed. The alkaline Ka for HCN = 4.9 ´ 10-10 and standard reduction
residue in the flask was acidified with dil. H2 SO4 , potential are
a solid (C ) precipitated. When this mixture was O2 (g) + 4H + (aq) + 4e - ¾® 2H2 O(aq ); E° = 1.229 V
boiled, C gets steam-distilled and was collected.
C was found to have formula C7H6 O3 . It dissolved in Au3 + (aq) + 3e - ¾® Au(s); E° = 1.498 V
aq. NaHCO3 with evolution of gas. Identify C. Au3 + (aq) + 2e - ¾® Au+(aq); E° = 1.401 V
OH OH (a) - 387.0 kJ (b) - 400.0 kJ
(a) C6H4 (b) C5H3==C
COOH COOH (c) 387.0 kJ (d) 400.0 kJ
5. Ozone is prepared in laboratory by passing electric
OH COOH
discharge to oxygen gas, 3O2 (g) ¾® 2O3 (g)
(c) C4H3— C==CHCOOH (d) C4H3C==CH— OH Assume that an evacuated vessel of steel with a
2. Dry air is passed through a solution containing 20 g volume of 10.00 L is filled with 32.00 atm of O2 at
of a non-volatile solute in 250 g water. The air was 25°C and an electric discharge is passed through the
then passed through pure water and then through a vessel, causing some of the oxygen to be converted
U-tube containing anhyd. CaCl2 . The mass loss in into ozone. As a result, pressure inside the vessel
solution was 26 g and the mass gain in U-tube was drops to 30.64 atm at 25°C. Calculate the final mass
26.48 g. What is apparent molecular weight of per cent of ozone in the vessel.
(a) 9.08% (b) 12.7% (c) 22.4% (d) 90.7%
solute? If solute is A2 B3 and dissociates in 2 A3 + and
3B2 - ions, calculate degree of dissociation of A2 B3 . 6. Chromium forms three isomeric compounds A, B
Atomic masses of A and B are 24 and 36 gmol -1 and C with per cent compositions 19.52% Cr, 39.91%
respectively. Cl and 40.57% H2 O. When a sample of each
(a) 20% (b) 30% (c) 25% (d) 28% compound was dissolved in water and aqueous
3. A blood-volume determination was carried out on a AgNO3 was added, a precipitate of AgCl formed
patient by injection with 20.0 mL of blood that immediately. A 0.225 g sample of compound A gave
had been radioactively labelled with Cr-51 to 0.363 g of AgCl, 0.263 of B gave 0.283 g of AgCl and
an activity of 4.10 mCi / mL. After a brief period to 0.358 g of C gave 0.193 g of AgCl. Which of the three
allow for mixing in the body, blood was drawn from compounds is likely to be the violet one?
(a) A (b) B
the patient for analysis. Unfortunately, a mix up in
(c) C (d) None of these
the laboratory prevented an intermediate analysis
and it was not until 17.0 days later that a 7. A non-stoichiometric iron oxide with the empirical
scintillation measurement on the blood was made. formula Fe xO, (where x is a number slightly less
The radiation level was then determined to be than 1) can be regarded as FeO in which some of Fe
0.00935 mCi / mL. If 51 Cr has t1 / 2 = 27.7 days. What is sites are vacant. It has a density 5.75 g cm -3 , a
the volume of blood in the patient? cubic unit-cell with an edge length of 431 pm and a
(a) 5.93 L (b) 6.02 L (c) 5.73 L (d) 5.20 L face-centred cubic arrangement of oxygen atoms.

95
Using X-rays with a wavelength of 70.93 pm, at and aq. K4 [Fe(CN)6 ], a chocolate precipitate H is
what angle would third-order diffraction be obtained. On addition of an aqueous solution of
observed from the planes of atoms that coincide BaCl2 to an aqueous solution of E, a white
with the faces of the unit cells? precipitate insoluble in HNO3 is obtained. Identify
(a) 20.0 ° (b) 14.3 ° . °
(c) 167 (d) 15.5 ° the compound H.
8. Consider the following statements. (a) Cu2[Fe(NH 3 )6 ] (b) Cu2[Fe(CN)6 ]
I. The ejection of a beta particle always means (c) CuS (d) [Cu(NH 3 )4 (NO 3 )2 ]
that a different element is formed because a 10. Consider the following reaction,
neutron has been converted into a proton. Br
Phthalimide + KOH (alc.) Δ
II. The smaller is the binding energy per nucleon, A Δ
B
the greater is the stability of the nucleus. H2O/H+,
Δ
III. All nuclei are thermodynamically unstable
with respect to iron-56. C+D
IV. One of the species in the uranium-238 series in The product D is
radon-222, an alpha emitter.
Choose the correct option regarding correct
statements. COOH C
(a) I, II and III (b) II, III and IV (a) (b) O
(c) I, III and IV (d) All of these C
9. A black coloured compound A on reaction with COOH
dil. H2 SO4 gives a gas B which on passing in a
solution of an acid C gives a white turbidity D. Gas B
when passed in an acidified solution of compound E C COOH

gives a precipitate F which is soluble in dil. HNO3 . (c) NH (d)


After boiling this solution when an excess of C COOH
NH4 OH is added, a blue coloured compound G is
formed. To this solution on addition of acetic acid

KNOWLEDGE
Coefficient
Quizzer (No. 18)

Winner of Knowledge Coefficient Chemistry Spectrum


Quizzer (No. 17) (April Issue) Arihant Media Promoters
C/O Arihant Prakashan
Satish Chopra (Hathras) Kalindi, T.P. Nagar, Meerut (UP)-250002

96

You might also like